X



トップページ数学
1002コメント414KB
分からない問題はここに書いてね447
レス数が1000を超えています。これ以上書き込みはできません。
0003132人目の素数さん垢版2018/09/17(月) 00:39:47.36ID:T7a194so
削除依頼を出しました
0004132人目の素数さん垢版2018/09/17(月) 09:47:40.16ID:sCsU7dE3
高2のベクトルの問題です
「原点O,点A(1,0,0),B(0,1,0)C(0,0,1)を頂点とする四面体OABCについて↑OA=a ↑OB=b ↑OC=cとする。
四面体OABCの体積とそれに内接する球の体積を求めよ。」
四面体の体積は簡単ですが球の体積がわからないです。
多分、四面体の内心を求めて球の半径を出すんだと思いますがやり方がわからないです。
解説お願いします。
0005132人目の素数さん垢版2018/09/17(月) 09:59:37.42ID:DDPvQ5//
内心の座標は求める必要ないです
平面における三角形の内接円の半径の求め方と同じようにして求めることができます
もっと具体的にいうと、体積についての等式を導きましょう
0006132人目の素数さん垢版2018/09/17(月) 10:10:13.35ID:sCsU7dE3
なるほど
四面体の体積=1/3*四面体の表面積*内接球の半径
を使うと半径が求められました。
頑張って内心を求めようとしてたのはアホでしたね(笑)
ありがとうございました。
0007132人目の素数さん垢版2018/09/17(月) 10:44:40.67ID:iDwWzM3i
>>4
平面ABCの式は x+y+z = 1,
内心 O(r,r,r)
△ABCの中心は H(1/3, 1/3, 1/3)
 OH = (1/3 - r)√3 = r,
 r = 1/(3+√3) = 0.211325

>>5
4面体の体積 V = 1/6,
儖AB = 儖BC = 儖CA = 1/2,
△ABC は1辺√2 の正△だから (√3)/2,
表面積 S = (3+√3)/2
 r = 3V/S = 1/(3+√3) = 0.211325

>>6
この問題では内心 O(r,r,r) なので、どちらでも同じ。
0008132人目の素数さん垢版2018/09/17(月) 10:49:52.24ID:uN/iN5jq
x^3+y^3=x^2+42xy+y^2 を満たす正の整数の組(x,y)をすべて求めよ

学校の宿題で出されました
全く歯が立ちません(><)
宜しくお願いしますM(__)M
0010132人目の素数さん垢版2018/09/17(月) 12:20:58.93ID:iDwWzM3i
>>8

x^3 + y^3 ≧ (1/4)(x+y)^3,

xx +42xy +yy ≦ 11(x+y)^2,

これらを与式に入れて
 x+y ≦ 44,

(x, y) = (1, 7) (7, 1) (22, 22)
0011132人目の素数さん垢版2018/09/17(月) 15:13:23.72ID:nHx7kmYQ
Haskell先生に探してもらいました。

*Main> print [(x,y) | x <- [1..1000], y <- [1..1000], x*x*x + y*y*y == x*x +42*x*y +y*y ]
[(1,7),(7,1),(22,22)]
0012132人目の素数さん垢版2018/09/17(月) 16:03:45.11ID:nHx7kmYQ
C言語に1万以下の正整数で探してもらいました。

http://codepad.org/ZZXRqHX7

off lineでも10万個にしてみました。

C:\pleiades\workspace\xy\Debug>xy 100000 100000
xy 100000 100000
1 : x = 1.000000, y = 7.000000
2 : x = 7.000000, y = 1.000000
3 : x = 22.000000, y = 22.000000
0013132人目の素数さん垢版2018/09/17(月) 18:16:27.49ID:8acpOrP5
自殺をしたら、地獄に落ちて苦しむか、生前よりもさらに辛い状態で生まれてくるか、
生前にクリアできなかった課題と全く同じ課題をクリアするために、
再び生まれてくることになるのでしょうか?
0014学術垢版2018/09/17(月) 18:26:22.39ID:jlhqH3K5
自殺は憑き物のコーチの質の良さが分かれ道。無神論者になるもよし、有神論なら
赤い悪魔が先達。サッカーコーチでも自殺点に無理解ではない。
0015132人目の素数さん垢版2018/09/17(月) 18:32:47.87ID:lVBV3yvT
>>13
「生きがいの創造」読んでみ
生まれ変わりが科学的に立証されてるって国立大学の教授がエピソード交えて喋ってるぞ
0016132人目の素数さん垢版2018/09/17(月) 18:35:34.72ID:vfa2x310
390×545の長方形の紙から117×156の長方形を出来るだけ多く切り取りたいです。但し、切った紙は糊やテープなどで貼り合わせる事は出来ません。

長辺を77切って捨てれば10枚切り取れますが、なんとか11枚切り取る方法はありませんか?ないとしたら、どうやって証明すればいいですか?

また、一般にm×nの長方形からa×bの長方形を切り取る最大の枚数を求める方法はありますか?
0018132人目の素数さん垢版2018/09/17(月) 19:55:29.73ID:UGjqumaZ
>>16
その手のやつは packing problem というやつで計算アルゴリズムは存在はするけど、実用的な速度で動くものはないと思う。
packing problem でググってみましょう。
0019132人目の素数さん垢版2018/09/17(月) 20:40:58.80ID:lVBV3yvT
>>17
アマゾンのレビュー見ればいい
評価自体はクソ高い
経済が専門の国立大学教授が生まれ変わりをテーマに生きがいを語る
っていうか人生観が変わったって言う色んな人のエピソードを紹介するのがメインの本
著者のスタンスとしては「この世は人間は生まれ変わっている。それは科学的に証明されている。
詳しくは巻末の各種論文を見てね。こう言う話をするとインチキ霊媒師とかのインチキ話も入りがちだから
参考に上げる論文はまともなアカデミックの論文だけだから信用性は大丈夫。」ってな感じ
で、そういう断りをしておいて、内容は「僕は先生の論文を読んだおかげで人生観が変わりました。あざっす」っていうお礼の手紙を紹介するのがほぼ全部
0021132人目の素数さん垢版2018/09/18(火) 02:11:37.75ID:tjDhjgNx
wolfram様によれば解析的に解けないらしい
テイラー展開で近似する方法はある
0024132人目の素数さん垢版2018/09/18(火) 04:03:31.11ID:VmGjAMY2
>>20

分子を (√B) e^(-Ayy/2) = Y とおく。 被積分函数をマクローリン展開して

√{B e^(-Ayy)} / {1 + B e^(-Ayy)}^(1/4)

 = Y / (1+YY)^(1/4)

 = Y -(1/4)Y^3 +(5/2^5)Y^5 -(15/2^7)Y^7 +(195/2^11)Y^9 -(663/2^13)Y^11 +(4641/2^16)Y^13 -(16575/2^18)Y^15 +(480675/2^23)Y^17 - …,

項別に積分すると

∫[0, x] Y^k dy = B^(k/2)∫[0, x] e^(-k・Ayy/2) dy = B^(k/2)・√(π/2kA)・erf(√(kA/2)・x),
0025132人目の素数さん垢版2018/09/18(火) 06:28:37.17ID:hOW38KGZ
Xを位相空間、pt∈Xとする
このとき1次ホモロジー群H_1(X,pt)とH_1(X,∅)が同型なことはEilenberg-Steenrodの公理系からどのようにして示せるでしょうか?

長完全系列
...→H_n(pt,∅)→H_n(X,∅)→H_n(X,pt)→H_n-1(pt,∅)→...から、
n≧2ではH_n(pt,∅)=0だからH_n(X,∅)とH_n(X,pt)は同型
n=0では分裂するのでH_n(X,∅)はH(X,pt)⊕H(pt,∅)と同型
までは分かるのですが、n=1のときが分かりません
0026132人目の素数さん垢版2018/09/18(火) 08:38:44.63ID:3cV882Ep
>>8
[第1段]:x^3+y^3=x^2+42xy+y^2 …@ の両辺はxとyの対称式だから、
(x,y) の存在性の考察や、もし (x,y) が存在するとしたときに (x,y) を求める考察では、x≧y≧1 としても一般性は失わない。
仮に、@ を満たすような正の整数の組 (x,y) が存在するとする。
1):x=y=1 とすると、@ の等号は成り立たないから (1,1) は不適。
2):(x,y)=(2,1) とすると、同様に、@ の等号は成り立たず (2,1) は不適。
3):(x,y)=(2,2) とすると、同様に、@ の等号は成り立たず (2,2) は不適。
4):x≧3、y=1 のとき。このとき、@ から x^3=x^2+42x だから、x≠0 から x^2−x=42。従って、x(x−1)=42 となる。
故に、x=7。逆に (x,y)=(7,1) は @ を満たす。故に、(x,y)=(7,1) は適する。
[第2段]、5):x≧3、y≧2 のとき。m=x+y とおく。x^3+y^3=m(x^2−xy+y^2) で、x^2−xy+y^2>0 だから、@ から、
m=(x^2+42xy+y^2)/(x^2−xy+y^2)=1+43xy/(x^2−xy+y^2) …A
で x^2−xy+y^2≧xy>x,y、従って x^2−xy+y^2 は2正整数 x,y のどちらをも割り切らない。
故に、x^2−xy+y^2 は 素数43 か 43x か 43y か 43xy のどれかを割り切る。
0027132人目の素数さん垢版2018/09/18(火) 08:43:48.37ID:3cV882Ep
>>8
(>>26の続き)
[第3段]:或る (x,y) が存在して x^2−xy+y^2 は 素数43 か 43x か 43y のどれかを割り切るとする。
5-1):x^2−xy+y^2 が43を割り切るとき。x≧y≧2 としているから x^2−xy+y^2=43 …B となる。
x≧3、y≧2 としているから、y^2 の値は4、9、16、25、36の何れかの値になる。従って、yの値は2、3、4、5、6の何れかになる。
5-1-1):y=2 のとき。このとき B から x^2−2x=x(x−2)=39。
39は 39=3・13 と素因数分解出来るから、xの値は存在しない。よって、矛盾。
5-1-2):y=3 のとき。このとき B から x^2−3x=x(x−3)=34。
34は 34=2・17 と素因数分解出来るから、同様に、xの値は存在しない。よって、矛盾。
5-1-3):y=4 のとき。このとき B から x^2−4x=27。しかし、x^2−4x−27=0 の
2解 x=2±√31 はどちらも正整数ではないから、正整数xについて矛盾が生じる。
5-1-4):y=5 のとき。このとき B は x^2−5x=18 となる。しかし、x^2−5x−18=0 の2解
x=(5±√97)/2 はどちらも正整数ではないから、正整数xについて矛盾が生じる。
5-1-5):y=6 のとき。このとき B は x^2−6x=7 となる。従って、x^2−6x−7=(x−7)(x+1)=0 から、x=7。
しかし、(x,y)=(7,6) のときは @ つまり x^3+y^3=x^2+42xy+y^2 について、
(左辺)−(右辺)=7^3+6^3−(7^2+42・6・7+6^2)=7^2・(7−1)+6^2・(6−1)−42・6・7
        =7^2・6+6^2・5−42・6・7
        =49・6+36・5−42^2=294+180−42^2=474−42^2
        ≠0
となって、(x,y)=(7,6) のときは @ が成り立たない。よって、矛盾が生じる。
5-1-1)〜5-1-5) から、x^2−xy+y^2 が43を割り切るとき、何れの場合も矛盾が生じる。
0028132人目の素数さん垢版2018/09/18(火) 08:53:41.24ID:3cV882Ep
>>8
(>>27の続き)
5-2):x^2−xy+y^2 が43xを割り切るとき。このとき、或る正整数nが存在して、n(x^2−xy+y^2)=43x となる。
よって、nは素数43か正整数xのどちらかを割り切る。
5-2-1):nが素数43を割り切るとき。43の正の約数は1と43の2つに限るから、n=43 としてよい。
そこで、n=43 とすると、x^2−xy+y^2=x、従って x(x−y−1)+y^2=0。x≧y≧2 としているから x<y+1、
故に x=y から、x^2−x=x(x−1)=0。しかし、これを満たすxは存在せず矛盾する。
5-2-2):nがxを割り切るとき。xの最大の約数はxなることに着目すると n=x としてよい。そこで、n=x とすると、x^2−xy+y^2=43、
ゆえに x^2−xy+y^2 は43を割り切る。しかし、5-1)のときと同様に考えると、矛盾が生じることになる。
5-2-1)、5-2-2) から、nについて何れのときも矛盾が生じる。
故に、x^2−xy+y^2 が43xを割り切るとき、正整数nは存在しないことになって、矛盾が生じる。
5-3):x^2−xy+y^2 が43yを割り切るとき。x≧y, x≧3, y≧2 としているから、5-2) と同様に考えると、矛盾が生じる。
5-1)、5-2)、5-3) から、何れのときも矛盾が生じるから、x^2−xy+y^2 (x≧y≧2, x≧3) が
43、43x、43y のどれかを割り切るようなxとyの組 (x,y) (x≧y≧2, x≧3) は存在しない。
0029132人目の素数さん垢版2018/09/18(火) 08:59:41.55ID:3cV882Ep
>>8
(>>28の続き)
[第4段]、5-4):x≧3、y≧2 であって、x^2−xy+y^2 が43xyを割り切るとき。
[第2段] までの議論に従い @ を満たす組 (x,y) が存在するとする。すると、x^2−xy+y^2>x,y であって、
x^2−xy+y^2 は x,y のどちらをも割り切らない。また、x^2−xy+y^2 は43、43x、43y の何れをも割り切らない。
43xy の約数をすべて挙げると43、x、y、43x、43y、xy、43xy となるから、x^2−xy+y^2 は xy か 43xy のどちらかを割り切る。
5-4-1):x^2−xy+y^2 が xy を割り切るとき。すると、xy の最大の約数は xy なることに着目すると x^2−xy+y^2=xy としてよい。
そこで、x^2−xy+y^2=xy とすると、(x−y)^2=0 となって、x=y を得る。従って、A から、
m=1+43xy/(x^2−xy+y^2)=1+43x^2/(x^2−x^2+x^2)=1+43=44。
m=x+y としていたから x+y=44 であり、x=y=22。逆に、(x,y)=(22,22) は @ を満たすから、(x,y)=(22,22) は適する。
5-4-2):x^2−xy+y^2 が 43xy を割り切るとき。x^2−xy+y^2 は43を割り切らないから、5-4-1)の議論に帰着される。
5-4-1)、5-4-2) から、@ を満たす正整数 x,y の組は (x,y)=(22,22)。
[第5段]:5-1)、5-2)、5-3)、5-4) から、x≧3、y≧2 (x≧y) のとき @ を満たす正整数 x,y の組は (x,y)=(22,22)   ( 5:x≧3、y≧2 のとき終わり )。
1)〜5) から、x≧y≧1 とした上での @ を満たす正整数 x,y の組は (x,y)=(7,1)、(22,22)。
[第6段]:@ の左辺 x^3+y^3 と @ の右辺 x^2+42xy+y^2 がxとyの対称式なることに注意して x≧y≧1 としていたから、
はじめに y≧x≧1 として上と同様に考えれば、@ を満たす正整数 x,y の組は (x,y)=(7,1)、(1,7)、(22,22) の3つ。
0031132人目の素数さん垢版2018/09/18(火) 09:11:13.54ID:zz7LfpDa
自分で解いた解答がださいと思ったので書かなかったが、遥かに上を行くのが現れた
0034132人目の素数さん垢版2018/09/18(火) 10:02:34.07ID:VmGjAMY2
>>9

43/3 = a とおく。

0 = x^3 + y^3 - (xx+42xy+yy)

= {x+y+(42-2a)}{xx-xy+yy -a(x+y) +a(42-2a)} - a(42-2a)^2

= X^3 + Y^3 +42(XX-XY+YY) -a(X+Y) -aa(4a-42)

= (X+Y+42)(XX-XY+YY -a) - a(42-2a)^2,

ここに、X = x-a, Y = y-a,

漸近線: x+y+(42-2a)=0 (X+Y+42=0)

>>33

4(x^3 + y^3) - (x+y)^3 = (x+y){4(xx-xy+yy) -(x+y)^2}
 = (x+y){3(x-y)^2}
 ≧ 0
0035132人目の素数さん垢版2018/09/18(火) 10:05:24.72ID:AUgeu19y
高千穂交易 イスラエルのスーパースマート社の新世代チェックアウトシステム「Supersmart」の取り扱いを開始
0039132人目の素数さん垢版2018/09/18(火) 11:19:23.67ID:hOW38KGZ
>>38
モニックになるところまでは分かっていたのに0→ H_1(X,∅)→H_1(X,pt) → Ker=0がつくれていることに気がついていませんでした
ありがとうござます!
0040132人目の素数さん垢版2018/09/18(火) 11:21:13.08ID:0aLrbzrN
3行目までの記述は厳密には
w=z/(z+1)、w≠1⇒ z = -w/(w-1)
だけど受験数学ではこの記述が
「逆にw≠1のとき、z = -w/(w-1)とおけば先の変形を逆にたどってw=z/(z+1)になる。」…@
と読んでもらえる。
もちろんこんなの厳密な数学の文章としてはアウト。
しかしそれは数学の教科書ではなく、受験数学の教科書だから、受験数学では書かなくても許してくれることを ”模範” 解答に書くことはない。
@のような拡大解釈は日本の長い受験制度のなかで”defuct standard”(=既成事実化された標準)として定まって来たものだから覚えとくしかない。
べつにそれは利用しなくてもいい事だから覚える必要もないけど。
0041132人目の素数さん垢版2018/09/18(火) 11:43:24.16ID:48smdFkf
>>40

誤: defuct standard
正: de facto standard
0043132人目の素数さん垢版2018/09/18(火) 14:01:29.36ID:Gqtu9UtM
a,bを実数とする。
媒介変数θ(0≦θ<2π)を用いて
x=acosθ+bsinθ
y=bcosθ-asinθ
と表されるxy平面上の曲線Cについて、以下の問に答えよ。

(1)Cが一点または線分になるときのa,bの値または条件を求めよ。答えのみでよい。

(2)C上の点のx座標の最小値をm、最大値をMとする。直線x=t(m≦t≦M)とCの交点の個数を求めよ。
0044132人目の素数さん垢版2018/09/18(火) 16:49:42.63ID:VmGjAMY2
>>43

(1)
 xx + yy = aa + bb,
 Cが一点となるのは a=b=0 のとき。それ以外は円周になる。
 線分にはならない。

(2) m = -√(aa+bb), M = √(aa+bb),
 t=m のとき 1個  (x, y) = (m, 0)
 m<t<M のとき 2個 (x, y) = (t, ±√(aa+bb-tt))
 t=M のとき 1個  (x, y) = (M, 0)
0045132人目の素数さん垢版2018/09/18(火) 18:40:43.54ID:I+fCkgCe
>>26-29って誤答おじさんだよね?
0046132人目の素数さん垢版2018/09/18(火) 18:52:24.72ID:DmF3CBzT
f(x,y)=1/(1+x^2+y^2)を(0,0)まわりでテイラー展開せよ
わからないのでどうかお願いします
0049132人目の素数さん垢版2018/09/18(火) 19:36:30.83ID:onEza3By
>>29
xとyが互いに素だと仮定してない?
互いに素ではなくない?

>43xy の約数をすべて挙げると43、x、y、43x、43y、xy、43xy となるから
0051132人目の素数さん垢版2018/09/18(火) 20:12:14.97ID:I+fCkgCe
>>49
昔から馬鹿で有名な誤答おじさんに何言っても無駄
0053132人目の素数さん垢版2018/09/18(火) 20:31:58.49ID:6aLe7Rjk
a>0として

∫(∞→a) -1/x^2 dx =[1/x](∞→a)= 1/aですよね?

起点の∞では-0に近づき、全域で常にマイナスのものを積分したのに、求めた面積が正になってしまうのはなぜですか?
0054132人目の素数さん垢版2018/09/18(火) 20:34:00.16ID:6aLe7Rjk
いや、単純に、aから∞まで積分するのの逆だからか………

いやでもなんでマイナスになるんだ……?

積分範囲を逆転させて常に負の関数を積分すると正の値が出るのはなぜですか?

図形的にはどういう意味があるんですか?
アホな質問ですみませんがお願いします
0055132人目の素数さん垢版2018/09/18(火) 21:08:54.44ID:48smdFkf
問7

同値な正方行列のトレースは等しいこと、すなわち

tr(P^(-1) * A * P) = tr(A)

を示せ。

この解答を見てみたところ、この問題よりも前の問題である問3と問5より明らか、と書いてありました。

同値な正方行列の固有多項式は等しいから、問5のみから明らかだと思います。

問3はどこで使うのでしょうか?


問3
n 次正方行列 A, B, C について、 A と B、 B と C が同値ならば A と C は同値であることを示せ。


問5
A の固有多項式を g_A(t) = t^n + a_(n-1) * t^(n-1) + … + a_1 * t + a_0

とするとき、

a_(n-1) = -tr(A)
0056132人目の素数さん垢版2018/09/18(火) 21:51:21.74ID:Jq2Da5XV
>>8
事実上 >>10 で終わってるけど。
p = x+y、q=xyとおいて >>10 より 2≦ p ≦ 44。
与式より
p^3 - p^2 - q(3p+40) = 0。
∴q = (p^3 -p^2)/(3p+40)。
∴27q = (9p^2-129p+1720)-68800/(3p+40)
∴3p+40 は46以上172以下の3でわって1余る68800の約数。
68800 = 2^6・5^2・43
であるから
3p+40 = 2^a 5^b 43^c とおくとき (a,b,c) = (6,0,0),(5,1,0),(2,2,0),(2,0,1)。
それぞれで(d,p,q) = (64,8,7),(100,20,76),(160,40,390),(172,44,484)。
このうちx^2 -px +q = 0が整数解をもつのは(p,q) = (8,7),(44,484)のとき。
0057132人目の素数さん垢版2018/09/18(火) 21:51:38.42ID:ywgy1XuA
問題
A,B,Cのカードが2枚、D,E,F,Gのカードが各1枚、合計10枚ある。このカードを無作為に横一列に並べたとき、左から2枚目がBのカードでかつ3枚目がEのカードである確率はいくらか。

解答
B,Eのカード以外はどのカードも関係ないので、それをまとめてXのカードとします。10枚のカードの中にBのカードが2枚、Eのカードが1枚、Xのカードが7枚あると考えましょう。
並べ方の総数は、同じものを含む順列の公式を用いて、
10!/2!1!7!=360(通り)です。
左から2枚目がBのカード、左から3枚目がEのカードであるのは、他の場所に残りのカード(B1枚、X7枚)を並べればよいので、
8!/1!7!=8(通り)
したがって、求める確率は、
∴8/360=1/45

なぜ、B,E以外のカードをまとめてXのカードとして考えるのか、理解できる人いますか?30歳の私に教えてください。
0059132人目の素数さん垢版2018/09/18(火) 22:19:04.92ID:zz7LfpDa
>>56
俺の解答と同じだ
上手く 3p+40 を上から押さえないと手計算放棄決定
というわけで、やっぱり実質>>10で終わってるな
0060132人目の素数さん垢版2018/09/18(火) 22:40:40.93ID:bTifwJNg
計算機でやっても何年にもなりそうとかならともかく、”x+y≦44を満たす正の整数の組” ぐらいまで絞れたら実質終了だね。
0061学術垢版2018/09/18(火) 23:01:41.80ID:bdccv7Cm
マイナスとマイナスじゃ超マイナスなはず。
0062132人目の素数さん垢版2018/09/18(火) 23:02:23.19ID:dotA1T5U
>>8
A=x+y,B=x-y とおけば、44A^2=A^3+3AB^2+40B^2
B^2について解くと
B^2=A^2(44-A)/(40+3A)
明らかにx,yは正整数なので2≦A、左辺は非負なので、A≦44
この範囲で右辺が整数になるのは、A=8,20,40,44で、平方数になるのはA=8,44
(x,y)=(1,7),(7,1)(22,22)
0063学術垢版2018/09/18(火) 23:06:21.75ID:bdccv7Cm
死神死族か。
0065132人目の素数さん垢版2018/09/18(火) 23:19:04.29ID:zz7LfpDa
>>62
すげえ、言われてみれば自然な解答だな
絵を描いたら思いつきそうか?
0066132人目の素数さん垢版2018/09/18(火) 23:20:11.61ID:8tNJHaXw
>>58
ヒントありがとうございます。しかし、まだ理解できません。
もう頭がパンクしそうです。
なぜ残りの7枚を同じ種類のカードとみなせるのか、不思議です。
0067学術垢版2018/09/18(火) 23:22:21.96ID:bdccv7Cm
あほな解き方だぞそれ。
0068132人目の素数さん垢版2018/09/18(火) 23:38:20.57ID:dotA1T5U
>>66
問題
A1,A2,B1,B2,C1,C2,E,F,G,Hの10枚のカードがある。
横一列に並べたとき、左から2番目がB1、3番目がEになる、または、
2番目がB2、3番目がEになる確率は?
というのと同じ
答え
並べ方の総数は、10!通り。2番目がB1、3番目がEになる並べ方は、
2番目にB1、3番目にEを置き、残り8箇所に自由にカードをおいてよいので、8!通り
2番目がB2、3番目がEになるのも同様なので、求められている確率は 
2*8!/10! =2/(10*9)=1/45
0069学術垢版2018/09/18(火) 23:40:20.37ID:bdccv7Cm
最後まで叩いて類推すればいいじゃない。
0070学術垢版2018/09/18(火) 23:40:57.43ID:bdccv7Cm
順列に確率を求めるのが運の尽きだよ
0071学術垢版2018/09/18(火) 23:41:23.64ID:bdccv7Cm
乱雑にカードを並べてみてさ。
0072132人目の素数さん垢版2018/09/19(水) 00:16:19.17ID:pjeh/wJ3
>>68
ありがとうございます!この解答だと理解できました。
0073132人目の素数さん垢版2018/09/19(水) 01:07:44.14ID:+Ofa35sM
自殺をしたら地獄に落ちますか?
0074132人目の素数さん垢版2018/09/19(水) 01:23:15.74ID:wiQUfdGa
N組のカップル(合わせて2N人)が無作為に横一列に並ぶ
どのカップルについても彼氏と彼女が隣り合わない確率を求めよ

N組のカップルをnとおくと

q={2^n+2^(n−1)−(n−1)^2−3}/{2^(n+2)−(n+2)^2+7}

この関数をゼータ関数を参考にして修正してくれ〜(・ω・)ノ
0076132人目の素数さん垢版2018/09/19(水) 01:54:19.45ID:nLnx1y/v
>>49
文字 x, y を使って単項式 43xy の形で表された正整数 43xy の約数を
見た目から「具体的に」すべて挙げると 1、43、x、y、43x、43y、xy、43xy となるが、
x≧3、y≧2 で x^2−xy+y^21(≧2) は1を割り切らないことはすぐ分かるので、議論上は
>>43xy の約数をすべて挙げると43、x、y、43x、43y、xy、43xy となるから
としても何ら問題は生じない。
0077132人目の素数さん垢版2018/09/19(水) 02:05:04.03ID:l8Z4jqyy
>>46 >>75

f(x, y) = 1/(1+xx+yy)
 = Σ[n=0, ∞] (-1)^n (xx+yy)^n
 = Σ[n=0, ∞] (-1)^n Σ[j=0, n] C[n, j] x^{2j} y^{2n-2j}
 = Σ[j=0, ∞] Σ[k=0, ∞] (-1)^{j+k} C[j+k, j] x^{2j} y^{2k}

(0, 0) の周りだからマクローリン展開か?
0078132人目の素数さん垢版2018/09/19(水) 02:10:13.84ID:nLnx1y/v
>>49
>>76の「x^2−xy+y^21(≧2)」は「x^2−xy+y^2(≧2)」。

再度書くが、単項式 43xy の形で表された正整数 43xy の約数を
「見た目から具体的に」すべて挙げると 1、43、x、y、43x、43y、xy、43xy
となる。
0080132人目の素数さん垢版2018/09/19(水) 02:19:03.59ID:pbeFETFR
>>74
>q={2^n+2^(n−1)−(n−1)^2−3}/{2^(n+2)−(n+2)^2+7}

これ何?
そもそも漸化式前スレで出てるやん。
この q それ満たしてないやん。
0081132人目の素数さん垢版2018/09/19(水) 02:21:39.68ID:wiQUfdGa
この関数を漸化式のすべての点を通るように
ゼータ関数を参考にして修正してくれ〜(・ω・)ノ
0082132人目の素数さん垢版2018/09/19(水) 02:33:00.35ID:ZxE0BCCu
F(n)=log (2n n) ※底は2とする

のとき

O(F(n))を求めよ。

ヒント

e(n/e)^n≦n!
とする


お願いします!!
0083132人目の素数さん垢版2018/09/19(水) 02:34:35.39ID:ZxE0BCCu
(2n n)= 2n C nです
0085132人目の素数さん垢版2018/09/19(水) 03:39:38.95ID:Ck89eeKN
>>82
log C[2n n]
= log 2n! - 2logn!
〜(1/2)log(4πn)+2n log(2n/e) - log2πn-2nlog(n/e)
= (1/2)log(4π)+(1/2)log(n)+2n log(n)+2n log(2)-2n - log2π- log n-2nlog(n)
= -(1/2)log(n) + 2n log(2) - (1/2)logπ
= log (4^n/√(πn))
0086132人目の素数さん垢版2018/09/19(水) 03:45:35.63ID:l8Z4jqyy
>>74
 q[1] = 0, q[2] = 2/7, q[3] = 5/14, q[4] = 12/35, q[5] = 29/86 → 3/8,

[前スレ.609] から
 a[1] = 0, a[2] = 1/3, a[3] = 1/3, a[4] = 12/35, a[5] = 47/135 → 1/e,

 a[n] = a[n-1] + {1/(2n-1)(2n-3)} a[n-2],
0087132人目の素数さん垢版2018/09/19(水) 03:45:36.34ID:ZxE0BCCu
>>85
どこからπがでてくるんですか?
0089132人目の素数さん垢版2018/09/19(水) 04:22:24.86ID:ZxE0BCCu
>>88
π使わないで出せませんか
0090132人目の素数さん垢版2018/09/19(水) 05:47:43.58ID:LXDQ8jJn
Σ[q-n-1, j=l](-1)^(j-1) C(q-1, n+j)[C(j, l)-C(j+1, l)]=0
になる理由がどうしてもわかりません。
おしえてください。
ここでCは2項係数です。
0091132人目の素数さん垢版2018/09/19(水) 06:46:06.14ID:h607bjyl
>>66
A1,A2,B1,B2,C1,C2,D,E,F,Gと書かれたカードを用意して、
10!通り全ての並べ方を網羅する

次に、
A1,A2,C1,C2,D,F,Gの7枚のカードの文字を、X1〜X7にそれぞれ書き換える

こうすると、B1,B2,E,X1〜X7のカード10枚を使った並べ変え方10!通りになるが、文字が変わっただけなので確率は全く同じ

要するに、この2つは等価と言ってるだけ。
0092132人目の素数さん垢版2018/09/19(水) 06:50:02.48ID:h607bjyl
「B2枚、X7枚を区別しないとする順列」を求めるときの計算は、結局X1〜X7に番号を振った時の全パターン10!通りを用意した後、
B1B2、X1〜X7を区別しないとして2!*7!で割ってるのと同じ。
0093132人目の素数さん垢版2018/09/19(水) 07:35:03.80ID:l8Z4jqyy
>>89

 y = log(x) は上に凸だから
 log(k) > ∫[k-1/2, k+1/2] log(x) dx,
より
log(n!) = Σ[k=2, n] log(k)
 > log(2) + ∫[5/2, n+1/2] log(x) dx
 = (n+1/2)log(n+1/2) -n +2 + log(2) - (5/2)log(5/2)
 > (n+1/2)log(n) -n + (5/2) + log(2) - (5/2)log(5/2)     (*)
 = (n+1/2)log(n) -n + log(√6),

*) log(n+1/2) - log(n) = log(1 +1/2n) = - log{1 -1/(2n+1)} > 1/(2n+1),

 {log(k-1)+log(k)}/2 < ∫[k-1, k] log(x) dx,
より
log(n!) = Σ[k=2, n] log(k)
 < (1/2)log(2) + ∫[2, n] log(x) dx + (1/2)log(n)
 = (n+1/2)log(n) -n +2 - (3/2)log(2)
 < (n+1/2)log(n) -n + log(√7),

∴ √(6n)・(n/e)^n < n! < √(7n)・(n/e)^n,
0094132人目の素数さん垢版2018/09/19(水) 07:37:20.54ID:IjLvLKf4
>>76
>>78

相変わらず馬鹿過ぎて話にならんな
笑ったwwwww

誤答おじさんの頭の悪さはどうにもならんwwwww
0096132人目の素数さん垢版2018/09/19(水) 07:53:19.31ID:h607bjyl

5-4-1):x^2−xy+y^2 が xy を割り切るとき。すると、xy の最大の約数は xy なることに着目すると x^2−xy+y^2=xy としてよい。

ここですね
0097132人目の素数さん垢版2018/09/19(水) 08:05:38.72ID:l8Z4jqyy
>>93 補足

 ∫ log(x) dx = x log(x) - x,

 {2 ・ (2e/5)^2.5}^2 = 6.079003 > 6

 {e^2 / 2^(3/2)}^2 = 6.824768754 < 7
0099132人目の素数さん垢版2018/09/19(水) 09:08:26.22ID:Fu0oOLgN
クラス会の費用を集めるのに全体で800円余る予定で一人1700円ずつ集めたが、予定 よりも全体で8000円多く費用がかかったので、一人300円を追加して集めたところ、ちょうど支 払うことができた。このとき、クラス会でかかった費用は全部で何円か、求めなさい。

これ分かる人いますか
0100132人目の素数さん垢版2018/09/19(水) 09:40:52.15ID:OD14AjpY
>>90
q-n-1=lのとき
Σ[q-n-1, j=l](-1)^(j-1) C(q-1, n+j)[C(j, l)-C(j+1, l)]
= Σ[l, j=l](-1)^(j-1) C(q-1, n+j)[C(j, l)-C(j+1, l)]
= (-1)^(l-1) C(n+l, n+l)[C(l, l)-C(l+1, l)]
はあきらかに0にならんけど?
0105132人目の素数さん垢版2018/09/19(水) 10:41:35.93ID:OD14AjpY
>>101
そもそもそのjpegの最初n行と最後の行に q = l+n+1 代入して成立してないんじゃね?
一行目=(-1)^(l-1)C[l+n+1,l+n]C[l,l] + (-1)^lC[l+1,l]=(-1)^(l+1)(l+n+1-l-1)=(-1)^(l+1)n
最終行=C[l+n,n+l-1] = l+n
で合ってない。
0106132人目の素数さん垢版2018/09/19(水) 11:11:40.33ID:nLnx1y/v
>>94-96
>互いに素ではなくない?
xとyが互いに素でないとする。
xとyに共通する素因数を p_1, …, p_n とする。 各 i=1,…,n に対して、p_i の指数を e_i とする。
xだけの素因数を q_1, …, q_m とする。各 i=1,…,m に対して、q_i の指数を a_i とする。
yだけの素因数を r_1, …, r_k とする。各 i=1,…,k に対して、r_i の指数を b_i とする。
xy を x^2−xy+y^2 で割った商をaとする。すると、a(x^2−xy+y^2)=xy、
x=(p_1)^{e_1}・…・(p_n)^{e_n}×(q_1)^{a_1}・…・(q_n)^{a_n}、 y=(p_1)^{e_1}・…・(p_n)^{e_n}×(r_1)^{b_1}・…・(r_n)^{b_n}   で、
x^2−xy+y^2=(p_1)^{2e_1}・…・(p_n)^{2e_n}×(q_1)^{2a_1}・…・(q_n)^{2a_n}、
          −(p_1)^{2e_1}・…・(p_n)^{2e_n}×(q_1)^{a_1}・…・(q_n)^{a_n}×}×(r_1)^{b_1}・…・(r_n)^{b_n}
          +(p_1)^{2e_1}・…・(p_n)^{2e_n}×(r_1)^{2b_1}・…・(r_n)^{2b_n}、
xy=(p_1)^{2e_1}・…・(p_n)^{2e_n}×(q_1)^{a_1}・…・(q_n)^{a_n}×}×(r_1)^{b_1}・…・(r_n)^{b_n}   なので、a(x^2−xy+y^2)=xy は
a( (q_1)^{2a_1}・…・(q_n)^{2a_n}−(q_1)^{a_1}・…・(q_n)^{a_n}×}×(r_1)^{b_1}・…・(r_n)^{b_n}+(r_1)^{2b_1}・…・(r_n)^{2b_n} )
=(q_1)^{a_1}・…・(q_n)^{a_n}×}×(r_1)^{b_1}・…・(r_n)^{b_n}
となる。X=(q_1)^{a_1}・…・(q_n)^{a_n}、Y=(r_1)^{b_1}・…・(r_n)^{b_n} とおけば、a(x^2−xy+y^2)=xy は
a(X^2−XY+Y^2)=XY となる。よって、X^2−XY+Y^2 は XY を割り切る。
あと a>1 とすると a≧2 で、相加・相乗平均の不等式から、a(X^2+Y^2)≧2aXY>(a+1)XY
だから、a(X^2−XY+Y^2)>XY となって、矛盾が生じる。よって、a=1 で、X^2−XY+Y^2=XY となる。
ここに、x^2−xy+y^2 と X^2−XY+Y^2、及び xy と XY は単項式としては同じ形。だから、上のような議論をすることは、実質的には
>5-4-1):x^2−xy+y^2 が xy を割り切るとき。すると、xy の最大の約数は xy なることに着目すると x^2−xy+y^2=xy としてよい。
と書くことと同じで、何も式の形としては変わっていない。変わったのは、xとyが互いに素でないときも考えて細かい議論をするかどうかの違い。
0107132人目の素数さん垢版2018/09/19(水) 11:22:16.75ID:OD14AjpY
>>106
>xy を x^2−xy+y^2 で割った商をaとする。
xy≦x^2−xy+y^2じゃね?
a=0、あまりx^2−xy+y^2になるよ?
0108132人目の素数さん垢版2018/09/19(水) 11:53:58.09ID:fbWt698J
>>106
昔から態度ばかり一人前だけど
対称式の頃から本当に成長してないな
もし数学の勉強をしてるのだとしたら
ここまで何年も最底辺レベルのまま成長しない奴も珍しいぜ
0109132人目の素数さん垢版2018/09/19(水) 11:58:58.59ID:Gn6ogjJL
後藤さん引退宣言したんでないの?
0110132人目の素数さん垢版2018/09/19(水) 12:13:15.00ID:nLnx1y/v
>>107
いわれてみるとそうだな。>>94-96は一体何だったんだろう。

>>94-96
>互いに素ではなくない?
xy≦x^2−xy+y^2 だから、xy を x^2−xy+y^2 で割ったときの商は0で余りをaとする。すると、x^2−xy+y^2+a=xy、
a≠0 とすると、(x−y)^2>−a で、(x−y)^2=−a に反し矛盾するから、a=0、故に。x^2−xy+y^2+a=xy。
蛇足だが、>>106のqの添え字mと、rの添え字kの書き間違いが何ヶ所かあるから、訂正して読んでほしい。
主に途中の派手な式のところにある。
0111132人目の素数さん垢版2018/09/19(水) 12:21:59.79ID:nLnx1y/v
ところで、コーコー数学や受験数学でデカルトの葉線ってやっていたっけ?
デカルトの葉線は何に書いてあるんだ?
0112132人目の素数さん垢版2018/09/19(水) 12:45:15.55ID:bI/clKdo
ある数列に対して、それが漸化式として表される場合、
その数列を作る漸化式はただ一つに定まりますか?
0114132人目の素数さん垢版2018/09/19(水) 13:06:19.96ID:iMuVMgfo
>慶應義塾大学大学院理工学研究科
>KiPAS数論幾何グループ
>『辺の長さが全て整数となる直角三角形と二等辺三角形の組の中には、
>周の長さも面積も共に等しい組が(相似を除いて)たった1組しかない』
>という、これまで知られていなかった定理の証明に成功した。

↑これってどのくりあ凄いことなの?
数学界の功績で言えばどのくらいですか?論文として今年度のトップ10くらいに入る?


自然数で表面積が等しく、かつ体積が等しい立体の組み合わせ
は存在するの?
その場合、立体 3つ1組 ですか?
0115132人目の素数さん垢版2018/09/19(水) 13:16:28.72ID:X/om76cf
>>114
トップ10に入るような業績ではないけど長く記憶されそうな業績。

そのような立体があるかは分からない。多分無い可能性が高いだろう
0118132人目の素数さん垢版2018/09/19(水) 14:47:01.97ID:Byy4q6sb
>>114
慶応の論文で出てきた直角三角形と二等辺三角形を底辺に持ち、高さが自然数の三角柱って
自然数で表面積が等しく、かつ体積が等しい立体の組み合わせにならないか?
高さは自然数なら何でもいいので無限にある
0120132人目の素数さん垢版2018/09/19(水) 14:55:20.32ID:X/om76cf
>>112
数列による

本質的には1つに定まるものが多いんじゃないか?(隣接2〜3項の関係のみで表し、既約なもの)

高校数学までの範囲なら全部定まるのでは
0121132人目の素数さん垢版2018/09/19(水) 15:00:25.63ID:Gn6ogjJL
「既約なもの」ってなあに?
0123132人目の素数さん垢版2018/09/19(水) 15:41:42.32ID:xWCfGFrt
xy平面上の曲線Cを、媒介変数θを用いて
x=2(cosθ)^2-3(cosθ+sinθ)
y=6(sin[2θ])
と定義する。
Cで囲まれる領域の面積を求めよ。
0124132人目の素数さん垢版2018/09/19(水) 16:03:20.74ID:nLnx1y/v
>>107
>>xy を x^2−xy+y^2 で割った商をaとする。
>xy≦x^2−xy+y^2じゃね?
>a=0、あまりx^2−xy+y^2になるよ?
x≧y と仮定していて x≧3、y≧2 だから、x=y≧3 のときもあり得て、
このときは xy=x^2 は x^2−xy+y^2=x^2 で割り切れて a=1 となる。見落としがあった。

>94-96、>107
>>110

>>107
>いわれてみるとそうだな。>>94-96は一体何だったんだろう。

>>94-96
>>互いに素ではなくない?
>xy≦x^2−xy+y^2 だから、xy を x^2−xy+y^2 で割ったときの商は0で余りをaとする。すると、x^2−xy+y^2+a=xy、
>a≠0 とすると、(x−y)^2>−a で、(x−y)^2=−a に反し矛盾するから、a=0、故に。x^2−xy+y^2+a=xy。

のところは削除。>>106の添え字を訂正して読めばいい。
0126132人目の素数さん垢版2018/09/19(水) 16:12:05.22ID:fbWt698J
>>124
毎度の事だけど
もう正解は出た後だから
無駄に長いだけで、間違いだらけな答案は要らないと思うの
0127132人目の素数さん垢版2018/09/19(水) 16:21:42.40ID:nLnx1y/v
>>94-96    (>>106の訂正。主に、添え字のみ訂正。文章の内容は大体同じ。)
>互いに素ではなくない?
xとyが互いに素でないとする。
xとyに共通する素因数を p_1, …, p_n とする。 各 i=1,…,n に対して、p_i の指数を e_i とする。
xだけの素因数を q_1, …, q_m とする。各 i=1,…,m に対して、q_i の指数を a_i とする。
yだけの素因数を r_1, …, r_k とする。各 i=1,…,k に対して、r_i の指数を b_i とする。
xy を x^2−xy+y^2 で割った商をaとする。すると、a(x^2−xy+y^2)=xy、
x=(p_1)^{e_1}・…・(p_n)^{e_n}×(q_1)^{a_1}・…・(q_m)^{a_m}、 y=(p_1)^{e_1}・…・(p_n)^{e_n}×(r_1)^{b_1}・…・(r_k)^{b_k}   で、
x^2−xy+y^2=(p_1)^{2e_1}・…・(p_n)^{2e_n}×(q_1)^{2a_1}・…・(q_m)^{2a_k}、
          −(p_1)^{2e_1}・…・(p_n)^{2e_n}×(q_1)^{a_1}・…・(q_m)^{a_m}×}×(r_1)^{b_1}・…・(r_k)^{b_k}
          +(p_1)^{2e_1}・…・(p_n)^{2e_n}×(r_1)^{2b_1}・…・(r_k)^{2b_k}、
xy=(p_1)^{2e_1}・…・(p_n)^{2e_n}×(q_1)^{a_1}・…・(q_m)^{a_m}×}×(r_1)^{b_1}・…・(r_k)^{b_k}   なので、a(x^2−xy+y^2)=xy は
a( (q_1)^{2a_1}・…・(q_m)^{2a_m}−(q_1)^{a_1}・…・(q_m)^{a_m}×}×(r_1)^{b_1}・…・(r_k)^{b_k}+(r_1)^{2b_1}・…・(r_k)^{2b_k} )
=(q_1)^{a_1}・…・(q_m)^{a_m}×}×(r_1)^{b_1}・…・(r_k)^{b_k}
となる。X=(q_1)^{a_1}・…・(q_m)^{a_m}、Y=(r_1)^{b_1}・…・(r_k)^{b_k} とおけば、a(x^2−xy+y^2)=xy は
a(X^2−XY+Y^2)=XY となる。よって、X^2−XY+Y^2 は XY を割り切る。
仮に a>1 とすると a≧2 で、相加・相乗平均の不等式から、a(X^2+Y^2)≧2aXY>(a+1)XY
だから、a(X^2−XY+Y^2)>XY となって、矛盾が生じる。よって、a=1 で、X^2−XY+Y^2=XY となる。
ここに、x^2−xy+y^2 と X^2−XY+Y^2、及び xy と XY は単項式としては同じ形。だから、上のような議論をすることは、実質的には
>5-4-1):x^2−xy+y^2 が xy を割り切るとき。すると、xy の最大の約数は xy なることに着目すると x^2−xy+y^2=xy としてよい。
と書くことと同じで、何も式の形としては変わっていない。
0129132人目の素数さん垢版2018/09/19(水) 16:38:05.23ID:+AYEmU2z
Mathematica を使っています。

出力結果を人間が普通書くのと同じように出力させることはできないのでしょうか?

https://imgur.com/vTWtvuD.jpg

↑例えば、これは3つの2次以下の多項式を直交化したものです。

出力結果は人間では考えられない形をしています。

人間が書くのと同じように出力してほしいという需要は非常に強いと思いますが、
なぜ、 Mathematica でそのような出力を選択するようなモードがないのでしょうか?

そんなに実現するのが難しいのでしょうか?
0130132人目の素数さん垢版2018/09/19(水) 16:38:09.23ID:nLnx1y/v
>>94-96
>>127の途中式の部分
>x^2−xy+y^2=(p_1)^{2e_1}・…・(p_n)^{2e_n}×(q_1)^{2a_1}・…・(q_m)^{2a_k}、
>          −(p_1)^{2e_1}・…・(p_n)^{2e_n}×(q_1)^{a_1}・…・(q_m)^{a_m}×}×(r_1)^{b_1}・…・(r_k)^{b_k}
>          +(p_1)^{2e_1}・…・(p_n)^{2e_n}×(r_1)^{2b_1}・…・(r_k)^{2b_k}、
>xy=(p_1)^{2e_1}・…・(p_n)^{2e_n}×(q_1)^{a_1}・…・(q_m)^{a_m}×}×(r_1)^{b_1}・…・(r_k)^{b_k}   なので、a(x^2−xy+y^2)=xy は
>a( (q_1)^{2a_1}・…・(q_m)^{2a_m}−(q_1)^{a_1}・…・(q_m)^{a_m}×}×(r_1)^{b_1}・…・(r_k)^{b_k}+(r_1)^{2b_1}・…・(r_k)^{2b_k} )
>=(q_1)^{a_1}・…・(q_m)^{a_m}×}×(r_1)^{b_1}・…・(r_k)^{b_k}

>x^2−xy+y^2=(p_1)^{2e_1}・…・(p_n)^{2e_n}×(q_1)^{2a_1}・…・(q_m)^{2a_m}、
>          −(p_1)^{2e_1}・…・(p_n)^{2e_n}×(q_1)^{a_1}・…・(q_m)^{a_m}×}×(r_1)^{b_1}・…・(r_k)^{b_k}
>          +(p_1)^{2e_1}・…・(p_n)^{2e_n}×(r_1)^{2b_1}・…・(r_k)^{2b_k}、
>xy=(p_1)^{2e_1}・…・(p_n)^{2e_n}×(q_1)^{a_1}・…・(q_m)^{a_m}×(r_1)^{b_1}・…・(r_k)^{b_k}   なので、a(x^2−xy+y^2)=xy は
>a( (q_1)^{2a_1}・…・(q_m)^{2a_m}−(q_1)^{a_1}・…・(q_m)^{a_m}×(r_1)^{b_1}・…・(r_k)^{b_k}+(r_1)^{2b_1}・…・(r_k)^{2b_k} )
>=(q_1)^{a_1}・…・(q_m)^{a_m}×}×(r_1)^{b_1}・…・(r_k)^{b_k}
に訂正。
0131132人目の素数さん垢版2018/09/19(水) 16:47:24.60ID:08zNaTf2
>>129
"Mathematica TeX"や"Mathematica LaTeX"でググれば?
自分の環境も書かないでそれ以上の回答は期待できないよ、こっちもエスパーじゃないんだから
0132132人目の素数さん垢版2018/09/19(水) 17:09:34.72ID:+AYEmU2z
TeX の話ではなく、例えば、√を含んだ式が人間にとって違和感のある式になっているのを改善したいという話です。
0134132人目の素数さん垢版2018/09/19(水) 17:56:18.08ID:SDPqlDZx
>>123
x = 2(cosθ)^2-3(cosθ+sinθ) = cos(2θ)-3√2sin(θ+π/4)+1
y = 6sin(2θ)
θ+π/4=φとおいて
x = cos(2φ-π/2)-3√2sinφ+1 = sin(2φ)-3√2sinφ+1 = (2cosφ-3√2)sinφ+1
y = 6sin(2φ-π/2) = -6cos(2φ)
x=x(φ),y=y(φ)とすると
x(φ)=-x(-φ),y(φ)=y(-φ)より左右対称
0<φ<πでx<1、π<φ<2πで1<x
0<φ<π/2で
x(φ)-x(π-φ) = 4cosφsinφ=2sin2φ > 0
y(φ) = y(π-φ)
よって面積は
2∫[0,π/2]2sin2φ*12cos(2φ)dφ = 6
0135134垢版2018/09/19(水) 18:09:11.12ID:SDPqlDZx
計算は間違ってるけど方針はこれでいけると思う
0136132人目の素数さん垢版2018/09/19(水) 18:10:10.37ID:iMuVMgfo
>>118
あ、本当だ。
この三角形の組に厚みを足すだけでいいね。
0137132人目の素数さん垢版2018/09/19(水) 18:44:38.10ID:ACAGiZvC
>これまで知られていなかった定理の証明に成功した。

修士論文ならともかく、博士論文なら当たり前では
既知の結果の別証明なんて(それにより一般化・抽象化が出来て新規の結果が出てこない限り)殆ど研究業績として認められんがな
0138132人目の素数さん垢版2018/09/19(水) 19:24:18.72ID:ACAGiZvC
ああ、博士論文ではないのね
それにしても論文なら新規の結果であって当然では
0139132人目の素数さん垢版2018/09/19(水) 19:40:51.86ID:08zNaTf2
>>137
>既知の結果の別証明なんて(それにより一般化・抽象化が出来て新規の結果が出てこない限り)殆ど研究業績として認められんがな
おっとカントールへの悪口はそこまでだw
0140132人目の素数さん垢版2018/09/19(水) 19:41:49.99ID:yx5p5nJm
>>138
すごく頭悪そうなレスだな
0142132人目の素数さん垢版2018/09/19(水) 21:44:48.43ID:uE2uC1cX
馬鹿みたいな質問なんですけど…
偏微分って結局何がしたいんですか?
何をどうしてるんですか?
何を求めたいのですか?
0145132人目の素数さん垢版2018/09/19(水) 22:33:33.65ID:uE2uC1cX
>>144
微分したいのは分かるんですよ。
例えば一次変数の微分は曲線の一部分を限りなく小さくして直線として考え求めるっていう目的(?)があるじゃないですか

2変数関数は偏微分して何が求まるのか分からないんですよ
0146132人目の素数さん垢版2018/09/19(水) 22:34:02.34ID:S18XlP4A
任意の2次の正方行列Xに対してAX=XAを満たす行列Aはどんだ行列か。
途中計算も含めてお願いします
0150132人目の素数さん垢版2018/09/19(水) 22:55:44.82ID:dHok8gN8
>>149
曲面に接する接面ができますよね
その面に上に直線を考えることができますけど、これはいろいろありますよね
xで偏微分する時は、x軸が正射影になるような直線を考えます
偏微分は直線の傾きを表します


めんどくさいですよね?
混乱するだけなので、普通に多変数のときの微分は偏微分って言うんだなーでいいんですよだから
0151132人目の素数さん垢版2018/09/19(水) 23:02:56.59ID:PaYlAUvO
1からNの数字の中から連続するk個の塊をm個取る組み合わせ数をN, k, mで表せ
ただし重複はなしとし、N >= k*m とする
(k=1のときは通常の組み合わせ C[N, m])

連続するk個の塊というのは、例えばN=5,k=2の場合
(1,2), (2,3), (3,4), (4,5) のことで、ここでさらにm=2だったら
(1,2)と(3,4), (1,2)と(4,5), (2,3)と(4,5) の3組が答えになります

よろしくおねがいします
0152132人目の素数さん垢版2018/09/19(水) 23:03:58.24ID:uE2uC1cX
>>150
あー。なんとなーく分かりました
曲面をxやyを固定して切断した時に出来る曲線の傾きって感じですか?

面倒ですね…w
しかし数学科なものでどういう意味かちゃんと理解しときたいのです…
0154132人目の素数さん垢版2018/09/19(水) 23:13:17.81ID:yy7XD51R
数学科なら、たとえF欄以下だったとしてもここできくより担当の講師かTAにきいた方がいいと思うが。
0155132人目の素数さん垢版2018/09/19(水) 23:13:29.59ID:dHok8gN8
>>152
あと方向微分とかいうのも調べておきましょう
偏微分は個人的には図形的イメージより数式でイメージできた方が良いと思います
0158132人目の素数さん垢版2018/09/19(水) 23:42:03.84ID:PaYlAUvO
>>156
ありがとうございます
計算してみるとそれで合っていそうなんですが
どういうふうに考えてその式を導いたのでしょうか?

よろしければ考え方を教えてくださいm(_ _)m
0159132人目の素数さん垢版2018/09/20(木) 00:09:33.17ID:nSUDamRJ
例えば、N=12、k=3、m=2とすると、
○○○○○○○○○○○○

○○○●●●○●●●○○
のような選び方がいくつあるかという問題だけど、●●●を■に置き換えると
○○○■○■○○
となる。逆に
○○○○○○○○
から、二つを選ぶ。例えば、
○■○○○○■○
とすると、ここで■を●●●に置き換えれば、
○●●●○○○○●●●○
になる。このように、どちら側にも変換可能。
この変換の時、いくつ減らせばいいかを考えると、●●●が■になるのだから、
つまり、k個を1個にするので、(k-1)個減り、
それが、m箇所あるので、m*(k-1)減ることになる。これをNから引けばよい。
ということで、C[N-m*(k-1),m]が出てくる
0160132人目の素数さん垢版2018/09/20(木) 00:17:47.42ID:zRtMQ4MM
>>159
なるほど!
すごくわかりやすいです!

図まで書いてくれて本当にありがとうございます
おかげさまで完全に理解できました
0161132人目の素数さん垢版2018/09/20(木) 01:57:40.94ID:7+n0UQHR
>>90

l ≦ q-n とする。
>>101 の画像は 要するに
S(q, l, n) = Σ[j=l, q-n] (-1)^{j-l} C(q, n+j) C(j, l)
 = Σ[j=l, q-n] (-1)^{j-l} {C(q-1, n+j) + C(q-1, n+j-1)} C(j, l)
 = Σ[j=l-1, q-n-1] (-1)^{j-l} C(q-1, n+j) C(j, l)   ← C(l-1,l)=C(q-1,q)=0
  + Σ[j=l-1, q-n-1] (-1)^{j+1-l} C(q-1, n+j) C(j+1, l)  ← jをずらす
 = Σ[j=l-1, q-n-1] (-1)^{j+1-l} C(q-1, n+j) {C(j+1,l) - C(j, l)}
 = Σ[j=l-1, q-n-1] (-1)^{j+1-l} C(q-1, n+j) C(j, l-1)
 = S(q-1, l-1, n)
を示す式で、これから
 S(q, l, n) = S(q-l, 0, n),
となる。
S(q', 0, n)
 = Σ[j=0, q'-n] (-1)^j C(q', n+j) C(j, 0)
 = Σ[j=0, q'-n] (-1)^j C(q', n+j)
 = Σ[j=0, q'-n] (-1)^j {C(q'-1, n+j) + C(q'-1, n+j-1)}  ← C(q'-1,q')=0
 = C(q'-1, n-1),
から
 S(q, l, n) = C(q-l-n, n-1),
0162132人目の素数さん垢版2018/09/20(木) 02:16:08.86ID:7+n0UQHR
>>161 訂正

q-l ≧n≧1 のとき
 S(q-l, 0, n) = C(q-l-1, n-1),
q-l = n のとき 1,
n=0 のとき
 S(q-l, 0, n) = (1-1)^(q-l) = δ_{q-l, n}
でした。
0163132人目の素数さん垢版2018/09/20(木) 03:46:06.21ID:7+n0UQHR
>>134 >>135
蛇足ですが…

0<φ<π/2 で
 x(φ) = √{1-(y/6)^2} -3√(1+y/6) +1,
 x(π-φ) = -√{1-(y/6)^2} -3√(1+y/6) +1,
 x(φ) - x(π-φ) = 2√{1-(y/6)^2} = (1/3)√(36-yy),
 y = -6cos(2φ),
 dy = 12sin(2φ)dφ,

S/2 = (1/6)∫[-6, 6] 2√(36-yy) dy = (1/6) (半径6の円の面積) = 6π,

S = 12π.
0164132人目の素数さん垢版2018/09/20(木) 04:56:34.13ID:7+n0UQHR
>>117

x^3 -3axy +y^3 = 0,
Descar?

x^3 -3axy +y^3 = (x+y+a){xx-xy+yy-a(x+y)+aa} - a^3,
から
∴ x+y+a = a^3 /{xx-xy+yy -a(x+y) +aa} → 0, |x|+|y|→∞

∴ 漸近線は x+y+a = 0,
0165132人目の素数さん垢版2018/09/20(木) 05:10:52.38ID:Ajky0sy3
媒介変数tを用いて表されるxy平面上の曲線
x=3cos(t+π/4)+4sin(t)
y=cos(t-π/3)+sin(t+π/6)
を考える。

以下、実数tは0≦t<2πの範囲を動くものとする。

xの最大値は( ア )であり、yの最小値は( イ )である。
dy/dx=0となる点は全部で( ウ )個ある。
したがって、Cが自己交差する点は全部で( エ )個ある。
0169132人目の素数さん垢版2018/09/20(木) 08:27:45.69ID:/JkfMF/D
最後は誤記で、-1/sinxとなって、正負が逆になってしまうということです。
0171132人目の素数さん垢版2018/09/20(木) 09:30:20.77ID:sA3mNheb
さすがにこのレベルで先生に頼っちゃダメだとは思うけど、ここに頼るよりまだマシかなぁ…
積分はあってる。
微分で(少なくとも)2カ所間違えてる。
0172132人目の素数さん垢版2018/09/20(木) 09:36:46.07ID:/JkfMF/D
f(x)が微分可能だとして
g(x)=log|f(x)| を微分すると
一般にg'(x)=f'(x)/f(x) これは合っていますよね?

2/sinx を微分するとlog|1 - cosx|ーlog|1 + cosx| +C (←模範解答)
=log|cosx - 1|ーlog|cosx +1| +C

log|cosx - 1|ーlog|cosx +1| を微分すると

-sinx / (cosx - 1) +sinx / (cosx +1)

=sinx *( (1/cosx + 1) - (1/cosx - 1))

=sinx * ( 2/-sin^2x)

= -2/sinx

となって正負が逆転したのですが
どこか計算ミスがあると思うんですが、どこがおかしいのでしょうか?
すみませんがお願いしますm(_ _)m
0174132人目の素数さん垢版2018/09/20(木) 09:41:34.15ID:sA3mNheb
もう一つどうしても言わせてくれ
絶対値は飾りっぽいけど、飾りじゃないからな。log(cosx-1)とかはまだ使っちゃダメだぞ
0175132人目の素数さん垢版2018/09/20(木) 09:44:38.54ID:7+n0UQHR
>>168 >>169

log|(cos(x)-1)| = log(1-cos(x)) = log(cos(x)-1) +iπ,

ですが、このiπは積分定数に繰り込めるので、結果に影響はないでしょう。

しかし 1/(cos(x)+1) - 1/(cos(x)-1) の計算ミスで符号が反対になったのはより深刻です。

簡単な分数計算ができてないのがイタイ。
0176132人目の素数さん垢版2018/09/20(木) 10:38:17.82ID:TFednSDK
>>146
146です。
この問題の行列の基本変形がわからないので3つめの変形の解説をお願いします

https://i.imgur.com/q4GIMLA.jpg
0178132人目の素数さん垢版2018/09/20(木) 13:23:17.83ID:z1K1qGzT
>>177
分母を平方完成→因数分解→部分分数分解→和積公式
分母と分子見比べてf'/f or f(g)g' の形を見つける
0179132人目の素数さん垢版2018/09/20(木) 14:34:11.98ID:JTFgvHMK
霊能者や霊媒師が、自殺をした人の霊は猛烈に苦しみ、とてつもなく後悔していると言いますが、
やはり、死後の世界はあるということなのでしょうか?
0181132人目の素数さん垢版2018/09/20(木) 15:23:03.56ID:IpTsImPW
>>179
いいことを教えてやろう。
実は今生きているこちらが死後だ。 幻の大地!
0182132人目の素数さん垢版2018/09/20(木) 16:06:37.68ID:7+n0UQHR
>>165 >>166

長軸
 t = 0.830291
 (x, y) = (2.81788 1.953136)
 a = 3.42858
 傾角α = 0.60611
 tanα = 0.69315
 sinα = 0.56968
 cosα = 0.82187

短軸
 t = 2.401087
 (x,y) = (-0.298341 0.430414)
 b = 0.523702
 傾角β = -0.96468
 tanβ = -1.44269
 sinβ = -0.82187
 cosβ = 0.56968

離心率
 ε = √{1-(b/a)^2} = 0.988265

 x・cosβ + y・sinβ = b・cos(t+0.740505)
 -x・sinβ + y・cosβ = a・sin(t+0.740505)
0190132人目の素数さん垢版2018/09/20(木) 22:10:56.69ID:+zFxMZL1
https://s3-ap-northeast-1.amazonaws.com/asset.bengo4.com/topics/8084.jpg

不快な画像を貼り付けるユーザーに対し、
匿名掲示板「ガールズちゃんねる」は1月16日、
法的措置をとることを決定した
アンケートサイト「SurveyMonkey」上で発表し、
サイトからリンクしていた(現在公開終了)
運営会社ジェイスクエアードは
「弊社が公表したもので間違いございません」と答えたが、
それ以外については回答を控えるとしている

具体的には、
ゴキブリの画像を大量投稿する特定ユーザーがいるとのこと
警告や投稿禁止措置をとっても、IPアドレスや端末情報を変更し、
投稿を続けているそうだ

ガールズちゃんねるは、このユーザーに対し、
「威力業務妨害罪」での刑事告訴と、
民事では「業務妨害」による損害賠償請求をする予定で、
顧問弁護士が手続きを進めているという
0191132人目の素数さん垢版2018/09/20(木) 22:15:49.85ID:uGl5dFIN
>>190
申し訳ございませんでした。
失礼致します。
0193132人目の素数さん垢版2018/09/21(金) 00:44:00.60ID:kiFkt26+
μ を (0, ∞) 上の σ 有限測度とする。∫[0, ∞] min(x, 1) μ(dx) < ∞ ならば
lim[x → 0+0] x μ(x, ∞)=0 であることを証明せよ。

バカなのでわかりません。教えて下さい。お願いします。
0195132人目の素数さん垢版2018/09/21(金) 02:26:00.75ID:rgDs3VYK
>>193
その主張は正しくないし何を写し間違えたのかもよく分からん。
もう一度問題文を読み直してくれ。
0197学術垢版2018/09/21(金) 09:14:38.90ID:AzK+Q3eB
ゼロというのは仮の仮象の数だと考えるべきだろ。無限とゼロはまた違うんだけど、
親和性が在るようでやはり異質だと思うよ。元をたどればやはり同じではないだろう。
交差して混ざり合っているかもしれないけど。あるところでは。ある時間に。
0198193垢版2018/09/21(金) 09:33:14.44ID:kiFkt26+
>>195
え?正しくないんですか?何か反例があるってことですか?問題文はこれで会ってる
と思います。反例があったら教えて下さいm(_ _)m
0199132人目の素数さん垢版2018/09/21(金) 11:42:03.62ID:L4/KH63z
自分は地理感覚が凄く悪くて、道路の名前とか位置関係とかがさっぱり分からないので、
もの凄く困っています。
これじゃあ車を運転してどこかに行くことすらできません。
自分の知っている範囲内ならなんとかなるのですが、知らない所だとどっちに行ったりすれば良いのかすら分かりません。
そこで質問があるのですが、そういう地理感覚などを鍛えたり理解したりできるようになるための学校みたいな所は無いでしょうか?
教えてください。
0203学術垢版2018/09/21(金) 14:01:43.93ID:AzK+Q3eB
田植えや軍隊の列は限界文明なのかな。
0204132人目の素数さん垢版2018/09/21(金) 14:05:01.18ID:0uIdegM1
>>202

{
{1, 0, 0},
{0, 1, 0},
{0, 0, -1}
}



{
{-1, 0, 0},
{0, -1, 0},
{0, 0, 1}
}

の固有値は 1 と -1 ですが、それらの固有多項式は異なります。
0205132人目の素数さん垢版2018/09/21(金) 14:34:49.07ID:b65ucfBh
>>182

 6(3-2√2)sin(2t) + (-9 +12√2 +2√3)cos(2t) = 0,
より
 tan(2t) = -{(7/2) +3√2 +√3 +(2/3)√6}
    = -11.1076846565436145

長軸
 t = 0.830291020343980
 π/2-t = 0.7405053064509164
 (x, y) = (2.817877632166427 1.953135730826556)
 a = 3.428581854483754
 傾角α = 0.60609558521919
 tanα = 0.693122976147462

短軸
 t = 2.401087347138877
 π-t = 0.7405053064509164
 (x, y) = (-0.298333540955400 0.430419350132652)
 b = 0.5237019368186468
 傾角β = -0.964700741575706
 tanβ = -1.442745420961562

 aa + bb = 29 - 12√2 = 12.02943725152286
 ab = (3√2 +3√6 -8)/2 = 1.795554957734410
 α-β = π/2,
0206132人目の素数さん垢版2018/09/21(金) 14:50:07.02ID:b65ucfBh
>>200

・相似な行列

・三角行列で、対角要素が同じ(か入替えた)もの。

 (固有ベクトルの情報はたぶん関係ない…)
0208132人目の素数さん垢版2018/09/21(金) 16:04:58.70ID:9KpTXP1n
>>200
「固有値が(重複度も込めて)同じ」というのが普通.

気取っていうならば,「ジョルダン分解の半単純部分が相似」.
0209学術垢版2018/09/21(金) 16:37:21.28ID:AzK+Q3eB
うーん数学の少数は乱数化しないと、植物や動物だけじゃないけど、
反抗期を迎えてしまうだろう。誰もいないのに。
0210学術垢版2018/09/21(金) 16:38:01.94ID:AzK+Q3eB
解までいくことだよ。それで合うことも少ない事であるなあ。
0211学術垢版2018/09/21(金) 17:34:25.41ID:AzK+Q3eB
心理はいいけど、精神の数学術への適応や、返し、出来栄えが最悪なのが
現代数学の一つの分析哲学、言語記号論的 なテーマになりえると思う。
0212学術垢版2018/09/21(金) 17:55:36.24ID:AzK+Q3eB
ダークカオス、の方が有利ということだよな。ラightもたまには。
0213132人目の素数さん垢版2018/09/21(金) 18:05:30.84ID:b65ucfBh
>>165

(ア) √(25-12√2),  t = 2arctan[(8-3√2)/{3√2+2√(25-12√2)}] = 0.72481223
(イ) -2,        t = 4π/3,
(ウ) 2,         t = π/3、4π/3.
(エ) 0

 y = 2cos(t -π/3) = 2sin(t+π/6),
0214132人目の素数さん垢版2018/09/21(金) 18:28:52.84ID:/sYU4+YY
東大法学部で断然トップの人は、どれくらい数学や物理学ができますか?
文系なので大したことないですか?
0215学術垢版2018/09/21(金) 18:35:36.95ID:AzK+Q3eB
数学は数学を集めていないから、スレ違う二人という意味で、国立の法学部
も優秀。僕はストラトプールとか ドレッシー デンぐらいしか知りません。
世界ランキングでも上位の下級ぐらいに若い才能があって・・・・。再上位は
隠し子でしょう。
0216132人目の素数さん垢版2018/09/21(金) 18:45:00.78ID:0/n0sIEP
成立学園1-F担任の岩崎柾典先生がヤバイ。
成立学園に勤めるのは4年目。
担当科目は数学。
女子テニス部の顧問をしている。
何がヤバイというと、2013年4月から2015年3月まで宮前平中に働いていたらしく、女子中学生とsexしたことがバレて、飛ばされたから。
今でも教師を続けているのがすごく不思議な感じだよ。
岩崎先生って、ツイッターとFacebookをやってるみたいだから、覗いてみては?
嘘だと思うなら、電話してみてね!

03-3902-4411

https://mevius.5ch.net/test/read.cgi/lovesaloon/1537343184/l50
https://2ch.vet/re_maguro_poverty_1535964420_a_0
https://ja-jp.facebook.com/masaoki.iwasaki.9
https://twpro.jp/kainoko1
https://www32.atwiki.jp/wslc/pages/21.html
https://twitter.com/mas20285
https://twitter.com/keepmathtop
https://twitter.com/K46_N700_hikari

https://i.imgur.com/VNvpdr1.jpg
https://i.imgur.com/GuhllEE.jpg
https://i.imgur.com/13xM5pA.jpg
https://i.imgur.com/EKFWYTU.jpg
https://i.imgur.com/YyEMHyP.jpg
https://i.imgur.com/eLIWo6B.png
https://i.imgur.com/KxU6xO2.jpg
https://i.imgur.com/REbOimQ.jpg
https://twitter.com/5chan_nel (5ch newer account)
0219132人目の素数さん垢版2018/09/21(金) 21:44:21.94ID:0uIdegM1
P を正則行列とする。

Inverse[P] * A * P

が対角行列になるような P を求めるということは考えますが、

Transpose[P] * A * P

が対角行列になるような P はなぜ考えないのでしょうか?
0220132人目の素数さん垢版2018/09/21(金) 22:00:46.92ID:Et5XzdMw
対角化は累乗が簡単に求められるからするんです

A^2=PP^(-1)APP^(-1)APP^(-1)=PΛΛP^(-1)

転置でやっても面白いこと起きませんよね
0221132人目の素数さん垢版2018/09/21(金) 22:13:42.26ID:rgDs3VYK
>>219
Aが実対称行列のとき
Transpose[S] * A * S
が±1,0からなる対角行列になるようなSが存在する(シルベスターの標準形)
0223132人目の素数さん垢版2018/09/21(金) 23:33:51.12ID:Zy8fxgFP
「概念」は存在すると言えるのでしょうか?

まず、「事実」は存在すると言えるのかを考えたいと思います。

例えば、目の前にリンゴが全部で10個あるとします。

そうすると、「リンゴが全部で10個あるという事実」は存在すると言えるのでしょうか?

さらに言うと、「リンゴがあるという事実」は存在すると言えるのでしょうか?

目の前にあるリンゴは、物理的に姿形のあるモノとして存在しますが、
そのリンゴがあるという事実はどう考えるのが妥当なのでしょうか?
0224132人目の素数さん垢版2018/09/21(金) 23:46:54.06ID:xIGgPrYx
>>223
哲学板行け
0225132人目の素数さん垢版2018/09/22(土) 01:08:29.78ID:U16PLyIz
自殺して無になってもう二度と有になりたくない。
0226132人目の素数さん垢版2018/09/22(土) 05:35:06.20ID:OM3JlOD/
>>74
とりあえず、n=1〜4で一致する式ができた

∵q={2^n+2^(n−1)+n−4}/{2^(n+2)+5n−14}

n=50のとき、

q=844424930131991/2251799813685366
0227132人目の素数さん垢版2018/09/22(土) 12:33:04.37ID:brB6HAEO
位相空間Xがコンパクトかつハウスドルフならば正規空間であることの証明ですが
これって選択公理使ってますか?
0229132人目の素数さん垢版2018/09/22(土) 13:17:11.20ID:E+fu1y5y
今日も「解いた側」の圧勝かぁ・・・。
毎日毎日、ラクラク解ける問題ばかりだから常勝なんだよね・・・。
たまには、解けない解けないっと悩んで負けてみたい、それが今の切実な悩み。
0232132人目の素数さん垢版2018/09/22(土) 13:36:06.55ID:giDGx0lh
>>231
全ての階に1台ずつ置いとけ
0233132人目の素数さん垢版2018/09/22(土) 13:40:35.45ID:4SLlyIcr
>>207

[9] △OABにおいて、辺OAを 1:3 に内分する点をC, 辺OBを 3:1 に内分する点をDとし、CDを 2:1 に外分する点をEとし、↑OA = ↑a, ↑OB = ↑b とする。
↑OE を↑a, ↑b で表わせ。


[10] 平行四辺形OABCにおいて、↑OA = ↑a, ↑OC = ↑b とする。
次のベクトルを、↑a, ↑b を用いて表わせ。
(1) ↑AB
(2) ↑CA
(3) BCの中点をDとしたときの ↑OD
(4) AB を 2:1 に内分する点Eに対する ↑OE
(5) ↑DE
(6) DEの中点Fに対する ↑OF


↑OC を ↑c にしないセンスがすごい…
0234132人目の素数さん垢版2018/09/22(土) 13:59:22.52ID:4SLlyIcr
>>36

x -1/3 = X, y -1/3 = Y とおくと

x^3 + y^3 - (xx+42xy+yy) = X^3 + Y^3 -42XY -(43/3)(X+Y) -130/27,

チョトちがう
0236132人目の素数さん垢版2018/09/22(土) 15:03:59.24ID:4SLlyIcr
>>177 >>178
 sinθ / (1-2a・cosθ+aa)
= (1/2i){e^(iθ) - e^(-iθ)} / {[1-a e^(iθ)][1-a e^(-iθ)]}
= (1/2ai) { 1/[1-a e^(iθ)] - 1/[1-a e^(-iθ)] }
= (1/2ai)Σ[m=0,∞] {a e^(iθ)}^m - Σ[m=0,∞] {a e^(-iθ)}^m   (← |a|<1)
= (1/2i)Σ[m=0,∞] a^{m-1} {e^(imθ) - e-(-imθ)}
= Σ[m=0,∞] a^{m-1} sin(mθ)
とフーリエ展開する。
和積公式で
∫[0,2π] sin(mθ) sin(nθ) dθ
 = (1/2)∫[0,2π] {cos((m-n)θ) - cos((m+n)θ)}dθ
 = π(δ_{m-n,0} - δ_{m+n,0})
0239132人目の素数さん垢版2018/09/22(土) 18:28:26.02ID:OM3JlOD/
N組のカップル(合わせて2N人)が無作為に横一列に並ぶ
どのカップルについても彼氏と彼女が隣り合わない確率を求めよ

N組のカップルをnとおくと

漸化式があっているかどうかわからないけれど
n=5まで一致する式ができた

   10n^3−n^4−35n^2+62n+12{2^(n−1)+2^n−6}
q=――――――――――――――――――――――――
   2{10n^3−n^4−35n^2+80n+6{2^(n+2)−18}}
0243132人目の素数さん垢版2018/09/22(土) 23:16:08.17ID:7sPQU0EZ
東大数学科で断然トップの人とビル・ゲイツはどっちの方が頭が良いですか?
0245132人目の素数さん垢版2018/09/22(土) 23:29:02.75ID:brB6HAEO
数学というかTeXに関する質問ですが
数式環境内で部分的に地の文にするにはどうしたらいいですか?

例えば、

abc
$x = y. abc f(x)$

と書いた場合、1行目と2行目ではabcの書体・サイズが変わりますが、2行目のabcも1行目のabcと同じ出力にしたいんです。
$x = y.$ abc $f(x)$
という書き直しじゃなく
$$は増やさずに何らかのコマンドで出来ませんか?
0248132人目の素数さん垢版2018/09/22(土) 23:50:26.48ID:JkJqy3uR
リアルタイムに TeX の出力結果が確認できるソフトってありますか?
0249132人目の素数さん垢版2018/09/23(日) 07:41:08.23ID:xBCN748C
シルベスターの慣性法則の「慣性」とは何ですか?
0252132人目の素数さん垢版2018/09/23(日) 12:17:16.99ID:PH84y1u6
ここはわからない問題を書くスレッドです
お願い事をするスレでも誰かに答えてもらえるスレでもありません
0254132人目の素数さん垢版2018/09/23(日) 13:31:14.25ID:n07erhZD
>>237

y = cos(√[1-{arcsin(x)}^2]) ≒ Σ[k=0,∞] c_k x^{2k}  (|x|≦sin(1))

c_0 = cos(1),
c_1 = (1/2)sin(1),
c_2 = (1/24){7sin(1) -3cos(1)}
c_3 = (7/720){22sin(1)-15cos(1)}
c_4 = (1/13440){2427sin(1)-2114cos(1)}

たるんだ放物線?

>>207 >>251
 >>233 を参照。
0255132人目の素数さん垢版2018/09/23(日) 13:45:25.70ID:n07erhZD
>>210

先従解始(先づ解より始めよ) …… 「戦国策」

(大意) 逆向きに解くんでしょうね。

>>237

y = cos(√[1-{arcsin(x)}^2]) ≒ 0.540302 + 0.420735x^2 + 0.177891x^4 + 0.101187x^6 + 0.0669681x^8 + …
0258132人目の素数さん垢版2018/09/23(日) 16:29:59.78ID:uN5miIY2
四色定理「平面上のいかなる地図も、隣接する領域が異なる色になるように塗り分けるには4色あれば十分だ」
この命題中の「平面上のいかなる地図」が地球儀のような「球面上のいかなる地図」となった場合、何色あれば塗り分けるのに十分なんでしょう?
0259132人目の素数さん垢版2018/09/23(日) 16:54:40.83ID:ZHLzUkgh
5色…とか?

最初の平面の地図だと、地図の外側のスペースは無として定義されている。
この無の部分に1つの色を与えて灰色とする。
地図を丸めて球体を作る。
この時、東西南北の端がくっつく部分で、重複が起こらないように灰色の欠片をあてて継ぎ接ぎする。

4色+灰色で5色
0261132人目の素数さん垢版2018/09/23(日) 17:04:38.93ID:krrkUlnT
>>258
球面も彩色数は4

いかなる球面上の地図も、彩色数を変えずに平面地図に置き換えることが可能
0263132人目の素数さん垢版2018/09/23(日) 18:28:52.28ID:6r+HqQTq
置き換えできるとかではなく偶然球面も4色で良かったってだけかもしれないんじゃない?
0264132人目の素数さん垢版2018/09/23(日) 18:44:33.37ID:VgtK+kEe
いや、球面上の地図なら平面上の地図の問題に還元できるやろ?
球面上の地図が与えられたら、いずれかの領域の内点をとって、その点を極としてRiemann球\{極}と平面の一対一対応を考えればいい。
0265132人目の素数さん垢版2018/09/23(日) 19:24:32.54ID:Z1V74VmH
数2の質問です
aを実数の定数とする。xy平面上に2円

c1: x^2+y^2=5

c2: (x-a)^2+(y-2a)^2=2がある。

(1) c1,c2が外接、内接するようなaの範囲をそれぞれ求めよ

(2) a=1のときc1,c2の2交点の座標

解説おねがいします
0266132人目の素数さん垢版2018/09/23(日) 19:53:11.93ID:dnCpmMyL
>>265
ちゅうしんとちゅうしんのきょりをかんがえる
多分教科書に似たような問題ある(傍用にもある)
交点の座標は計算する
計算の仕方も大事
0267132人目の素数さん垢版2018/09/23(日) 19:56:52.35ID:7FSyqEIr
>>265
c1の中心が(0,0)で半径が√5
c2の中心が(a,2a)で半径が√2
中心間の距離は(√5) |a|

(1)
外接する時
中心間の距離が、半径の和に等しいので
(√5) |a| = (√5) + √2
a = ±{1 + √(2/5)}

内接する時
中心間の距離が、半径の差に等しいので
(√5) |a| = (√5) - √2
a = ±{1 - √(2/5)}

(2)
x^2 +y^2 = 5
(x-1)^2 +(y-2)^2 = 2
上から下を引いて
2x +4y -5 = 3
x + 2y = 4
x = -2y +4
最初の式に代入して
(-2y +4)^2 +y^2 = 5
5y^2 -16y +11 = 0
(5y -11)(y-1) = 0
y = 11/5, 1

y = 11/5 のとき x = -2/5
y = 1 のとき x = 2
0268132人目の素数さん垢版2018/09/23(日) 21:21:19.85ID:ZHLzUkgh
>>264
平面を球面に置き換えて同じ結論がえられるってまじかよ、
それじゃ >>259がバカみたいじゃん。
0269132人目の素数さん垢版2018/09/23(日) 21:30:56.29ID:7FSyqEIr
>>268
というか、この手の発想は3色では不可能な事の証明でもよく使われる
知らない人は悩むってだけで
正四面体の面の塗分けは3色では不可能だから
面の1つに穴を開けて
(面はゴムのようなものでできていると思って)平面上に広げれば
3色で塗分け不可能な地図ができる
って具合に
0270132人目の素数さん垢版2018/09/23(日) 23:18:08.51ID:KSTpRWA6
四色定理の空間バージョンの定理ってありますか?
つまり、例えば、立体パズルにおいて隣接してる(0以上の面積を共有してる)ピースは別の色にして塗るということにした場合
何色あれば十分ですか?
0272132人目の素数さん垢版2018/09/24(月) 00:14:43.51ID:ccjS23v2
>>270
空間をいくつかの領域にわけるという意味なら明らかに何色あっても無理。
100色用意しても101完全グラフ用意して各点にたいし、その点とその点から出てる確辺のまん中までを1領域とする分割を考えれば100色では無理。
E^2に埋め込めない一般の場合という意味ならその地図を埋め込める種数ごとに必要最低限度の色数は決定されてる。
https://ja.wikipedia.org/wiki/%E5%9B%9B%E8%89%B2%E5%AE%9A%E7%90%86
0273132人目の素数さん垢版2018/09/24(月) 01:52:24.23ID:Ple4QkIq
>>239
n=6まで一致する式ができた

   2n^5−63n^4+500n^3−1605n^2+2594n+297×2^(n+1)−2616
q=―――――――――――――――――――――――――――――――――
   66{10n^3−n^4−35n^2+80n+6{2^(n+2)−18}}
0274132人目の素数さん垢版2018/09/24(月) 02:45:52.27ID:f7uXOSwA
最適化問題です。
どういった方法で解を出すかという方針
だけでも教えていただきたいです。


変数Piとして、それ以外は定数とする。
min 煤mi=1からN]Pi

条件
0≦Pi≦Pmax(i=1,,,N)
Σ[i=1からN]A×Pi+煤mi=1からN、ただしi≒j]Σ[j=1からN]√(PiPj)×B ≧C
0275132人目の素数さん垢版2018/09/24(月) 02:47:36.97ID:f7uXOSwA
>>274
?になっている部分はシグマです
0276132人目の素数さん垢版2018/09/24(月) 04:36:49.10ID:WgV4wCes
数学IIの図形と方程式の問題です。

(1)以下の不等式で表されるxy平面上の領域Dを図示せよ。
(x+y-1)(-2x+y-3)(-x-2y+4)≧0

(2)一辺の長さ1の正三角形Tをxy平面上に置く。TとDの重なる部分の面積を最大にするようにTを置くときのGの座標を求めよ。
ただしGはTの重心である。
0277132人目の素数さん垢版2018/09/24(月) 11:20:03.20ID:C29H7b6e
>>236
Σ[m=0,∞] a^m e^(imθ)
 = Σ[m=0,∞] {a e^(imθ)}^m
 = 1/{1-a e^(iθ)}
 = {1-a e^(-iθ)}/(1-2a・cosθ+aa)
 = {(1-acosθ) +ia sinθ}/(1-2a・cosθ+aa),
の虚部から
Σ[m=1,∞] a^m sin(mθ) = a・sinθ/(1-2a・cosθ+aa),

一方、実部から
Σ[m=0,∞] a^m cos(mθ) = (1-a cosθ)/(1-2a・cosθ+aa),

1/(1-2a・cosθ+aa) = {1/(1-aa)}{1 + 2Σ[m=1,∞] (a^m)cos(mθ)},

2a cosθ/(1-2a・cosθ+aa) = (1+aa)/(1-2a・cosθ+aa) -1,
0278132人目の素数さん垢版2018/09/24(月) 16:56:33.37ID:Y2Cz0M7v
(X_i) は i∈I を添え字集合とする集合列とします
Pr_i は Π_i X_i の第i射影とします

知られている通り、 Pr_i(Π_j X_j)=X_i ですが、この証明(⊇について)には選択公理を使いますよね?
0279132人目の素数さん垢版2018/09/24(月) 17:18:28.20ID:3sb6z9vD
定理 … 公理を用いて証明された命題

公理 … 証明が不要で前提とする事柄

↑ とあります。
高校までの数学で作られてからもっとも新しい公理 (理論) って何ですか?
複素平面? 微積分?
0280132人目の素数さん垢版2018/09/24(月) 17:28:30.79ID:nFKM7Z34
>>279
高校数学はそういう難しいことは考えないで適当に作られてますから考えるだけ無駄です
0282132人目の素数さん垢版2018/09/24(月) 21:09:11.61ID:uyI4OG9o
曲線Cをy=sin(πx)の0≤x≤1の部分とする。
また以下の曲線Dと直線Eはいずれも、Cとx軸とで囲まれる部分の面積を2等分するという。
正数a,bの大小を比較せよ。
D y=asin(πx/2)
E: y=bx
0283132人目の素数さん垢版2018/09/25(火) 00:16:21.34ID:Mf+IIU9l
>>282
曲線Cとx軸で囲まれる部分の面積は
 ∫[0,1] sin(πx) dx = [ -(1/π)cos(πx) ](x=0,1) = 2/π = 0.636619772367581343

a = 0.5857864376268

b = 0.8062893052025

∴ a < b
0286132人目の素数さん垢版2018/09/25(火) 04:44:40.72ID:Mf+IIU9l
>>283

C: y = sin(πx),
D: y = a sin(πx/2), a = 0.5857864376268
E: y = b x,     b = 0.8062893052025

CとDの交点 (x,y) = (0.810763906019775 , 0.5600968657158)

CとEの交点 (x,y) = (0.782633029520911 , 0.6310286460088)

DとEの交点 (x,y) = (0.559244088133690 , 0.4509125272599)
0288132人目の素数さん垢版2018/09/25(火) 15:41:23.42ID:gzqxMuxe
2^2-1^2、3^2-2^2、4^2-3^2・・・
と続く数列の答えはそれぞれ2n-1になるらしいけど、
方程式では解けてもなぜそうなるか疑問です。
丁寧に答えて下さる方いませんか
0289132人目の素数さん垢版2018/09/25(火) 15:53:57.58ID:RwC3xJIG
計算したらそうなったんですよね
だからそういうもんだ、でいいんですよ
そのための文字式なんです
何にでもそういう理由を求めようとするのは、疲れるだけであまり本質ではないことが多いですからやめといた方が良いでしょうね

でも今回の場合は正方形考えるといいかとしれないですね
玉を正方形に並べます
一列増やしてちょっと大きな正方形作るにはどうすれば良いでしょうか
0290132人目の素数さん垢版2018/09/25(火) 16:10:47.12ID:Mf+IIU9l
>>284
 aの方は

CとDの交点を(c, d) とおく。
 sin(πc) = a sin(πc/2),
 a = 2 cos(πc/2),
より
∫[0,c] {sin(πx) - a sin(πx/2)} dx = (1/2π)(4-aa) -(a/π)(2-a) = (1/2π)(2-a)^2,
これが 1/π に等しいから、
 a = 2-√2 = 0.585786437626905
 c = (1/π)arccos(2(1-√2)) = (2/π)arccos(1-(1/√2)) = 0.810763906019740
 d = sin(πc) = (√2 -1)√(2√2 -1) = 0.560096865715887

bの方は分かりませぬ…
0291132人目の素数さん垢版2018/09/25(火) 16:10:52.13ID:q3cJ7uMj
●●●○
●●●○
●●●○
○○○ +○

タテ3✕ヨコ3に並べた丸に●に、

○をタテ3コ、ヨコ3コ、角っこうめるためもう1コ付けると4✕4になりますね

3^2(もと●)  +  3*2+1(追加○) =4^2

こういうことです。
0292132人目の素数さん垢版2018/09/25(火) 17:34:12.78ID:gzqxMuxe
>>289
>>291
確かにそういう計算をしてることになりますね!数式って凄いなあ
0293132人目の素数さん垢版2018/09/25(火) 18:15:15.20ID:QJVCmX3z
次の無限級数が収束するxの範囲をそれぞれ求めよという問題です
一様収束ではなく収束なので解き方が分からないですどうかお助けを……

Σ[n=1,∞]1/(1+nx^n)
Σ[n=1,∞]1/(n^2-x)
Σ[n=1,∞]|x|/(1+|x|)^n
0294132人目の素数さん垢版2018/09/25(火) 18:49:12.43ID:Oj/s8CIQ
>>273
n=7まで一致する式ができた

   1783n^5−83n^6−15785n^4+71005n^3−166892n^2+198292n+1485×2^(n+3)−112080
q=―――――――――――――――――――――――――――――――――――――――――――――――
   66{63n^5−3n^6−545n^4+2405n^3−5572n^2+6892n+480(2^n−9)}
0295132人目の素数さん垢版2018/09/25(火) 20:56:27.99ID:LFmeOtFE
>>293
Σ[n=1,∞]1/(1+nx^n)
|x|<1のときは項が0に収束しない。|x|>1のときは絶対収束する。
x=1のときは対数発散する。x=-1のときはn=1の項が1/0になって未定義。(n=1の項が無ければ条件収束)

Σ[n=1,∞]1/(n^2-x)
x=-1,-4,-9,-16,... なら1/0の項が出てくるので未定義。それ以外なら絶対収束する。

Σ[n=1,∞]|x|/(1+|x|)^n
具体的に計算できる。x=0のとき0、それ以外のとき1に収束する。
0296132人目の素数さん垢版2018/09/25(火) 21:41:16.32ID:n/GFgogk
集合Sに対して、P(S)でSの巾集合を表す。
Fin(S) := {A∈P(S)|Aは有限集合} とする。

Xを集合とする。
S⊆P(X)とする。
O(S)でSによって生成される開集合系とする。

O(S)を具体的に表したい。

O(S) = { ∪_{T ∈ F} ∩T | F ⊆ Fin(S) }
でいいんですかね?
0298132人目の素数さん垢版2018/09/25(火) 23:14:24.80ID:w+XVQKQt
二次関数の最大と最小を求める時に最後
8a-4とかの文字式が答えになるんですがどこをどう代入すればこの式になるか分かりません
グラフは描けるんですが…
0303132人目の素数さん垢版2018/09/25(火) 23:57:46.88ID:Y5pYVzUb
>>302
どの問題について聞いてるの?
そのページのどの問題を解いても
8a-4なんて式は出てこないようだが
0304132人目の素数さん垢版2018/09/26(水) 00:00:10.49ID:Cc/6inZ7
>>301
Σ[n=1,∞]1/(1+nx^n)
|x|<1のときは項が0に収束しない。←自明
|x|>1のときは絶対収束する。←n≧2のとき |1+nx^n| > (n|x|^n)-1 > |x|^n と評価する。
x=1のときは対数発散する。← 1/(1+n) > ∫[n+1〜n+2] (1/x) dx と評価する。
x=-1のときはn=1の項が1/0になって未定義。(n=1の項が無ければ条件収束)←絶対値が単調減少する交代級数は収束する。

Σ[n=1,∞]1/(n^2-x) 訂正
× x=-1,-4,-9,-16,... なら1/0の項が出てくる
○ x=1,4,9,16,... なら1/0の項が出てくる

xがこれらの値以外であるとき m^2-x>0 を満たすmを適当に選ぶと n≧m+1 のとき
n^2-x = (n-m)^2 + 2nm + m^2 - x > (n-m)^2
Σ[n=1,∞]|1/(n^2-x)| < Σ[n=1,m]|1/(n^2-x)| + Σ[n=m+1,∞]1/(n-m)^2 < ∞

Σ[n=1,∞]|x|/(1+|x|)^n
ただの等比級数の和
0306132人目の素数さん垢版2018/09/26(水) 01:05:06.95ID:bHGY9i2p
>>303
適当な例題をアップしてしまったのが悪かったですね…
8a-4のことは忘れていただいて構いません

a<0のとき 最小値a^2+1
0≦a≦2のとき…
とあるんですが問題の始めに与えられた式y=x^2-2ax+a^2+1 (0≦a≦2)
からa^2+1などの文字式をどうやって導き出すのかが分からないんです
0307132人目の素数さん垢版2018/09/26(水) 01:41:12.56ID:2yFoJMu6
>>306
ちゃんと例題の真似をして解いたのか?
区間の両端か軸での値として計算すれば出てくるはず
0311132人目の素数さん垢版2018/09/26(水) 02:23:48.47ID:u24AtJNa
真面目に教えてください。お願いします。
0313132人目の素数さん垢版2018/09/26(水) 02:54:12.98ID:GaEXENYv
真面目に教えてください。お願いします。
0314132人目の素数さん垢版2018/09/26(水) 05:33:34.75ID:WJI1Ssah
∠B=∠Cである△ABCがある。
その辺CAを一辺とする正三角形△CADで、頂点Dが直線CAに関してBと反対側にあるようなものを作る。
このとき、以下の問いに答えよ。

(1)∠Bの内角を2等分する直線Lの上に△CADの内心Iが乗るという。△ABCの形状はどのようであるか述べよ。

(2)(1)において、内心Iを以下に置き換えた場合、△ABCの形状はどのようであるかを述べよ。
 (i) 外心O
 (ii) 重心G
 (iii) 垂心H
0315132人目の素数さん垢版2018/09/26(水) 05:56:33.12ID:WJI1Ssah
大量の白板と黒板があり、どちらの板も一辺の長さが1の正方形の形状をしている。
いま床の上に白板1枚が置かれている。
この状態から次のような操作(T)を行う。

(T)表が出る確率が0.8のコインがある。
このコインを振って表が出れば、一番右側の板に白板1枚を貼り付ける。
ただし板が1枚の場合はその板を「一番右側の板」とみなす。以下も同様である。
裏が出れば、一番右側の板に黒板k枚を貼り付ける。ここでkは自然数である。
いずれの操作を行った場合も、板を貼り付けて出来上がった新しい板は、縦の長さが1、横の長さが1より大きい自然数の長方形となる。

このとき、以下の問いに答えよ。

(1)(T)を繰り返し、板の並びに「黒白黒」が現れた時点で操作を終了する。最終的に出来上がった長方形の横の長さの期待値E(k)をkで表せ。

(2)8≦E(k)≦10となるkの範囲を求めよ。
0317132人目の素数さん垢版2018/09/26(水) 07:54:54.86ID:roNfZuDf
5人中3人が1列に並ぶときの並び方の総数を求めなさい。

お願いします。。。
0318132人目の素数さん垢版2018/09/26(水) 10:59:00.34ID:TpX5a0Yg
>>317
それくらいはまず書き出せよ
どうすればもれなく書き出せるかを考えてみれば数式もたぶんわかる
0320132人目の素数さん垢版2018/09/26(水) 13:11:13.74ID:zomwMvsu
>>319

証明は、Casoratian の定義式だけあればよく、
C(r) = | u1(r) u2(r) |
    | u1(r+1) u2(r+1) |
  = u1(r) u2(r+1) - u2(r) u1(r+1)
  = u1(r) u1(r+1) {u2(r+1)/u1(r+1) - u2(r)/u1(r)}
  = u1(r) u1(r+1) Δ{u2(r)/u1(r)},
よって
u2(n)/u1(n) = u2(0)/u1(0) + Σ[r=0,n-1] Δ{u2(r)/u1(r)}
  = u2(0)/u1(0) + Σ[r=0,n-1] C(r)/{u1(r)u1(r+1)},
 ここで u2(0)=0 を使うと…

Casoratian はつまり Wronskian の 差分version かな。
0321132人目の素数さん垢版2018/09/26(水) 13:16:03.71ID:vAGGSnkZ
>>320
u2(0)=0とはどこにも書いてないんですけど?
0323132人目の素数さん垢版2018/09/26(水) 13:34:08.83ID:vAGGSnkZ
>>322
画像の黄色く光っているところの文字列をグーグルで検索してみてください
0325132人目の素数さん垢版2018/09/26(水) 14:25:32.07ID:zomwMvsu
〔問題〕
次の2階差分方程式を考えよう。
 u(n+2) + p1(n) u(n+1) + p2 u(n) = 0,
その解を u1(n),u2(n)、それらのCasoratian を C(n) とするとき
 C(n+1) = p2 C(n) = …… = (p2)^{n+1} C(0),
を示せ。

このスレも 過疎らし庵...
0327132人目の素数さん垢版2018/09/26(水) 14:47:30.72ID:o1ctSWEs
>>315
(T)をシミュレーションしてみました。
黒白黒=裏表裏と続くときの表と裏の回数の表の回数、裏の回数の10万回シミュレーションでの平均値は

[1] 28.98207
[1] 7.24779

長方形の横の長さの期待値E(k)は 28.98207 + 7.24779*k

に近似するという結果が得られました。

解析でとく頭はないのでご容赦。
0328132人目の素数さん垢版2018/09/26(水) 15:02:53.21ID:zomwMvsu
>>293
蛇足ですが…

(2) 無限級数Σ[n=1,∞] 1/(nn-x) は x≠平方数 のとき収束し、

x>0,x≠平方数のとき  {1 − (π√x) cot(π√x)}/2x,
x=0 のとき        ζ(2) = ππ/6 = 1.644934…
x<0 のとき         {(π√(-x))coth(π√(-x)) − 1}/2(-x),
0329132人目の素数さん垢版2018/09/26(水) 15:08:05.91ID:o1ctSWEs
>>318
総数より、列挙する方が難しかった。注目する3人が1,2,3とするとその並び方は

> perm[i,]
[,1] [,2] [,3] [,4] [,5]
[1,] 1 2 3 4 5
[2,] 1 2 3 5 4
[3,] 1 3 2 4 5
[4,] 1 3 2 5 4
[5,] 2 1 3 4 5
[6,] 2 1 3 5 4
[7,] 2 3 1 4 5
[8,] 2 3 1 5 4
[9,] 3 1 2 4 5
[10,] 3 1 2 5 4
[11,] 3 2 1 4 5
[12,] 3 2 1 5 4
[13,] 4 1 2 3 5
[14,] 4 1 3 2 5
[15,] 4 2 1 3 5
[16,] 4 2 3 1 5
[17,] 4 3 1 2 5
[18,] 4 3 2 1 5
[19,] 4 5 1 2 3
[20,] 4 5 1 3 2
[21,] 4 5 2 1 3
[22,] 4 5 2 3 1
[23,] 4 5 3 1 2
[24,] 4 5 3 2 1
[25,] 5 1 2 3 4
[26,] 5 1 3 2 4
[27,] 5 2 1 3 4
[28,] 5 2 3 1 4
[29,] 5 3 1 2 4
[30,] 5 3 2 1 4
[31,] 5 4 1 2 3
[32,] 5 4 1 3 2
[33,] 5 4 2 1 3
[34,] 5 4 2 3 1
[35,] 5 4 3 1 2
[36,] 5 4 3 2 1
>
0331132人目の素数さん垢版2018/09/26(水) 15:54:28.25ID:zomwMvsu
>>328 補足

x > 0, x≠平方数のとき
 y≒0 では πcot(πy) ≒ 1/y,
また、cot(πy) は周期1をもつから、
 πcot(πy) = 1/y + Σ[n=1,∞] {1/(y-n) + 1/(y+n)}
     = 1/y + 2yΣ[n=1,∞] 1/(yy-nn),

x<0 のとき
 y≒0 では πcoth(πy) ≒ 1/y,
また、coth(πy) は周期 i をもつから、
 πcoth(πy) = 1/y + Σ[n=1,∞] {1/(y-ni) + 1/(y+ni)}
     = 1/y + 2yΣ[n=1,∞] 1/(yy+nn),
0332132人目の素数さん垢版2018/09/26(水) 16:41:05.92ID:roNfZuDf
>>318

理解できました!
5C3 だと思っていましたが、5P3でしたね。。

どうもすみませんでした。
0333132人目の素数さん垢版2018/09/26(水) 16:53:15.04ID:D649zj2u
【天文台閉鎖、FBI】 アポロ捏造のキューブリックも真っ青、太陽に映ったのはマ@トレーヤのUFO
http://rosie.5ch.net/test/read.cgi/liveplus/1537840672/l50

おまいらが注目しないから宇宙人は出てこれない、その結果、地球の放射能危機がどんどん進んでしまう!
0334132人目の素数さん垢版2018/09/26(水) 16:57:12.23ID:WJI1Ssah
(1)k! + m! = n!を満たす自然数の組(k,m,n)をすべて求めよ。

(2)いずれも2以上の自然数かつすべて異なる自然数の組(m,n,p,q,r,s)で、以下の等式を満たすものは存在するか。
mCn = pCq + rCs
0335132人目の素数さん垢版2018/09/26(水) 17:19:51.16ID:EZjvvW8g
>>294
n=8まで一致する式ができた

   7{589n^7−76252n^6+1473418n^5−12519640n^4+55110541n^3−127896988n^2+150467292n+66825×2^(n+7)−83666160}
q=――――――――――――――――――――――――――――――――――――――――――――――――――――――――
   495{34286n^5−25n^7−1316n^6−317240n^4+1446935n^3−3416084n^2+4304724n+5040{2^(n+6)−551}}
0337132人目の素数さん垢版2018/09/26(水) 17:40:42.99ID:WJI1Ssah
領域3x^3+(4y-1)x^2-(37y^2+22y-1)x+(14y^3+23y^2-6y)≧0
と直線x=tとの共有点のうち、y座標が最大となるものの座標を求めよ。
0341132人目の素数さん垢版2018/09/26(水) 21:04:01.46ID:kMXjNQ4p
>>334 (1)
k! < k! + m! = n! より k < n。
よって、k!/n! ≦ (n-1)!/n! = 1/n。同様に、m!/n! ≦ 1/n。
1 = n!/n! = k!/n! + m!/n! ≦ 2/n より、n≦2。
したがって、(k,m,n)=(1,1,2)のみ。
0342132人目の素数さん垢版2018/09/26(水) 21:25:13.59ID:o1ctSWEs
Haskell先生に100以下を計算してもらいました。

Prelude> let fact n = if n == 0 then 1 else n * fact (n - 1)
Prelude> print [(k,m,n) | k <- [1..100], m <- [1..100], n <- [1..100], fact(k) + fact(m) == fact(n) ]
[(1,1,2)]
0343132人目の素数さん垢版2018/09/26(水) 23:20:55.60ID:LiB/jXp0
よろしくお願いします。

モルモットにAを投薬したところ、
250匹中200匹の治療に成功した。
B薬の場合は、180匹中162匹であった。
B薬の方がA薬より有効性が高いかどうか、有意水準5%で検定しなさい。
0346132人目の素数さん垢版2018/09/26(水) 23:52:50.37ID:o1ctSWEs
>>334

6C2=15
5C4=5
10C9=10

6C2 = 5C4 + 10C9

10以下の組み合わせをHaskellで出すと
[(6,2,5,4,10,9),(6,2,10,9,5,4),(9,2,6,4,7,5),(9,2,7,5,6,4),(10,2,5,3,7,4),
(10,2,7,4,5,3),(6,3,5,2,10,9),(6,3,10,9,5,2),(9,4,10,3,6,5),(9,4,6,5,10,3),
(9,4,6,5,10,7),(9,4,10,7,6,5),(8,5,9,2,6,3),(8,5,6,3,9,2),(8,5,6,3,9,7),
(8,5,9,7,6,3),(9,5,4,2,10,3),(9,5,4,2,10,7),(9,5,10,3,4,2),(9,5,10,7,4,2),
(8,7,3,2,5,4),(8,7,5,4,3,2),(9,8,3,2,6,5),(9,8,6,5,3,2),(10,8,5,2,7,3),
(10,8,5,2,7,4),(10,8,5,3,7,4),(10,8,7,3,5,2),(10,8,7,4,5,2),(10,8,7,4,5,3),
(10,9,3,2,7,6),(10,9,4,3,6,5),(10,9,6,5,4,3),(10,9,7,6,3,2)]
0348132人目の素数さん垢版2018/09/27(木) 00:03:24.10ID:QdrW3DdV
>>343
> prop.test(c(200,162),c(250,180))

2-sample test for equality of proportions with continuity
correction

data: c(200, 162) out of c(250, 180)
X-squared = 7.1275, df = 1, p-value = 0.007591
alternative hypothesis: two.sided
95 percent confidence interval:
-0.17095378 -0.02904622
sample estimates:
prop 1 prop 2
0.8 0.9
0349132人目の素数さん垢版2018/09/27(木) 00:04:45.01ID:QdrW3DdV
>>344
> prop.test(c(200,225),c(250,250))

2-sample test for equality of proportions with continuity
correction

data: c(200, 225) out of c(250, 250)
X-squared = 9.0353, df = 1, p-value = 0.002648
alternative hypothesis: two.sided
95 percent confidence interval:
-0.1659795 -0.0340205
sample estimates:
prop 1 prop 2
0.8 0.9
0350132人目の素数さん垢版2018/09/27(木) 00:06:43.63ID:83McNs2U
>>304
答えて貰って恐縮なのですが
Σ[n=1,∞]1/(n^2-x)の解説において
n^2-x = (n-m)^2 + 2nm + m^2 - x > (n-m)^2
とありますが等号の変形間違っていませんか?そうすると後の式も導けないような
勘違いでしたらすみません
0351132人目の素数さん垢版2018/09/27(木) 00:16:15.99ID:jPVYoETD
流れをぶった切る割に、皆さまにとっては簡単な問題で申し訳ないですが、f(x)=(2x-1)/(x-x^2)の逆関数を求めることができません。
どなたかご教授いただけないでしょうか。
よろしくお願いします。
0354132人目の素数さん垢版2018/09/27(木) 00:34:30.52ID:+C9yx15o
三角関数がまったく理解できないのですが、どうすれば理解できるようになりますか?
勉強する際のコツなどがあれば教えてください。
0357132人目の素数さん垢版2018/09/27(木) 00:42:54.96ID:jPVYoETD
>>353,354
ご教授いただきありがとうございました。
ゆっくり検算等を行って理解を深めていきたいと思います。
0358132人目の素数さん垢版2018/09/27(木) 00:54:20.65ID:+C9yx15o
>>356
二項定理はどうやったら理解できるようになるのでしょうか?
コツを教えてください。
0359132人目の素数さん垢版2018/09/27(木) 02:53:23.11ID:bYCrvdC8
>>334
(1)
k≦m<n としてもよい。このとき
1 = (n! - m!)/k! = (m!/k!){(n!/m!) - 1}
∴ (m!/k!) = 1, (n!/m!) -1 = 1,
∴ (k, m, n) = (1, 1, 2)

(2)
 n=m-1, q=p-1, s=r-1
のとき
 C[m, n] = C[m, m-1] = m,
 C[p, q] = C[p, p-1] = p,
 C[r, s] = C[r, r-1] = r,
そこで m = p+r とする。
但し m≧8, m-3≧p≧[(m+1)/2]+1, [m/2]-1≧r≧3,
 m>n>p>q>r>s.
最小解は (m, n, p, q, r, s) = (8, 7, 5, 4, 3, 2)
0360132人目の素数さん垢版2018/09/27(木) 03:06:05.22ID:bYCrvdC8
>>331 によれば
 πcot(πy) = 1/y + 2yΣ[n=1,∞] 1/(yy-nn),
 πcoth(πy) = 1/y + 2yΣ[n=1,∞] 1/(yy+nn),
y で積分すれば
 log|sin(πy)| = log|y| + Σ[n=1,∞] log|1 - (y/n)^2| + logπ,
 log|sinh(πy)| = log|y| + Σ[n=1,∞] log|1 + (y/n)^2| + logπ,
よって
 sin(πy) = πy・Π[n=1,∞] {1 - (y/n)^2},
 sinh(πy) = πy・Π[n=1,∞] {1 + (y/n)^2},
yを1/2ずらせば 同様に
 cos(πy) = Π[n=1,∞] {1 - yy/(n-1/2)^2},
 cosh(πy) = Π[n=1,∞] {1 + yy/(n-1/2)^2},
          … オイラーの無限乗積表示
0361132人目の素数さん垢版2018/09/27(木) 03:22:29.86ID:7YOH+E82
P≠NP予想の証明に取り掛かろうと思うのですが、これを証明するにはまずは何を勉強した方が良いのでしょうか?
数学だけでなく計算機科学とか物理学も勉強した方が良いですか?
0362132人目の素数さん垢版2018/09/27(木) 03:26:58.52ID:bYCrvdC8
>>334 (2)
n = m-1 のとき
C[m, n] = C[m, m-1] = m,

p, q, r, s はいずれも2以上の自然数かつすべて異なる。
m = C[p, q] + C[r, s]
とおく。
0364132人目の素数さん垢版2018/09/27(木) 13:13:58.23ID:QdrW3DdV
>>362
(1)の延長で(2)は存在しないという答になるのかと思っていたんだけど
(1)と(2)は無関係だったのかなぁ?
0365132人目の素数さん垢版2018/09/27(木) 15:12:22.52ID:bYCrvdC8
>>334 >>359
(2)
 n=m-2, q=p-2, s=r-2
のとき
 C[m, n] = C[m, m-2] = m(m-1)/2,
 C[p, q] = C[p, p-2] = p(p-1)/2,
そこで
 m = p+1, p = C[r, s], n=p-1, q=p-2, 但し r>s
とする。
 m>p>n>q>r>s.
 C[p, p-2] + C[r, s] = p(p-1)/2 + p = p(p+1)/2 = C[p+1, p-1]
0366132人目の素数さん垢版2018/09/27(木) 15:26:27.33ID:P8qJtskS
平面上に△ABCを与える(固定する)。その内角∠Bを2等分する直線をLとする。
また、直線CAに関してBと反対側の領域を動く点Pがあり、△PACの内心をIとする。
以下の問いに答えよ。

(1)相異なる定点S,Tと、動点Xがある。Xが色々動くとき、△STXの内心Uが動ける領域を求めよ。

(2)△ABCの内心をJとする。点Pが色々動くとき、与えられた△ABCの形状にかかわらず、次の条件を満たす点Pの位置が少なくとも1つ存在すると言えるか。
「Lは4点B,J,I,Pの全てを通る」
0368132人目の素数さん垢版2018/09/27(木) 17:39:33.84ID:bYCrvdC8
>>337

3t^3 + (4y-1)t^2 - (37y^2 +22y-1)t + (14y^3 +23y^2 -6y)
= 14y^3 + (23-37t)y^2 - (6 +22t -4tt)y + (t -t^2 +3t^3)
= 14 (Y^3 -3PY +2Q),
ここに
 P(t) = (781-778t+1201tt)/(42^2),
 Q(t) = (20861 -38181t +34737t^2 -33391t^3)/(42^3),
 Y = y + (23-37t)/42,

さて、どうするか?
0369132人目の素数さん垢版2018/09/27(木) 18:44:49.70ID:bYCrvdC8
>>367
そのまま解く。

第2問 2.
(i)
 -{d^2 u/(dx)^2} + 2λ^2 {u(x)^3 - u(x)} = 0,   … (3)
の両辺に du/dx をかけて、
 -{d^2 u/(dx)^2}(du/dx) + 2λ^2 {u(x)^3 -u(x)}(du/dx) = 0,
その積分を求めると
 -(1/2)(du/dx)^2 + 2λ^2 {(1/4)u(x)^4 -(1/2)u(x)^2} = c,
 -(1/2)(du/dx)^2 + (1/2)λ^2 {u(x)^4 -2u(x)^2 +A} = 0,
 du/dx = ±λ√{u(x)^4 -2u(x)^2 +A},    … (4)
が成立する。ここで、Aは積分の定数である。

(ii)
 x→±∞ のとき u(x) →±1, du/dx →0 より A=1
 また du/dx > 0 となる所がある。

(iii)
 du/dx >0, λ>0, |u(x)|≦1 により
 du/dx = λ{1 - u(x)^2}

 {1/(1-u) + 1/(1+u)}(du/dx) = 2λ

 log((1+u)/(1-u)) = 2λx+2c,
u(0)=0 ゆえ c=0
 u(x) = tanh(λx),
0370学術垢版2018/09/27(木) 18:58:53.42ID:8ZNOee3m
よくできているが、単数では数字にイメージがわかないから、割り算や
分数、二次以上の関数や漠然とした少数を乱用する方が自然界のイメージには近いでしょう。
0371132人目の素数さん垢版2018/09/27(木) 19:05:23.10ID:6Mk1qjy4
R上ユークリッド位相間の写像fが連続かつ狭義単調増加のとき開写像であることを示して下さい
0372学術垢版2018/09/27(木) 19:10:09.38ID:8ZNOee3m
漢文では、数理が表現できないから、創造と違うものが、示されるべきで。
0373学術垢版2018/09/27(木) 19:10:26.28ID:8ZNOee3m
想像と。
0374学術垢版2018/09/27(木) 19:12:38.36ID:8ZNOee3m
上ののも見返して、考え直してね。
0375学術垢版2018/09/27(木) 19:18:48.36ID:8ZNOee3m
裏を返せばそれで表象されるもの自体が、数式から独立して離れて、
独り歩きするようになる方が、心理に近いということ。
0376学術垢版2018/09/27(木) 19:19:28.88ID:8ZNOee3m
イメージにあるものが吹き出しにかかれるなら、数学者のマンガ
なんてバカ売れするだろうな。
0377132人目の素数さん垢版2018/09/27(木) 19:38:26.28ID:W0ybPQXa
>>371
任意の x に対し快区間 U = (f(x-1),f(x+1)) は仮定よりf(x)の開近傍。
y ∈ Uに対し中間値の定理よりyはim fに含まれる。
すなわち U ⊂ im f である。
よって
im f = ∪ [x ∈ im f] (f(x-1), f(x+1))
は開集合。
0378学術垢版2018/09/27(木) 19:42:23.98ID:8ZNOee3m
短文だね。ヴィトゲンシュタイン〜ピタゴラスからの何たる零落だろう。
0380132人目の素数さん垢版2018/09/27(木) 23:14:51.35ID:83McNs2U
自分も位相についての質問です
位相間の連続写像fi:S'→Siが存在するとき
Siの直積位相Sに対してg:S'→S、fi=pri*g(priはSiへの射影)となるような連続写像gが一意的に存在することを証明せよという問題です
連続になることはわかりますがそもそも存在の証明方法がわからず詰まっていますので助けて下さい
0381132人目の素数さん垢版2018/09/28(金) 00:19:08.98ID:wpvX3I7e
その写像gを作ればいいだけ。
必要な情報はすべて問題の中に書かれている。
即ち、s∈S'に対してg(s)=(t_{i})∈ΣS_{i} と表される筈であるが、
そのときこの各t_{i} はどうなっていなければならないかを考える。
0383132人目の素数さん垢版2018/09/28(金) 00:30:53.10ID:geQfbUSq
>>381
ありがとうございます
よく考えてみたいと思います
>>382
一応gの存在を認めるとfi=pri*gやfiとpriの連続性からgも連続であることが言えませんか?
0385132人目の素数さん垢版2018/09/28(金) 01:00:58.02ID:ssYGT9g8
器用なやっちゃな。でも初等開集合の原像がどうなるかは考えといた方がいいと思うぞ。
0386132人目の素数さん垢版2018/09/28(金) 01:38:16.42ID:nYhI5qFO
「無」と「数学の未解決問題全てを1分50秒で証明した人」はどっちの方が凄いですか?
0387132人目の素数さん垢版2018/09/28(金) 02:00:15.16ID:ssYGT9g8
つーかよく考えたらf∘gが連続でfが連続でもgが連続とは言えなかった。
例えばg(x)=-1 (x<0), g(x)=1 (x>=0), f(x)=|x| と置けば(f∘g)(x)=1だべ。
0388132人目の素数さん垢版2018/09/28(金) 09:40:21.64ID:phrHQfEJ
>>86

漸化式から、n>>1 では
 a[n] 〜 α{1 -1/(4n) -3/(32n^2) -1/(384n^3) +361/(6144n^4) +12799/(122880n^5) +(377221/2449120n^6) + …}
    〜 α(1 - 1/n)^(1/4),

 ここに α = lim(n→∞) a[n],

[前スレ.609] では
 a[1] = 0, a[2] = 1/3, a[3] = 1/3, a[4] = 12/35, a[5] = 47/135,
 a[6] = 731/2079, a[7] = 1772/5005, a[8] = 20609/57915,
 a[9] = 1119109/3132675, a[10] = 511144/1426425, …, a[∞] = 1/e
0391132人目の素数さん垢版2018/09/28(金) 18:30:53.14ID:0t11U44j
>>390
んなの闇の帝王 フグ田タラオの前では
どんぐりの性比べ程度の違いしかない
0392132人目の素数さん垢版2018/09/28(金) 19:48:15.52ID:agTum+EB
pが素数、m,nが自然数のとき
p^m+1=m^nを満たす(p,m,n)の組み合わせを全て求めよ
授業で難問の宿題として出されたんですけど検討つかないです
0393学術垢版2018/09/28(金) 19:55:55.65ID:o765lpmk
計算量の多い方がそろばんの伝統や中国の人口数近いんだろうな。
回り道もいいかもしれない。早く解くのはバランスが悪い時が多い。
0394132人目の素数さん垢版2018/09/28(金) 20:38:28.52ID:PQc32ans
V を線形空間
U1, U2, U3 を V の部分空間

とする。

U1 ∪ U2 ∪ U3 が V の部分空間になるための必要十分条件は、

U1, U2, U3 のどれか1つが他の2つを含むことである

ことを証明せよ。

但し、 V は {0, 1} 上のベクトル空間ではないとする。
0395132人目の素数さん垢版2018/09/28(金) 21:05:21.52ID:ZS4vyl6B
>>388
a[n] = a[n-1] + {1/(2n-1)(2n-3)} a[n-2],が成立する証明式はありますか?

それとも、こうなるであろうという演繹ですか?
0396132人目の素数さん垢版2018/09/28(金) 23:28:22.51ID:phrHQfEJ
>>395

c[n] = (2n-1)!!・a[n] について漸化式
 c[n] = (2n-1)c[n-1] + c[n-2]
が成り立つ理由が [前スレ.623] に示されています。

これから a[n] の漸化式を求めると、その式になります。

 (2n-1)!! = 1・3・5…(2n-1)
0398132人目の素数さん垢版2018/09/28(金) 23:52:42.11ID:b1hXYTTV
>>395
横レス。
それは証明できるよ。
条件をみたすカップルの並び方の数をA[n]とする。
A[n]に属する列のうち
一番先頭の相方が別のカップルに挟まれていない場合の数が 2n(2n-2)A[n-1] 通り。
一番先頭の相方が別のカップルに挟まれていて3番めの場合(ABab…の形)の数が 2nA[n-1] 通り。
一番先頭の相方が別のカップルに挟まれていて3番めでない場合(A…Bab…の形)の数が 2n(2n-2)A[n-1] 通り。
∴ A[n] = 2n(2n-1)A[n-1] + 2n(2n-2)A[n-2]。
両辺を2n!で割って
a[n] = a[n-1] + 1/((2n-1)(2n-3))a[n-2]。
0399132人目の素数さん垢版2018/09/29(土) 00:25:03.47ID:vaCW7X53
>>392
>>392
Zsigmondyの定理を使えばできた。
http://integers.hatenablog.com/entry/2016/12/30/183841
ーー
p^m=m^n-1
m=2のとき。
pは奇素数である。
よってこのときp^m ≡ 1 (mod 4)により2^n-1≡1(mod 4)。
∴ n=1であるが p^2 = 1 となり解無し。
(m,n) ≠ (2,6) かつ n≠2 かつ m≠2 のとき。
Zsigmondyの定理よりm^n-1はm-1と互いに素である素因子をもつ。
しかしm^n-1、m-1の素因子はpしかありえない。
∴ m-1=1。∴ m=2。∴ 解無し。
(m,n) = (2,6)のとき。
p^2 = 63 より解無し。
n=2 かつ m≠2 のとき。
このときp^m = (m+1)(m-1)。
このときm+1,m-1はいずれも1でなく最大公約数は1または2。
しかし互いに素だと右辺が素因子を2つ以上持つことになり矛盾。
∴ (m+1,m-1) = 2。
∴ p = 2。
よってm+1、m-1はともに2べきで差が2だからm = 3。
∴ (p,m,n) = (2,3,2)。
0400132人目の素数さん垢版2018/09/29(土) 00:32:10.83ID:RVJSlbLo
需要関数に線形モデルを仮定した時の需要の価格弾力性係数(E)を求めなさい。更に需要の価格弾力性係数と価格の関係を説明しなさい。

ただし、線形モデルは以下のものとする。ただし、y を需要、x を価格、α、βはパラメータとする。
yi=α+βxi
0401132人目の素数さん垢版2018/09/29(土) 01:11:18.21ID:7jO6lw+J
なんで経済の人って、経済の問題を数学板で質問するんですかね

他の分野の人はそんなことしませんよ
0403132人目の素数さん垢版2018/09/29(土) 04:53:36.16ID:u/jq2Qwz
サーバーエンジニアと医師はどっちの方が頭が良いですか?
0404132人目の素数さん垢版2018/09/29(土) 06:27:50.01ID:DjGEpWd+
名古屋大学のアゴラにあった問題なのですが,

証明したい事柄:
「nを2以上の自然数とする.
1,2,…,2nの2n個の自然数から,
n+1個の自然数をとると,
そのうちの2つについて,
一方が他方の倍数になっているものが存在する.」

次のような解答で合っていますか.
教えてください.
よろしくお願いします.

「数学的帰納法」と「引き出し論法」を使いました.

[basis]
n=2のとき,
{1,2,4},{3}の2組に分けると,
3個とれば,{1,2,4}の中から2個はとることになるので
成り立つ.
n=3のとき,
{1,2,4},{3}の2組に対して,
6は,{3}に入れて{3,6}とし,
5は{5}とする.
{1,2,4},{3,6},{5}の3組に分けることができる.
4個とれば,{1,2,4},{3,6}の少なくともどちらからは2個とるので
成り立つ.
n=4のとき,
{1,2,4,8},{3,6},{5},{7}の4組に分けることができる.
5個とれば,成り立つ.

[induction step]
n=k(k≧2)で成り立つと仮定する:
1,2,…,2kの2k個の自然数が,
n=2,3,4のように,
{1,2,4,…},{3,6,…},{5,10,…},…という具合に,
k個の組に分けることができると仮定する.
(ここから,k+1個を選べば成り立つことがわかる.)
このとき,2k+1については,{2k+1}として,1組作り,
2(k+1)については,k+1の属している組に入れれば,
n=k+1のときも,k+1個の組に分けることができる.
(したがって,ここからk+2個をとれば成り立つことがわかる)

以上から,証明したい事柄は,証明された.□□

よろしくお願いします.
0405132人目の素数さん垢版2018/09/29(土) 07:06:28.20ID:RzsrefTj
この問題が分からないので教えてください。お願いします。

相対無=自分以外の何かが無いこと。
絶対無=全てが無いこと。

・無というのは無いことなので、当たり前だが存在しない。
・つまりあるのは有だけというか有が全てになる。
・それを無と呼ぶ。
・そして、有の全てを「全」と呼ぶ。
・全は無限つまり永続性があるものなので、完全消滅は不可能。
・完全消滅できるのは有限なモノだけ。

例えばリンゴが目の前にあったとして、それを完全消滅させたらどう解釈することになるのか?
相対無になるのだろうか?そもそもそういったものを無と呼んで良いのだろうか?
仮にこれを無と呼んで良いのなら、これをリンゴという有限のものに限定しないで、
全に置き換えてみよう。しかし、全は無限つまり永続性のあるものなので完全消滅はできない。
しかし、一番最初の方に絶対無という概念を書いた。
絶対無とは全てが無いこと。
じゃあ、この絶対無という考え方が間違っているということなのだろうか?
相対無はどうだろう?
相対無というのは自分以外の何かが無いことなので、
一見この概念なら正しそうな気もするが、
例えばさっきの例のリンゴに関して言うと、
目の前にあるリンゴを完全消滅させたら、これをどう解釈するのかが無に対する考え方が異なるため難しくなる。
目の前にあるリンゴを完全消滅させて、それを相対無と呼ぶのなら、
>・無というのは無いことなので、当たり前だが存在しない。
この考え方がおかしくなるのだが、そうすると、目の前にあるリンゴを完全消滅させた場合、
それをどう解釈するのかが分からなくなってくる。
>・無というのは無いことなので、当たり前だが存在しない。
これを継承して、且つ無と言うのは相対的な無だけつまり相対無だけがあり得るとし、
絶対無というのはあり得ないとするか、
そもそも、
>・無というのは無いことなので、当たり前だが存在しない。
これ自体が絶対無で、現在あるものが無になることを相対無と呼ぶのかなど、
いろいろ考えられるが、今現在はまだはっきりしていない。
0406132人目の素数さん垢版2018/09/29(土) 07:27:02.16ID:8eQPc9R7
>>404
だめ。
>n=k(k≧2)で成り立つと仮定する:
と書いたらこれは
1,2,…,2kの2k個の自然数から,
k+1個の自然数をとると,
そのうちの2つについて,
一方が他方の倍数になっているものが存在する.
と仮定する。
の意味にしかならない。
>{1,2,4,…},{3,6,…},{5,10,…},…という具合に,
>k個の組に分けることができると仮定する.
の意味にはならない。
そもそも
>n=2,3,4のように,
こんな記述は通用しない。
どのようにわけたのか?なぜそのように分けたらうまくいくのかを明示しないと駄目。
0407132人目の素数さん垢版2018/09/29(土) 08:40:59.71ID:TipkCLLM
2k+1と2k+2という数を加えるとき、{2k+1}という新しいグループを作る一方、2k+2は、{k+1}の
グループに入れることができ、グループは一つしか増えないことをきちんと説明しているから、
数学的帰納法を使った証明として、成立していると思うがね。

要は、1〜2nの自然数を、2^k*(2m-1) の形で表したとき、m は、n 通りで十分ということ。
これに触れれば、数学的帰納法等使わず、説明できる。
0408132人目の素数さん垢版2018/09/29(土) 10:48:38.41ID:qskZCtdd
>>404
面白い証明ですね。正しいと思います。

自然数は必ず{奇数x2^(k-1) (kは自然数)}の形に書けるので、
これで2n以下の自然数を分類すればn個の組み分けになるという
ことですね(帰納法で証明するのは簡単)。
0409132人目の素数さん垢版2018/09/29(土) 11:03:17.91ID:qskZCtdd
>>407
被りましたね。すみません。
0410132人目の素数さん垢版2018/09/29(土) 11:52:59.74ID:G2jS7PMy
与えられた整数nが、ある自然数kとmを用いて
n=2^k+3^m+m+k
の形で表せるとき、nはどのような整数でなければならないか。
0411132人目の素数さん垢版2018/09/29(土) 12:33:49.63ID:7rwNoxs+
>>406
まるで誤答おじさんみたいなレスだが

> >n=k(k≧2)で成り立つと仮定する:
>と書いたらこれは

最後のコロンは、すなわちの意味で使われてるから問題ない

>どのようにわけたのか?なぜそのように分けたらうまくいくのかを明示しないと駄目。

上に例示されているし問題無いし
数学的帰納法の初期値において
なぜうまく行くかなんて理由付けは全く必要ない

頭が悪すぎなんでは
0414132人目の素数さん垢版2018/09/29(土) 15:35:22.63ID:Twhf0ZOK
>>404
>証明したい事柄:
>「nを2以上の自然数とする.
>1,2,…,2nの2n個の自然数から,
>n+1個の自然数をとると,
>そのうちの2つについて,
>一方が他方の倍数になっているものが存在する.」
の「そのうちの2つについて」とは、「取った n+1 個の自然数の中の2つについて」のことだろう。
2=2・1 は1の倍数で、1と2を含む n+1 個の自然数を選べば
条件を満たすように構成的に存在性を証明出来るから、証明したい命題は
「nを2以上の自然数とする.」は「nを1以上の自然数とする.」と一般化出来る。
0415132人目の素数さん垢版2018/09/29(土) 15:58:22.13ID:5t2MTazF
>>407 >>408

2n以下の奇数が { 2m-1 | m=1,2,…,n } のn個であることは自明ですね。


{1,2,…,3n} の中の数を、3で割れるだけ割れば、3n以下の「3で割り切れない数」になる。
3n以下の「3で割り切れない数」は2n個あるから、2n類に分類される。
2n+1個の自然数をとると、少なくとも2つは同じ類に含まれる。(←鳩ノ巣原理)
このとき、一方が他方の3ベキ倍になっている。
0416132人目の素数さん垢版2018/09/29(土) 19:24:59.39ID:DjGEpWd+
404です.
407さん,408さん,411さん,ありがとうございます.
414さん,415さん,示唆を頂きありがとうございます.
雲が晴れました.
0417132人目の素数さん垢版2018/09/29(土) 20:40:39.88ID:T4zEucpS
滑らかな多様体Mから実数直線Rへの滑らかな関数fがあるとき、{x∈M ; f(x)<a} (a∈R)はMの部分多様体になりますか?
なるならどのように考えればいいか教えてください。
0418132人目の素数さん垢版2018/09/29(土) 20:44:55.73ID:uT1RU4nf
開部分集合だからなりそうな希ガス
0419132人目の素数さん垢版2018/09/29(土) 22:19:13.91ID:BrcVBHe2
>>412
何がダメなのでしょうか?
0421132人目の素数さん垢版2018/09/29(土) 23:18:35.04ID:sReFGpyG
■■■■■■■■■■■
■□□□□□□□□□■
■□■■■■■■■□■
■□■□□□□□■□■
■□■□■■■□■□■
■□■□■□□□■□■
■□■□■■■■■□■
■□■□□□□□□□■
■□■■■■■■■■■

■■■■■■
□□□□□■
□■■■□■
□■□□□■
□■■■■■
0422132人目の素数さん垢版2018/09/30(日) 03:38:13.59ID:1xQJjky/
>>418
ありがとうございます
aが正則値のとき{x∈M ; f(x)≦a}が境界付きの滑らかな多様体になることはどのように言えるでしょうか?
f^-1(a)がMの部分多様体になることは分かるのですが...
0424132人目の素数さん垢版2018/09/30(日) 11:41:13.70ID:kQna5dy5
nを正の整数とするとき、n(n+1)(2n+1)は6の倍数であることを証明せよ。

↑これ教えてください
0425132人目の素数さん垢版2018/09/30(日) 11:42:02.21ID:kQna5dy5
nを正の整数とするとき、n(n+1)(2n+1)は6の倍数であることを証明せよ。

↑これ教えてください
0428132人目の素数さん垢版2018/09/30(日) 12:25:45.46ID:sTxrQmd0
2n+1=(n-1)+(n+2)
0430132人目の素数さん垢版2018/09/30(日) 13:29:52.79ID:60e7kxgM
>>427

n(n+1)(2n+1) = Σ[k=1, n] {k(k+1)(2k+1) - (k-1)k(2k-1)}
 = Σ[k=1, n] k{(k+1)(2k+1) - (k-1)(2k-1)}
 = 6Σ[k=1, n] k^2
 = 6 (1^2 + 2^2 + …… + n^2),
0432132人目の素数さん垢版2018/09/30(日) 15:13:38.20ID:DJsf8lH+
ある本の複素数の部分で
|α|〜|β|≦|α±β|≦|α|+|β|
と書いてあるのだが、この用法で「〜」とはどういう意味?
0436132人目の素数さん垢版2018/09/30(日) 16:55:34.61ID:QXkD3Yad
n=9まで一致する式ができた

   7{589n^7−76252n^6+1473418n^5−12519640n^4+55110541n^3−127896988n^2
   +150467292n+66825×2^(n+7)−83666160}−{(n^2−9n)^4+60(n^2−9n)^3
   +1308(n^2−9n)^2+12176(n^2−9n)+40320}

q=―――――――――――――――――――――――――――――――――――――
   495{34286n^5−25n^7−1316n^6−317240n^4+1446935n^3−3416084n^2
   +4304724n+5040{2^(n+6)−551}}+{(589545/128)(n^8−36n^7+546n^6
   −4536n^5+22449n^4−67284n^3+118124n^2−109584n+40320)}

この関数を検算してくれ〜(・ω・)ノ
0437132人目の素数さん垢版2018/09/30(日) 18:36:00.67ID:092iedVI
>>436
絶対間違ってるし邪魔だからもうやめて
そのアプローチで正解でないっていつ気づくの?
0438132人目の素数さん垢版2018/09/30(日) 20:42:11.32ID:60e7kxgM
>>82 のヒント

〔補題〕
 (n^n)/n! ≦ e^(n-1),

(略証)
(1 +1/j)^j = Σ[L=1, j] C[j, L](1/j)^L = Σ[L=1, j] (1-1/j)(1-2/j)…(1-(L-1)/j)/L!
はjについて単調増加。
∴ {(j+1)/j}^j = (1 + 1/j)^j < e,
j=1,…,n-1 を入れて掛けると
 (n^n)/n! ≦ e^(n-1),

(別法)
マクローリン展開から
 e^x > x^{n-1} /(n-1)! + (x^n)/n! + x^{n+1} /(n+1)! + x^{n+2} /(n+2)!
   = (x^n)/n! {n/x + 1 + x/(n+1) + xx/(n+1)(n+2)},
 e^n > (n^n)/n! {2 + n/(n+1) + nn/(n+1)(n+2)} > (n^n)/n! e,   (n≧2)
∴ e^(n-1) > (n^n)/n!,
 n=1 は直接確かめる。   (終)

不等式スレ9-724
0442132人目の素数さん垢版2018/09/30(日) 23:21:24.03ID:TIqo4Krx
>>441
当然できるし
5443827829522773148812913954810360866828706145317982945705254293391295458292023589605615870185673878007736004782284270451993721349385643643132361467286011701708486202105261498599716
/14835085087653253718972529896308389386983938057985425384853569746252839606857062625405021609091862498949562417985042968819817371813012648154614367517235455765561610758304595947265625

閉じた形のものだったら、前スレ https://rio2016.5ch.net/test/read.cgi/math/1534342085/609
> a[n] = {1/(2n-1)!!}i[I_{3/2}(-1)・K_{n+1/2}(1) - K_{3/2}(1)・I_{n+1/2}(-1) ]
> ここに I_m(z), K_m(z) は変形ベッセル函数。
があるだろ
0444132人目の素数さん垢版2018/09/30(日) 23:54:54.50ID:eo+flm29
数Uの問題です。(1)の外心と(2)を教えていただきたいですm(__)m

aは正の実数とする。点A(1,a)、B(-1,a)、O(0,0)がある。

(1)△OABの重心の座標と外心の座標をそれぞれ求めよ。

重心の座標は (0、2a/3)とでました

外心の座標は、それぞれ三点を x^+y^+lx+my+n=0に代入して解こうと思ったのですが

最後
a^+ma=-1
a^+ma=-1
とまったく同じ式がでてきてしまいうまく出せませんでした。

(2)重心と外心が一致するときのaの値を求めよ
0445132人目の素数さん垢版2018/10/01(月) 00:06:38.74ID:ncGHhicg
>>444
図描けよ
外心も x 軸上にあるから x 座標を文字でおいて各頂点までの距離^2 を立式すれば方程式ができる
0447132人目の素数さん垢版2018/10/01(月) 00:09:25.09ID:HKRS9tcF
>>444
円の方程式を持ち出しての計算にするなら
外心は、y軸上にあるから、外心の座標を(0,r)とおいて式を立てれば楽なんじゃない?
x^2+(y-r)^2 = r^2
(代入した後に整理ミスしているだけだと思うけど…Lどこ行ったんだよw)

図形的に考えても面倒じゃないと思う。

ダブってるけど、書いたからそのまま投稿するw
0448132人目の素数さん垢版2018/10/01(月) 00:38:24.87ID:eM2YcEDk
>>438

〔補題'〕
 (n^n)/n! ≦ e^(n-1) ≦ (n^n)/(n-1)!

(略証)
(1 -1/kk)^k > 1 -1/k,   … AM-GM
(1 +1/k)^k = (1 -1/kk)^k /(1 -1/k)^k > 1/(1 -1/k)^(k-1) = {1 +1/(k-1)}^(k-1),
∴ (1 +1/k)^k = {(k+1)/k}^k は単調増加
∴ {(k+1)/k}^k < e,
k=1,2,…,n-1 を入れて掛けると
 (n^n)/n! ≦ e^(n-1),

{kk/(kk-1)}^k > (1 +1/kk)^k > (1 +1/k),   … AM-GM
∴ {k/(k-1)}^k = {kk/(kk-1)}^k・(1 +1/k)^k > (1+1/k)^(k+1)
∴ (1 +1/k)^(k+1) = {(k+1)/k}^(k+1) は単調減少
∴ {(k+1)/k}^(k+1) > e,
k=1,2,…,n-1 を入れて掛けると
 (n^n)/(n-1)! ≧ e^(n-1),
0449132人目の素数さん垢版2018/10/01(月) 03:34:44.35ID:/kB4AWKy
教えてほしいことがあります。

ド底辺高校卒の高卒でしかもブランクが何年もある人間がアメリカやイギリスの名門大学に入る方法ってありますか?
やっぱり無いですか?
本当は日本国内の一流大学に入りたいと思っていたのですが、
日本はやっぱりどうやら18歳で入学する人が圧倒的に多いということで、
歳をとってから大学に入ることについて否定的な見方をする人がかなり多いので、
厳しいかなと思いました。
そこで、ド底辺高校卒でしかもブランクがかなりある人間が、
米英の名門大に入れる方法は無いかと思ったのですが、やっぱり無いですよね?
0454132人目の素数さん垢版2018/10/01(月) 08:19:40.14ID:u9b4EZVw
>>452
真面目に教えてください。お願いします。
0458132人目の素数さん垢版2018/10/01(月) 14:40:28.59ID:JNMd+HEC
見栄をはってチャート式の二項定理の問題を聞いたら回答が来たけど、それがわからなかった(大爆笑)
0459132人目の素数さん垢版2018/10/01(月) 16:03:19.61ID:uQ+IEVvw
線形計画法の本では、なぜタブローなどという分かりにくいものを使うんですか?

コンピューターで計算する時代にはタブローなど意味ないですよね。

連立一次方程式をそのまま書いた方が分かりやすいですよね。
0460132人目の素数さん垢版2018/10/01(月) 18:02:33.00ID:WGyB9cPW
暗算や筆算の計算ミスが多すぎて、数学物理化学全部やばいのですが、どうしたらいいですか?
成績がそれほど悪いわけではないのですが(前回の全国模試で数学は上位1%くらいでした)、

例えば16/3を計算しようとして、パッと8.33333・・・・と暗算してしまったり
割り算で13000-10624を計算して、繰り下がりを1376としてしまったりというようなミスが頻発します

本番でこれをやったらと思うとノイローゼで死にそうで、特に化学の多ケタの割り算は高確率でつまずくのですが
どうすれば改善しますか?
0461132人目の素数さん垢版2018/10/01(月) 19:30:09.73ID:lSP8i6OA
f(x)=(x+1)(x-1)(ax+b)が-1≦x≦1の範囲で極大値と極小値をとるとき、実数aとbの条件を求めよ。
0463132人目の素数さん垢版2018/10/01(月) 21:49:36.14ID:9/hS0X0z
∫(1-4x^2)’(1-4x^2)^(-1/2)dx = 2*(1-2x^2)^(1/2) + C

これの式変形がわかりません。どなたか教えていただきませんか?
0464132人目の素数さん垢版2018/10/01(月) 22:42:26.10ID:NFGqB/Wz
n{2^n+2^(n−1)}/{n{2^(n+2)+2^(n−1)}}という式に

n=0を入力すると1/3が出力されるのはなぜですか?
0467132人目の素数さん垢版2018/10/02(火) 00:56:44.94ID:VNedEoPb
>>451
3. 点zを原点を中心としてπ/2だけ回転した点を表わす複素数をαとする。
  → iz = α,  (反時計回りとする)
 原点が点2+3iに移るような平行移動で、点αが点zに移る。
  → α + (2+3i) = z,
 辺々たすと iz + (2+3i) = z,
 ∴ z = (2+3i)/(1-i) = (2+3i)(1+i)/2 = (-1+5i)/2,

>>459
 計算機のない時代の遺物。統計学で層別計算してたのも同じ。

>>460
 もちつけ、兄者。

>>461
 f(x) は極値を2つ以上もつから3次以上。a≠0

 ロルの定理から、2つの根の間に極大 / 極小がある。
 g(x) = ax+b = 0 の根が -1≦x≦1 にあればよい。
 0 ≧ g(-1)g(1) = bb-aa,
 あるいは | -b/a | ≦ 1,

 以上より、|a|≧|b|, a≠0.

>>463
 置換積分でググれ

>>464
 前処理ソフトが約分して呉れたんぢゃね?

>>465
0 = n(n+1)(n+2) -120 = (n-4)(nn+7n+30),
nn+7n+30 = (n+7/2)^2 + 71/4 > 0,
∴ n-4 = 0,
0468132人目の素数さん垢版2018/10/02(火) 01:29:53.84ID:xOs+qnbe
n=0,αn/βn,α={2^n+2^(n−1)},β={2^(n+2)+2^(n−1)}

分母と分子の両方にゼロ掛けているのに
なんで1/3が出力されるねん?(´・ω・`)
0469132人目の素数さん垢版2018/10/02(火) 03:16:51.81ID:ee+PvINm
AB = 2 を直径とする半円の弧の部分に2点C,Dがあり以下を満たしている。
 (i) △ACDは二等辺三角形である
 (ii) △ABCと△ACDの内接円の半径は等しい
このとき,△ABCの内接円の半径を求めよ。

お願いします。
0470132人目の素数さん垢版2018/10/02(火) 07:59:35.48ID:ortyAoQt
xy平面の単位円上に正五角形ABCDEがある。ただし点Aの座標は(1,0)であり、各頂点はこの順に反時計回りに並んでいる。
線分AC上の点Pで、∠DPEが最大になるものを考える。

(1)Pの座標を求めよ。
(2)線分の長さの積PB・PD・PEを求めよ。
0471132人目の素数さん垢版2018/10/02(火) 08:16:34.30ID:vOLg0Hxo
初歩的な質問ですが、
定積分の証明で
S(t)=F(t)+C
というのがでてきますが、
Cにはすべての数が入りうるのに
Cが−F(a)ときまっているのは
なぜですか?
F(a)が変数だからだとしても
納得いきません。
そもそもCって
なにものですか?
0472132人目の素数さん垢版2018/10/02(火) 08:17:04.11ID:VNedEoPb
>>469

(ア) A-D-C-B の順に並ぶとき
 AD < AC, DC < AC より AD=DC,
 ∠ACD = ∠DAC = θ < 45゚, AC = 2sin(2θ),
 △ACDの内接円の半径 r1 = sin(2θ)tan(θ/2) = 2sinθcosθtan(θ/2) = 2(1-cosθ)cosθ
 ∠ABC = ∠ABD + ∠DBC = ∠ACD + ∠DAC = 2θ, ∠ACB = 90゚, ∠BAC = 90゚-2θ,
 AC = 2sin(2θ), BC = 2cos(2θ),
 僊BC = (1/2)AC・BC = sin(4θ),
 僊BCの内接円の半径 r2 = 2僊BC/(AB+BC+CA) = sin(4θ)/{1+cos(2θ)+sin(2θ)},
 r1 / r2 = 1 とおくと sin(3θ/2) = cosθcos(θ/2),
 θ = 34.5626526262゚
 r = 0.290687304
 僊BC = 0.6658737165
 AC = 1.8687238802   BC = 0.7126507276   AB+BC+CA = 4.5813746078

(イ) A-C-D-B の順に並ぶとき
 AC < AD, CD <AD より AC=CD,
 ∠ADC = ∠CAD = θ < 45゚, AD = 2sin(2θ),
 △ACDの内接円の半径 r1 = sin(2θ)tan(θ/2) = 2sinθcosθtan(θ/2) = 2(1-cosθ)cosθ
 ∠ABC = ∠ADC = θ, ∠ACB = 90゚, ∠BAC = 90゚-θ,
 AC = 2sinθ, BC = 2cosθ,
 僊BC = (1/2)AC・BC = 2sinθcosθ,
 僊BCの内接円の半径 r2 = 2僊BC/(AB+BC+CA) = 2sinθcosθ/(1+cosθ+sinθ),
 r1 / r2 = (1-cosθ)(1+cosθ+sinθ)/sinθ = sinθ + (1-cosθ),
 r1 / r2 = 1 とおくと sinθ-cosθ = 0, θ = 45゚, r = √2 -1,
 このとき D=B, 僊BC = 僊CD である。
0476132人目の素数さん垢版2018/10/02(火) 13:46:59.41ID:mtlgLTzy
立方体ABCD-EFGHがあり辺CD、GH上にそれぞれM,Nを
|↑AM|+|↑MN|+|↑MF|の値が最小となるうにとる。
↑AB=↑a , ↑AD=↑b ↑AE=↑cとするとき次のベクトルを↑a , b, cを
用いて表わせ。
(1)三角形FMNの重心をPとするとき↑AP
(2)EからFMNに垂線EQを下ろす。このとき↑AQ

(1)は展開図を考えわかりました。↑AP=2/3 (↑a+↑b+↑c)
(2)がわからないのでお願いします (1)を利用するのでしょうか?
答えは8/9 ↑a +3/9 ↑b+7/9 ↑c らしいのですが解き方がわかりません
0477132人目の素数さん垢版2018/10/02(火) 14:19:00.53ID:0t8uq4AS
APを使えばAM,ANベクトルはすぐ求まって、FM、FNも求まるから
FQベク=sFMベク + tFNベクと置いて
EQベク⊥FMNだから、
EQ⊥FM、EQ⊥FNででいけるんじゃないの?

多分傍用にも類題があると思う
0479132人目の素数さん垢版2018/10/02(火) 14:59:17.90ID:zLpsNvIM
>>477
やっぱりそうやるしかないですか… 結構計算が面倒そうなので
なんか簡単に解く方法があるのかとも思ったのですが
0480132人目の素数さん垢版2018/10/02(火) 15:01:38.34ID:zLpsNvIM
>>478
答えは100%あってます。 答えしか本にのってないのです
0482132人目の素数さん垢版2018/10/02(火) 15:06:55.51ID:zLpsNvIM
>>481
ある大学の過去問なんです。答えおかしいですか?
0483132人目の素数さん垢版2018/10/02(火) 15:10:59.77ID:++Pj2SEU
EQ = 8/9 a + 3/9b - 2/9c
になるけどこれ
FM = -1/3a + b、NM = -1/3a-c
に直交してない希ガス。
0484132人目の素数さん垢版2018/10/02(火) 15:14:59.80ID:0t8uq4AS
>>479
計算は下手にバラバラにせずにまとめたままで計算すればそれほどでもないと思う
けど、平面の方程式が得意なら、そっちつかったほうが楽かな。
0485132人目の素数さん垢版2018/10/02(火) 15:44:47.76ID:ortyAoQt
xy平面上の2点A(1,0),B(0,1)を直径とする円のy>0の部分をCとする。
C上に異なる2点P(cosα,sinα),Q(cosβ,sinβ)を固定する。AB上を動く点Rとの距離の和PR+RQを最小にしたい。

(1)この時のRの座標をαとβで表せ。
(2)RはPR+RQを最小にする位置にある。α<βとする。AP+PR+RQ+QBをαとβで表せ。
0486476垢版2018/10/02(火) 16:42:01.01ID:zLpsNvIM
すいません 476の問題ですがどうしても計算が合いません。
↑FQ=s↑FM+t↑FNとおいて
↑FQ=s(−2/3 a +b-c )+t(-1/3 a +b)
↑EQ=(1−2/3 s−1/3 t)a+(s+t)b-sc
↑EQ・FM=0 より22s+11t−6=0
↑EQ・FN=0 より11s+10t−3=0  連立してt=0 s=3/11となってしまうのですが
どこで間違えたのでしょうか?
0487132人目の素数さん垢版2018/10/02(火) 18:19:45.39ID:zLpsNvIM
失礼 486 解決したので無視して下さい
0489132人目の素数さん垢版2018/10/02(火) 23:01:08.54ID:WVFRN6vC
>>488
楕円上の点(x,y)は(x-αy, βx +(√3)γy) に移るので
(x-αy)^2 + {βx +(√3)γy}^2 = 1

(1+β^2)x^2 +(α^2 +3γ^2) y^2 -2{α -(√3)βγ} xy = 1
楕円の式と比べて
β^2 = 2
α^2 + 3γ^2 = 9
α = (√3)βγ

したがって
β = √2
α = (√6) γ = √6
γ = 1
0492132人目の素数さん垢版2018/10/02(火) 23:22:15.10ID:9LiRKrfn
>>489
ありがとうございます
分かりました
0493132人目の素数さん垢版2018/10/02(火) 23:37:33.22ID:0t8uq4AS
いや、なぜ高2で一次変換をやってるのかそこから説明が聞きたいんだが・・・
0495132人目の素数さん垢版2018/10/03(水) 00:12:26.87ID:aSuhJUlr
>>493
高専2年生です
0496132人目の素数さん垢版2018/10/03(水) 00:27:28.41ID:s6MXA51P
【問題】
以下の条件を全て満たす実数xの関数f(x)の具体例を1つ挙げよ。

(A) f(x)は常に正
(B) -∞<x<∞で微分可能
(C) ∫[-∞→∞] f(x) dx は収束する
(D) (C)の積分値をaとおき、また ∫[0→1] f(x) dx = b とおくと、b/a>3/4
(E) f’(0) = -2

【発展】
(1)条件(D)の不等式をb/a>c (1>c>3/4)と置き換えた場合のf(x)の具体例を1つ挙げよ。
(2)条件(E)で f'(0) < -2018 とした場合のf(x)の具体例を1つが挙げよ。
(3)上記(1)(2)を共に満たす場合はどうか。
0498132人目の素数さん垢版2018/10/03(水) 00:32:35.06ID:JYGM9rOO
Any finite topological tree T {belongs to} C with 2 verices at 0 and 1
determines a unique Belyi Plynomial.

の例をしめしてください。
0499132人目の素数さん垢版2018/10/03(水) 00:35:17.71ID:TLYZIUEu
集合論の質問です。

今公理 C を
C : ∀X ∃f : Pow(X)\{∅} → X ∀S ∈ Pow(X)\{∅} f(S) ∈ S
とします。(いわゆる選択公理)
ZF 上ではこれで良いとして BG では
C1 : ∀X : small ∃f : Pow(X)\{∅} → X ∀S ∈ Pow(X)\{∅} f(S) ∈ S
C2 : ∀X ∃f : Pow(X)\{∅} → X ∀S ∈ Pow(X)\{∅} f(S) ∈ S
の2つが考えられると思いますが

1) この2つは同値ですか?それともC2 の方が真に強い公理ですか?
2) BG + C1 の無矛盾性と BG + C2 の無矛盾性が同値である事を証明できますか?
3) 一般に BG 上の選択公理といえばどちらを指しますか?

よろしくお願いします。
0501132人目の素数さん垢版2018/10/03(水) 07:54:55.57ID:7h2ip4rW
>>496

f(x) = b・p(x; σ^2) + (a-b)・q(x; δ)
は (A) (B) (C) を満たす。

p(x; σ^2) = 1/√(2πσ^2) exp{-(x-1/2)^2 /(2σ^2)  … 正規分布}

 σ=0.2 のとき ∫[0, 1] p(x; σ^2) dx ⁼ 0.98758
 σ=0.1 のとき∫[0, 1] p(x; σ^2) dx ⁼ 0.999999

q(x; δ) = 0,          (x≦-3δ)
   = (x+3δ)^2 /(4δ^3)  (-3δ≦x≦-2δ)
   = 1/(2δ) - (x+δ)^2 /(4δ^3)  (-2δ≦x≦0)
   = (x-δ)^2 /(4δ^3)   (0≦x≦δ)
   = 0,             (δ≦x)

 ∫[-3δ, δ] q(x)dx = 1,
 δは
(E)   f '(0) = (a-b)q '(0) = -(a-b)/(4δ^2),
 を満たすように決める。
0502132人目の素数さん垢版2018/10/03(水) 17:35:50.06ID:7h2ip4rW
代数的数の全体がなす体をKとする。

〔Belyiの定理〕
射影直線上 高々3点のみで分岐する被覆によって 全てのK上の非特異完備代数曲線が表わされる。
これをBelyi多項式と云う。

標数0の体上の完備非特異曲線XがK上定義される曲線と同型となる条件は、
P^1 の分岐被覆X→P^1 であって、高々3点(0,1,∞としてよい)のみで分岐するものが存在すること。
これをBelyi関数と云う。

すべてのQの有限次代数拡大は P^1 - {0,1,∞} の基本群への作用から得られる。
0505132人目の素数さん垢版2018/10/04(木) 02:42:20.41ID:Lvh1QYjd
a,bは正の実数とする。
s(x+a) < ∫[0→1] (a+b)/(ax+b) dx < s(x+b)
となるxの一次分数関数s(x)を1つ求めよ。
0506132人目の素数さん垢版2018/10/04(木) 07:47:31.04ID:Lvh1QYjd
一辺の長さが1の立方体OABC-DEFGがある。
また、ACを直径とし底面OABCと垂直に交わる半円周をKとし、K上に点Pがある(Kは立方体の内部にある)。
OからPを経由して頂点Xに至る最短経路の長さをd(P,X)と表す。Pが動くとき、以下を求めよ。

(1)min{d(P,B)}
(2)min{d(P,F)}
(3)min{d(P,E)}
0507132人目の素数さん垢版2018/10/04(木) 10:29:35.11ID:XgUpOSQ3
ABC内の点FからAC上の点Gに垂線を下ろすとき、|FG|の最大値を求めよという問題をベクトルゴリ押しで解こうとしたんですが、|FG|^2=0とかいうありえない計算結果になりましたどこで計算ミスしたのか教えて下さい

https://i.imgur.com/vsEWWZI.jpg
0508132人目の素数さん垢版2018/10/04(木) 10:31:29.06ID:XgUpOSQ3
本来αβのとる範囲には多項式の条件がある問題です。
まずαβ、bcの式でFGを表してから解こうとしたということです
0509132人目の素数さん垢版2018/10/04(木) 11:54:22.32ID:sxpMnp/q
計算チェックまでする気はないけど、FがABC内にあるなら、F=Gになる時が最小になって当然じゃないの?
0511132人目の素数さん垢版2018/10/04(木) 12:06:07.97ID:fAxXilhM
>>507
> ABC内の点FからAC上の点Gに垂線を下ろす
この表現とか6にしか見えないGのほうが気になる
0513132人目の素数さん垢版2018/10/04(木) 12:19:07.11ID:sxpMnp/q
あんまり関係ないけど
この問題で、AGベクトルはAFベクトルの正射影ベクトルだけど
セットになるべきFGベクトルの名前はついているのでしょうか。
3次元なら割と綺麗な式になるから名前付いてそうで、なんか気になる

AGベクトルの単位ベクトルをeとして
FG = (AF×e)×e
AG = (AF・e)e
0514132人目の素数さん垢版2018/10/04(木) 12:47:35.74ID:sxpMnp/q
おまけの別解
上にも書いたように、FGベクトル = ((AFベクトル)×e)×e (但し eはAGベクトルの単位ベクトル)
なので
FG = ((αb+βc)×e)×e = (αb×e)×e だから
|FG| = |αb|
で片付いてスッキリする
0516132人目の素数さん垢版2018/10/04(木) 13:51:05.45ID:sxpMnp/q
sin抜けてるやん…
|FG|=|αbsin(ABとACの角度)|
スッキリしてるけど俺の頭がすっきりしてないらしい
0518132人目の素数さん垢版2018/10/04(木) 16:27:58.78ID:wFWA09/F
>>504

まづ y だけ動かす。
 sin(y) + cos(x+y) = cos(π/2 -y) + cos(x+y)
 = 2cos(π/4 +x/2)cos(π/4 -x/2 -y)
 ≦ 2|cos(π/4 +x/2)|,

次にxを動かして
 f '(x) = cos(x) - sin(x) ± sin(π/4 +x/2) = 0,

 x = 0.204830928474733243276 + 2nπ,
 f(x) = 2.44471599169833602703 (最大)

境界点は
 cos(π/4 +x/2) = 0, x = (1/2 +2n)π, f(x) = 1,
 ゆえ最大でない。
0520132人目の素数さん垢版2018/10/04(木) 22:23:43.58ID:5nChFh8I
>>506
OP = 1 (一定) なので min PX を考える
d(P, B) = 2 (一定)
d(P, F) ≥ 1 + √(2 - √2)
d(P, E) ≥ 1 + √(3/2) - √(1/2)
0521132人目の素数さん垢版2018/10/05(金) 02:23:53.03ID:7iOX1iCn
>>506 >>517

O (0, 0, 0)
A (1, 0, 0)
B (1, 1, 0)
C (0, 1, 0)
D (0, 0, 1)
E (1, 0, 1)
F (1, 1, 1)
G (0, 1, 1)
P ( (1+cosθ)/2, (1-cosθ)/2, sinθ/√2)     0≦θ≦π,
とおく。

(1) PB = OP = 1,

(2) PF = √{2-(√2)sinθ} ≧ √(2-√2),

(3) PE = √{2-cosθ-(√2)sinθ} = √{2-(√3)sin(θ-a)} ≧ √(2-√3) = (√3 -1)/√2,
0522132人目の素数さん垢版2018/10/05(金) 02:30:55.82ID:7iOX1iCn
>>473

パスカルの漸化式
 C(n,r) = C(n-1,r) + C(n-1,r-1)  (1≦r≦n-1)
 C(n,0) = C(n,n) = 1,
と数学的帰納法を使えば出る。
0524132人目の素数さん垢版2018/10/05(金) 17:05:04.83ID:+MTpncFe
複素平面上の相異なる2点A(α),B(β)を通る直線に原点から下ろした垂線の足をH(γ)とおく。
A,BがO(2)を中心とする円|z-2|=1上を動くとき、△OABの重心をG(δ)とする。
線分GHが通過する領域の面積を求めよ。
0525132人目の素数さん垢版2018/10/05(金) 19:18:16.00ID:+MTpncFe
どの桁の数字も0または1または2である自然数の全体からなる集合をSとする。
このとき以下の命題の真偽を述べよ。
「任意の自然数nに対して、Sの要素のうちnの倍数であるものが存在する。」
0529132人目の素数さん垢版2018/10/06(土) 00:12:34.84ID:AFWx1g8T
2x5=10
3x37=111
4x25=100
5x2=10
6x(5x37)=1110
7x1573=11011
8x125=1000
9x123456789=1111111101

からどうにかならんか?
0530132人目の素数さん垢版2018/10/06(土) 02:54:56.00ID:QuzzCzhX
>>525 >>528
 真

T = {0, 1, 11, 111, 1111, …, (10^n -1)/9}
の要素をnで割った剰余は 0 〜 n-1 のいずれか。
#T = n+1 ゆえ、いずれか2つは同じ類に含まれる。 (←鳩ノ巣原理)
その差はnの倍数であり、かつ
 11…1100…00 または 1……1 の形だから Sの要素である。
0532132人目の素数さん垢版2018/10/06(土) 05:20:05.00ID:6rd0x0IU
nを正の整数とする。2数の積
n×123456789
のすべての桁の数字が1となるようなnを考える。

(1)そのようなnを1つ求めよ。

(2)そのようなnは無数に存在するか。
0533132人目の素数さん垢版2018/10/06(土) 05:50:24.78ID:AK1pEjwX
x_{ij}と、添字が二つ付いている変数は、数字で例を作るとどうなります?
\sigma^a_{i=1} \sigma^b_{j=1} x_{ij}
の説明を読んでいてx_{ij}の具体例が浮かばず、式の意味をイメージできず詰まっています。
たとえば、変数に数字を割り当てて、計算例を出してもらえるとわかる気がするのですが、、、
統計学の教科書で、具体例がないまま式だけでて困っています。
0535132人目の素数さん垢版2018/10/06(土) 06:40:56.07ID:AK1pEjwX
>>534
ありがとうございます。
それは、2*2の行列があって、そこに入っている数字で計算するみたいなイメージでOKですか?
行列にしたら下みたいな感じですか?
| |x1.|x2.|
|x.1| 1 | 2 |
|x.2| 3 | 4 |
0536132人目の素数さん垢版2018/10/06(土) 06:43:47.63ID:AK1pEjwX
ちょっと表を訂正します
(x21=3になるように訂正)
| |x.1|x.2|
|x1.| 1 | 2 |
|x2.| 3 | 4 |
0538529垢版2018/10/06(土) 10:42:57.49ID:AFWx1g8T
>>530
おお、あまりにも明快簡単な証明。
恐れ入りました。
0539132人目の素数さん垢版2018/10/06(土) 12:08:38.99ID:AFWx1g8T
>>532
あるんかいな?
あるとすれば無数にあるのはほぼ自明だけど。
nx(10^桁数 +1)としてあらたなnを作っていけば桁数を無限に伸ばせるから。
0541132人目の素数さん垢版2018/10/06(土) 12:15:14.03ID:yKExIr/P
自分自身を含む6つの素因数が順不同で3つ
A+C+E=B+D+FかつB+C+D=E+F+A
となるような組はあるかどうか?
0542132人目の素数さん垢版2018/10/06(土) 12:22:54.73ID:iwFEpJz2
123456789・9と10は互いに素だから
k がφ(123456789・9) の倍数のとき
10^k-1 ≡ 0 (mod 123456789・9)
ただしφはオイラーの関数。
0546132人目の素数さん垢版2018/10/06(土) 12:53:56.70ID:yKExIr/P
>>545
辺の長さの違う組み換え可能な6角形を求めたいのだが、C=Fって事は1辺は同じ数になるってこと?
それとも俺の作った組が間違えてる?
0548132人目の素数さん垢版2018/10/06(土) 13:44:19.53ID:GD5uvEmE
>>547
すぐ上の定理に代入してるだけ
0549132人目の素数さん垢版2018/10/06(土) 14:14:41.96ID:V97Lm1H9
>>546
6元集合Xをいかに3元集合の和A∪B.C∪Dと分けようとも片方の分け方はもう片方の分け方の一個ずつを選んで交換したものにしかならない。
交換して和が不変などあり得ない。
0550132人目の素数さん垢版2018/10/06(土) 14:16:31.10ID:JVbQz5AH
0と代数的数αって一次独立じゃないですよね
てことは代入ってできないと思うのですが、すみません詳しくお願いします、、!
0551132人目の素数さん垢版2018/10/06(土) 14:32:10.15ID:iZgcyYIE
小学生向けの問題で恐縮ですw

みかんを何人かの子供に分けることになりました。
1人に3個ずつ分けると21個あまり、5個ずつ分けると11個足りません。
みかんの個数は全部で何個ですか?

答えしかなく、計算式が載ってない。計算式おねがいします。
ちなみに、答えは69個です。
0552132人目の素数さん垢版2018/10/06(土) 14:32:58.70ID:YIz2WDOP
「すなわち、〜」の前の部分を陽に使うのであれば

代数的数βでe^α=βとなったとします
e^α=β=β*1なので「すなわち」の前の部分からβ=0ですね
一方でe^z=0となる複素数zは存在しませんね
よってe^αは代数的数ではないですね
したがってe^αは超越数ですね
0553132人目の素数さん垢版2018/10/06(土) 14:45:03.56ID:sMR0Hk38
>>551
あと11個あったら5個ずつ分けるとピッタリで3個ずつ分けると32個余ることになる
これは、あと11個あったら3個ずつ分けたあと、さらに余った32個を2個ずつ分けるとピッタリになるわけだから(以下略
0554132人目の素数さん垢版2018/10/06(土) 14:59:14.79ID:YIz2WDOP
>>552
ごめん
変なこと言ってるから訂正

e^α=βとなったとします
これは1*e^α+(-β)*1=0となり、これはe^αと1がalg(Q)上一次従属であることになります
これは「すなわち」の前の部分に矛盾します
したがってe^αは代数的数ではない、すなわち超越数です
0555132人目の素数さん垢版2018/10/06(土) 16:19:47.63ID:lZB6drwe
共同ツール 1
https://seleck.cc/685

https://trello.com/
ボードのメニュー → Power-Upsから拡張可能 Slack DropBoxなど
Trello Chrome拡張機能 elegant
ttp://www.kikakulabo.com/service-eft/
trelloのオープンソースあり

共同ツール 2
https://www.google.com/intl/ja_jp/sheets/about/

共同ツール 3
https://slack.com/intl/ja-jp
https://www.dropbox.com/ja/
https://www.google.com/intl/ja_ALL/drive/
https://getpocket.com/a/queue/
https://gsuite.google.co.jp/intl/ja/products/calendar/
https://bitbucket.org/
https://ja.atlassian.com/software/sourcetree
https://ja.atlassian.com/software/jira/pricing?tab=self-hosted 千円
https://www.sketchapp.com/
ttp://photoshopvip.net/103903
ttps://goodpatch.com/blog/sketch-plugins/

trelloと他のサービスの連携 IFTTT
https://ferret-plus.com/7940
https://chrome.google.com/webstore/search/trello?_category=extensions
0557132人目の素数さん垢版2018/10/06(土) 17:09:15.84ID:JVbQz5AH
>>554
シャワーしてたら同じこと思いつきました!(笑)

丁寧な説明ありがとうございます!
0558132人目の素数さん垢版2018/10/06(土) 19:42:30.88ID:0CbnY1eI
本を読んでいたら
円が一番高い時で1ドル135.2円
円が一番安い時で1ドル87.1円
36%の変動があった
と書かれていました

そもそも変動というものを知らなかったので調べたら2つの方法が載っており
@
87.1÷135.2×100で出るとのことでそしたら64%になってしまいました
100から引くと本に書いてある36にはなりました
A
(87.1-135.2)×100÷135.2
で求められるそうで-35.57…四捨五入して36がでました

@とAで答えが反対になるのはそれぞれどのように考えているからなのでしょうか?
それと調べた時にどちらも変動率ではなく変化率と書いてありました
変動率と変化率の違いもわかりません

もしよろしければ@とAの計算式はどのような考え方で成り立っているのか、変動率や変化率について教えてください
0559132人目の素数さん垢版2018/10/06(土) 20:16:25.91ID:m2GNmx3Y
普段は1000円で売っているものがセールで900円で売られていました
何%の割引だったでしょう?

@ 900円は1000円の90%だから、割り引かれた金額は1000円の10%分である
900÷1000×100=90, 100-90=10

A 割り引かれた金額は100円分で、それは1000円の10%である
(1000-900)÷1000×100=10

の違い
0560132人目の素数さん垢版2018/10/06(土) 20:43:07.28ID:uc+03N+V
>>551
■何人かの子供をx人とする

3x+21=5x−11……A

2x=32

x=16

子供は全部で16人いる

みかんの個数はAにxを代入して

∵3x+21=5x−11=69個.
0563132人目の素数さん垢版2018/10/06(土) 22:58:40.44ID:few7ZUvi
死ね
0564132人目の素数さん垢版2018/10/06(土) 23:15:03.79ID:6rd0x0IU
>>562
アホみてーな何とか算教えるくらいならさっさと方程式教えろっつーの
日本の教育はよお
0565132人目の素数さん垢版2018/10/06(土) 23:21:35.18ID:6rd0x0IU
(問題)
平面上に凸四角形ABCDと動点Pがあるとき、線分長の和L=PA+PB+PC+PDを最小にする点はどこか。

(発展)
kは実数で、先の(問題)のLの最小値以上の値をとる。
A(0,0),B(1,0),C(a,1),D(b,c),とおくとき、
L=kとなる点全体からなる図形を平面上に示せ。
0566132人目の素数さん垢版2018/10/06(土) 23:25:32.04ID:f628einX
なんとか算は後々役に立つ
方程式の未知数の数を直感で一つ減らす能力は後付するのは難しい

ついでになんとか算を習ってる連中は、>>560の方程式位なら解けるし、立式できる生徒も多い
塾によっては>>560の解法がメインのところもあるだろ
0567132人目の素数さん垢版2018/10/07(日) 00:19:47.33ID:E9xbjymX
>>532

(1)
123456789・9 = (3^4)・3607・3803

>>542 により
φ(123456789・9) = φ(3^4)φ(3607)φ(3803)  … 乗法的函数
 = 54・3606・3802
 = 740340648

実際は k = φ(…)/36 = 20565018 でよい。
 10^k - 1 ≡ 0  (mod 123456789・9),
 n = (10^k - 1)/(123456789・9),

(2)
 存在する。
 n = {10^(20565018m) - 1}/(123456789・9),  m∈N
0568132人目の素数さん垢版2018/10/07(日) 00:57:19.16ID:E9xbjymX
〔類題〕
nを正の整数とする。2数の積
 n×12345679
のすべての桁の数字が1となるようなnを考える。

(1)そのようなnを1つ求めよ。

(2)そのようなnは無数に存在するか。
0575132人目の素数さん垢版2018/10/07(日) 12:22:38.63ID:dRzMmBrK
人┏┯┯┯┯┯┯┯┯┯┯┯┯┯┯┯┯┯┯┓ こんなかんじでみかんの数を長方形の面積で考える
数┃                 ┃         ┃ 3個ずつ分けたらBのエリアのみかんがあまり、
 ┃                 ┃  B:21個  ┃ 5個ずつ分けたらCのエリアのみかんが足りない
 ┃                 ┃         ┃ BとCを足せば32(個)、1人当たりのみかんの個数は
 ┃       A         ┣┿┿┿┿┿┿┫ 32÷2=16(個)、3人なら16*3=48(個
 ┃                 ┃         ┃ :Aのエリア)
 ┃                 ┃         ┃ 求めるみかんの数は48+21=69(個)
 ┃                 ┃  C:11個  ┃
 ┃                 ┃         ┃
 ┗┷┷┷┷┷┷┷┷┷┷┷┷┷┷┷┷┷┷┛
 0                 3          5 1人当たりのみかんの個数
0576132人目の素数さん垢版2018/10/07(日) 14:40:25.11ID:evu0+YND
>>551
鶴亀算では、「仮に全てが鶴だとすると脚の数は○○であり、実際の数と△△違うから、...」
という考えで問題と解くのが一般的。これを応用すると...

仮に20人いるとすると、みかんの個数は前半からは 3*20+21=81個、後半からは 5*20-11=89個。ずれが8個
仮に21人いるとすると、みかんの個数は前半からは 3*21+21=84個、後半からは 5*21-11=94個。ずれが10個
一人増やすと、「ずれ」が8個から10個に、2個増えた。
「ずれ」を0にするためには、20人の時から、4人減らせばよい。つまり、子供の数は16人
みかんの数は、前半から 3*16+21=69 であり、後半からも 5*16-11=69 と同じ値が出る。

あえて計算式を書くとすると、3 * {(21-(-11))/(5-3)} + 21
0577132人目の素数さん垢版2018/10/07(日) 17:36:57.12ID:zksXVA/M
過不足算は、ある物を何人かで分配するときに、1人分の数量や分配後の
余りまたは不足などから全体の数量や人数を求める算術です。

全体の差
最初に余り、次にちょうど → 最初の余り
最初に不足、次にちょうど → 最初の不足
最初に余り、次も余る → 余り-余り
最初に不足、次も不足 → 不足-不足
最初に余り、次に不足 → 余り+不足

人数=全体の差÷1人分の数量の差

総数
余る場合 → 1人分の数量×人数+余り
不足する場合 → 1人分の数量×人数-不足
0578132人目の素数さん垢版2018/10/07(日) 18:10:17.17ID:ICgU2uBX
>>569
だよね。
でも、nを求めよって言ってるから、具体的な数値を書けってことかも。
オイラーの関数って初耳だけど、どうやんの?
(存在自体は、おっしゃるように鳩ノ巣なんたらと、10と12…9x9が
互いに素から、10^k-1 ≡0となるk が存在するって初等的に証明できる
んだけど)
0579132人目の素数さん垢版2018/10/07(日) 18:13:56.30ID:ICgU2uBX
すまん、>>567を読んでなかった。
2000万桁の数なんて書き下せんわw
0580132人目の素数さん垢版2018/10/07(日) 18:40:00.87ID:5LTPL5bP
>>436
n=12まで

    {2^n+2^(n−1)+n-4-α/12+643(n-5)α/120
   -2251β/720+501(n-7)β/112+20107a/840
   +80167(n-9)a/90720+1925209b/259200
   +1109375429934433(n-11)b/13305600}
q=―――――――――――――――――――――――――
    {2^(n+2)+2^(n-1)+2n-10-{(n-2)^2(n-4)}
   +607(n-5)α/40-357β/40+10607(n-7)β/840
   +1339a/20+822251(n-9)a/362880+18769033b/907200
   +264154294609541(n-11)b/1140480}

,α=(n-1)(n-2)(n-3)(n-4),β=α(n-5)(n-6)
,a=β(n-7)(n-8),b=a(n-9)(n-10)
0582132人目の素数さん垢版2018/10/07(日) 21:24:58.09ID:vtlFnQU8
      | Hit!
      |
   ぱくっ|
     /V\
    /◎;;;,;,,,,ヽ   そんなエサで
 _ ム::::(,,゚Д゚)::|   俺様が釣られると思ってんのか!!
ヽツ.(ノ:::::::::.:::::.:..|)
  ヾソ:::::::::::::::::.:ノ
   ` ー U'"U'
0583132人目の素数さん垢版2018/10/07(日) 21:28:25.63ID:ICgU2uBX
すべての桁数の数字が1となるような素数で11より大きいものはあるか?
0585132人目の素数さん垢版2018/10/07(日) 23:01:47.43ID:E9xbjymX
>>583

{10^(ab) -1}/9 は (10^a -1)/9 及び (10^b -1)/9 の公倍数。

(10^3 -1)/9 = 3 x 37,
(10^5 -1)/9 = 41 x 271,
(10^7 -1)/9 = 239 x 4649,
(10^11 -1)/9 = 21649 x 513239,
(10^13 -1)/9 = 53 x 79 x 264371653,
(10^17 -1)/9 = 2071723 x 5363222357,
ゆえ、>>584 が最小のもの。
0587132人目の素数さん垢版2018/10/07(日) 23:37:23.37ID:X/c1GjM/
高2 行列
この連立方程式を行列を用いて解いてください
(出来ればクラメルの公式以外でお願いします)
https://i.imgur.com/N2py1ii.jpg
0589132人目の素数さん垢版2018/10/07(日) 23:45:48.39ID:ExsNFjY/
下手に素人がアレコレ考えても専門家の作ったもんにはかなわない。
自分がその専門家を目指すならともかく。
あくまでグレブナー基底のユーザーなら偉い人の作ったやつそのまま使うのが吉。
0591132人目の素数さん垢版2018/10/08(月) 00:26:43.00ID:6Cwpy4cK
>>584,585
流石!

では、すべての桁数が1となる素数が無数にあることを証明せよ。
0593132人目の素数さん垢版2018/10/08(月) 02:52:31.57ID:wsugaKT2
http://www5e.biglobe.ne.jp/~emm386/2015/equation/c04.html
このページの式(5)の2番目以降の解がどのように出て着たのかがよくわかりません
すぐ上のy=ωB1+ω^2C1から計算してみても辿り着けなかったのですが、どのように導出されるのでしょうか?
0594132人目の素数さん垢版2018/10/08(月) 06:30:10.06ID:moWJj/Va
>>587

(3)
ax+y+z = 1,
x+ay+z = a,
x+y+az = aa,

・a=1 のとき、x+y+z = 1 全体。

・a=-2 のとき
 与式を辺々たすと
 (a+2)(x+y+z) = 1+a+aa > 0,
 ∴ 解なし。

・a≠1, a≠-2 のとき
係数行列
 [ a, 1, 1 ]
 [ 1, a, 1 ]
 [ 1, 1, a ]
の行列式=(a-1)(a+2)≠0 で、逆行列が存在する。
 [ a+1, -1, -1 ]
 [ -1, a+1, -1 ] /
 [ -1, -1, a+1 ]

これを右辺に乗じて
 x = -(a+1)/(a+2),
 y = 1/(a+2),
 z = (a+1)^2 /(a+2),
0595132人目の素数さん垢版2018/10/08(月) 06:39:58.55ID:JVgPvsCi
>>593
三倍角の公式に cos(3θ) = 4(cosθ)^3 - 3cosθ 等がありますが、cosθを未知数 x 、cos(3θ)を定数 a と考えれば、
4x^3-3x=a 
となります。どんな三次方程式でも、二次の項は平行移動で消すことができ、
三次の係数と一次の係数の比を4:3になる様に、スケール変換すれば、この形に持って行けます。

|a|≦1なら、cost=aとなるtを持ってくると、cos((t+2πk)/3)、k=0,1,2 が解になります。
0596132人目の素数さん垢版2018/10/08(月) 07:12:10.04ID:m3fUDFm2
>>594
ありがとうございます
0597132人目の素数さん垢版2018/10/08(月) 07:14:29.11ID:moWJj/Va
>>595

|a|≧1 のときは
実数解が
 r = (1/2) { [a+√(aa-1)]^(1/3) + (1/2)[a-√(aa-1)]^(1/3) },
虚数解が
 (1/2) {-r±i√(a/r - rr)},
なんだろうな…
0599132人目の素数さん垢版2018/10/08(月) 07:34:53.07ID:moWJj/Va
>>594 訂正

の行列式=(a-1)^2・(a+2)≠0 で、逆行列が存在する。
 [ a+1, -1, -1 ]
 [ -1, a+1, -1 ] /{(a-1)(a+2)}
 [ -1, -1, a+1 ]
だった。
0600132人目の素数さん垢版2018/10/08(月) 09:12:26.53ID:6Cwpy4cK
>>592
ありがとう!!
おかげさまで無駄に時間をつぶさなくて済んでよかった。

しかし、こんな項目があるのなら、もっと早く紹介して欲しかった。
0601132人目の素数さん垢版2018/10/08(月) 13:21:41.03ID:UjxGSNCg
部分分数分解の要領でやるのと思ったのですが、どうしても導けなかったので手順を教えてください

(x-1) / (3x+2)
が、
1/3 - 5 / (3(3k+2))

なるものです
0602132人目の素数さん垢版2018/10/08(月) 13:34:43.66ID:UjxGSNCg
>>601
あ、k と書きました x と読み替えてください
0605132人目の素数さん垢版2018/10/08(月) 15:33:44.59ID:ClttM/Xa
                           
                                −−
                                (馬^ェ^)    ーー
                               f´     ,.}     (鹿^ェ^ )
                               ,ム ィ´_}._.小. / .`     `ヽ     ーーー
                               Y.ゝ‐´   |. ∨ーfト. __ . 、 廴}|    ( ★^ェ^  )   
                               :| ヽ阪 .ノ!゙1 /:|       ト._リ  ,。-"       ~ヽ
                              .弋._ノ`{:  | 弋リ f、   。  |   /            }
                                    }、.ノ     ! ` 、_ .ノ!   |   {_ .-、      f: メ.
                                  {. リ    ‘.   京__ノ    l  / 三! .  ノ|´ l
                              弋_)      マ リ       マ   ア~    ̄ !、 ‘.
                                            { ー'|       〉r‐'       l! マ 〉
                                          }: {       i |    o    ハ `´
                                           { ヘ         | } 、      ノ !
                                           ̄       l   `::禿   :!
                                                 ゝ==イ `|    ,' 👀
Rock54: Caution(BBR-MD5:1341adc37120578f18dba9451e6c8c3b)
0606132人目の素数さん垢版2018/10/08(月) 18:29:49.39ID:Aq/jFjy9
>>584
Haskellでそれが素数であることを確認してみました。

Prelude Data.List> import Data.List
Prelude Data.List> divisor n = find (\m -> n `mod` m ==0 )[2..floor.sqrt.fromIntegral $ n]
Prelude Data.List> divisor $ (10^19-1) `div` 9
Nothing
0608132人目の素数さん垢版2018/10/08(月) 19:46:59.03ID:m3fUDFm2
>>607
本当に高専2年です
高専の数学問題集2の問題ですが解説抜きで答えだけ書いてあるので解説してもらいたくて載せました
0609132人目の素数さん垢版2018/10/08(月) 19:50:41.93ID:Q/DjdR62
>>604

「第 n 列に沿っての余因子展開し、」

って日本語がおかしくないですか?
0610132人目の素数さん垢版2018/10/08(月) 20:01:04.50ID:Q/DjdR62
>>608

(1) D_n = D_(n-1) + D_(n-2)

(2) D5 = D4 + D3 = D3 + D2 + D3 = 2*D3 + D2 = 2*3 + 2 = 8
0612132人目の素数さん垢版2018/10/08(月) 20:03:48.50ID:UjxGSNCg
>>603
ありがとうございます

3x+2 を x-1 でくくって 5 がでてくるとこまではいけましたが
分数を二つに分けるとこまでは理解できず…
雰囲気は感じることができましたが、僕は数学のセンスは無いんでしょうね…
0613132人目の素数さん垢版2018/10/08(月) 20:05:35.75ID:Q/DjdR62
第 n 列に関して展開すると、

D_n

=

(-1)^[(n-1)+n] * (-1)^[(n-1)+(n-1)] * (-1) * D_(n-2)

+

(-1)^[n + n] * D_(n-1)

=

(-1)^[4*n - 2] * D_(n-2) + (-1)^[2*n] * D_(n-1)

=

D_(n-2) + D_(n-1)
0614132人目の素数さん垢版2018/10/08(月) 20:20:38.35ID:m3fUDFm2
>>613
今自分でもやってみましたが第n列で展開するとdet A_(n-1)-A_(n-1,n)[余因子展開]になり、A_(n-1,n)は-det A_(n-2)+0となりますね。さっきは計算ミスで0にならなくて困ってました(笑)解説ありがとうございます。
0616132人目の素数さん垢版2018/10/08(月) 23:13:41.55ID:moWJj/Va
>>604

〔問題〕

nを2以上の自然数として、n次の正方行列A_n = (a_{i,j}) を次のように定める。

a_{i,j} = 1,   i-j = 0 または -1
   = -1,   i-j = 1
   = 0,   |i-j|≧2

たとえば A_5 = … (ry … である。

(1) D_n = det A_n とする。第n列に沿って余因子展開し、 D_nに関する漸化式を求めよ。
(2) D_5 を求めよ。      (新潟大*, 類:電通大*)

蛇足ですが、
 D_n = F_{n+1}  …… フィボナッチ数
0617132人目の素数さん垢版2018/10/09(火) 01:14:11.74ID:GgPxPPOK
>>616
>>610さんの回答で尽きていますよ。
D_0=1と置くのは乗法の自然な措定。
改めてフィボナッチなどと言及せずとも自明なことなのです。
0618132人目の素数さん垢版2018/10/09(火) 08:13:43.63ID:HEM5WUg1
どの桁も0と1からなり、最高位の数字が1の自然数を考える。

いま数字列100,101,110,111のうち1つを無作為に選び、この自然数の最高位にそれを付け加え、新しく3n+3桁の自然数を作る。
すなわち元の自然数をNとすれば、それに101を付け加えた新しい自然数とは{N+101^(n+2)}である。

初期状態100からこの操作を繰り返し行うとき、n回目の操作で出来た自然数が7の倍数となる確率p[n]を求めよ。
0619132人目の素数さん垢版2018/10/09(火) 10:09:47.56ID:yBLic6yD
>>618
>どの桁も0と1からなり、最高位の数字が1の自然数を考える。
どの桁も0と1なら、最高位の数字は1しかない。

>新しく3n+3桁の自然数を作る。
nが未定義。桁数だとすれば、n+3桁じゃねーの?

>それに101を付け加えた新しい自然数とは{N+101^(n+2)}
N+101*10^nではなくて?

やりなおし。
0621132人目の素数さん垢版2018/10/09(火) 18:41:48.70ID:HEM5WUg1
n,kは自然数、pは素数で、2<n, 0<k<nである。
nCk=p!
となる(n,k,p)の組を全て決定せよ。
0622132人目の素数さん垢版2018/10/09(火) 20:45:12.53ID:xcOAMVL5
確率ってなんですか?確率という値を計算するその体系に矛盾はないし数学分野として成り立っているとは思いますが、それの意味ってなんでしょう
別に600回サイコロ投げたからってそれぞれの目が100回ずつになるわけではないしn回投げたときに1の出た回数をp(n)としたときにp(n)/nの極限が収束するとも言えないわけですから
0623132人目の素数さん垢版2018/10/09(火) 21:46:28.53ID:cJoPTE1+
そもそも確率はギャンブルから生まれたもの
数学が2000年以上前に生まれたものであるのに対し
確率という概念の歴史はわずか300年程度だという事実
0624132人目の素数さん垢版2018/10/09(火) 21:47:19.62ID:ftvdk1wC
>>622
確率をcredibilityと考えた方が現実世界ではすっきりする。
降水確率とか、予報士の確信度の指標。
0625132人目の素数さん垢版2018/10/09(火) 22:33:24.27ID:bCXG4PtT
>>621
import Data.List
divisor n = find (\m -> n `mod` m ==0 )[2..floor.sqrt.fromIntegral $ n]
choose n r = product[1..n] `div` product[1..n-r] `div` product[1..r]
[(n,k,p) | n <- [2..], k <- [1..(n-1)], p <-[2..], divisor p == Nothing, choose n k == product[1..p]]

[(2,1,2)
0627132人目の素数さん垢版2018/10/09(火) 23:40:40.21ID:OI8jFpH4
>>622
>n回投げたときに1の出た回数をp(n)としたときにp(n)/nの極限が収束するとも言えないわけですから

言えますよ
大数の法則と言います
p(n)/nの値を経験的確率といいますが、経験的確率と数学的確率が一致するということですね
0630132人目の素数さん垢版2018/10/10(水) 13:41:42.04ID:pvkW6d0e
https://i.imgur.com/is4mya8.jpg

この問題の(3)の回答がどうしても納得いきません。

y=a+btと置くのですがaとbを求めて
yイコールのxの2次式と連立するのですが何故y=a+btと置くのかが分かりません。

変数も違うし1次式だし

先生に質問したら微分したから次数が下がってると言われましたがxの二次関数なのに微分したらtの一次関数ってのでさらに混乱してしまって分かりません
0631132人目の素数さん垢版2018/10/10(水) 14:02:46.51ID:wEZbtXig
xとtは線形と書いてあるからyを微分してxの一次式になるならtの一次式でも書けるんじゃない?
0633132人目の素数さん垢版2018/10/10(水) 16:03:04.81ID:vEXC+dXU
書き込むところ間違えてしまったのでマルチになりますがすいません
https://i.imgur.com/Yu5U8ny.jpg
この数学的帰納法の右辺を変形するという解説を読んでいますが、一行目から分かりません
なぜこう変わるのか分かりやすく解説して頂けるとありがたいです
0639132人目の素数さん垢版2018/10/10(水) 17:02:52.95ID:vEXC+dXU
>>638
この一行目に間の式がありませんが、いきなりこんな風に出せるものですか?
また、共通因数と見つける事が出来なかったんですが、どう考えたら見つけられますか?
0641132人目の素数さん垢版2018/10/10(水) 17:09:06.58ID:VXF0ffa4
>>636
んじゃ、1行目はわかるだろ
(1/4)(k+1)^2をくくっただけだよ
2行目は中括弧内を展開してまとめた
最後は因数分解
0642132人目の素数さん垢版2018/10/10(水) 17:10:18.05ID:wEZbtXig
>>633
画像一行目の左辺、2つの式の足し算になってるけど、両方(k+1)^2で割れるのはわかる?
両方を(k+1)^2で割って足してるだけだよ

a*b + a*c = a*(b+c)

(k+1)^2で割り切れるのはひと目でわかる
0643132人目の素数さん垢版2018/10/10(水) 17:14:27.81ID:Ax45ymrl
nは平方数でない自然数とする。
√nを十進法で無限小数の形に表記したときの、小数点以下i桁目の数字をa[n,i]とする。
次の命題は偽であることを証明せよ。

「任意の自然数kに対しa[n,k]が0または1となるようなnが存在する。」
0644132人目の素数さん垢版2018/10/10(水) 17:22:30.93ID:cFkgEp8b
「整数x,y,zに対し、5x^3+11y^3+13z^3=0 ⇒ x=y=z=0を示せ」
ぐぐったら海外の掲示板が出てきて、mod 7 を使うっぽいんだけど、明確な答えがありませんでした…
分かる人いますか…?
0646132人目の素数さん垢版2018/10/10(水) 17:30:41.76ID:H2Q7m9TT
>>644
まさにmod 7でいいじゃん。
|x|+|y|+|z|が0でない解が最小となるものとってくる。
mod 7で考えると全部7の倍数。するとx/7,y/7,z/7も解になって矛盾。
0647132人目の素数さん垢版2018/10/10(水) 17:30:58.59ID:wEZbtXig
>>644
整数の3乗を7で割った余りは0か1か6しかない
5p+4q-r=0(pqrは016のどれか)を満たすpqrは000しかない

xyz全て7の倍数ならそれぞれを7で割ったwvuについても最初の三乗についての等式が成立しないとおかしい
しかしwvuも全て7の倍数ではないといけないのでそれぞれ7で割ったtsrについても最初の等式が成り立たないとおかしい
しかしtsrも全て7の倍数なので……
こんな感じで無限に小さい組が作れてしまうので矛盾
000以外解がない
0650132人目の素数さん垢版2018/10/10(水) 17:35:48.89ID:ylJVFA/f
>>639
帰納法やってるなら解答ぐらいの途中式で出せるべき
考え方っていうより計算の数こなすのが一番
わからないうちは(k+1)=tみたいな感じで置くと分かりやすいのかも
0651132人目の素数さん垢版2018/10/10(水) 17:36:13.66ID:vEXC+dXU
>>648
ありがとうございます
本のようにいきなりは出せませんが繋がっていることは分かりました
ちなみに本のように間の式なく出せるものなんでしょうか?
0653132人目の素数さん垢版2018/10/10(水) 17:40:38.96ID:vEXC+dXU
>>650
ありがとうございます
やはり本くらいの途中式で出せるんですね
もっと問題演習をこなして精進します
皆さんありがとうございました
0654132人目の素数さん垢版2018/10/10(水) 18:03:18.85ID:VXF0ffa4
>>640
かなり数学をやり慣れている人の文字に見える
k+1が共通していることに気付けないとは思えない
ちょっと疲れてるんでは?
0656132人目の素数さん垢版2018/10/10(水) 18:17:24.68ID:RG/gU3xe
>>630
「性質yを温度測定に使用する」は「yと温度は線形関係にあるとみなす」ってことじゃないの?
0658132人目の素数さん垢版2018/10/10(水) 18:22:15.48ID:FjabLsPu
p=7, a≠0 (mod p) とすると、フェルマーの小定理より
 (a^3 +1)(a^3 -1) = a^(p-1) - 1 ≡ 0 (mod p)
 a^3 ≡ ±1  (mod p)

>>646 >>647
 (p, q, r) = (1, 6, 1) (6, 1, 6)
0662132人目の素数さん垢版2018/10/10(水) 19:42:57.22ID:4xxA7Z/e
基準点0mのA地点で1ポイント
B地点ではXポイント
C地点ではYポイント
D地点だとZポイント
AからDへ行くに従って増加するポイントを計算する方法を教えてください
たとえばBは500m地点にあり300P、Cは1000m地点にあり800P、Dは2000mで1400Pという場合
どういう式になるのでしょうか?
0663132人目の素数さん垢版2018/10/10(水) 20:25:32.68ID:DH+UqkdO
いや、分からない問題って問題の意味が分からない問題のスレじゃないんだぞ
0664132人目の素数さん垢版2018/10/10(水) 20:27:29.46ID:2GqbBXjh
今、三角関数のページを読んでるけど、本当に難しい・・・・・。
何が難しいかって、今までだったらとりあえず論理は追えたけど、
三角関数はそうはいかない。
この数字どこから出てきたの!!!!!????????
そんなのばっかり・・・・・・・・・・・・・・。
マジで意味不。
0667132人目の素数さん垢版2018/10/10(水) 21:00:39.95ID:/kAilI1U
n=10まで一致する式

    {2^n+2^(n−1)+n−4−α/12+643(n−5)α/120−2251β/720
   +501(n−7)β/112+20107γ/840+80167(n−9)γ/90720}
q=―――――――――――――――――――――――――――――――――
    {2^(n+2)+2^(n−1)+2n−10−{(n−2)^2(n−4)}+607(n−5)α/40
   −357β/40+10607(n−7)β/840+1339γ/20+822251(n−9)γ/362880}

,α=(n−1)(n−2)(n−3)(n−4),β=α(n−5)(n−6),γ=β(n−7)(n−8)


この関数をガンマ関数を使って補正してくれ〜(・ω・)ノ
0670132人目の素数さん垢版2018/10/10(水) 22:15:53.18ID:VAAOTxkF
『アルゴリズムイントロダクション』を読んでいます。

枢軸変換をしていって、目的「関数」 z が以下のようになったときに、
最適解が、 28 になるのは明らかですよね?

z = 28 - (1/6) * x_3 - (1/6) * x_5 - (2/3) * x_6

『アルゴリズムイントロダクション』には、


本章で後ほど証明するが、この状況は、基底解が最適解であるように
線形計画が書き換わったときにだけ起きる。


などと書いてあります。

これは、なぜでしょうか?
0671132人目の素数さん垢版2018/10/10(水) 22:16:51.80ID:VAAOTxkF
訂正します:

『アルゴリズムイントロダクション』を読んでいます。

枢軸変換をしていって、目的「関数」 z が以下のようになったときに、
最適目的値が、 28 になるのは明らかですよね?

z = 28 - (1/6) * x_3 - (1/6) * x_5 - (2/3) * x_6

『アルゴリズムイントロダクション』には、


本章で後ほど証明するが、この状況は、基底解が最適解であるように
線形計画が書き換わったときにだけ起きる。


などと書いてあります。

これは、なぜでしょうか?
0672132人目の素数さん垢版2018/10/10(水) 22:19:14.22ID:VAAOTxkF
>>671

実際、後に、双対性により証明しています。
でも、明らかですよね。
0674132人目の素数さん垢版2018/10/11(木) 00:00:15.32ID:pE1ftl4e
>>672

別にわざわざ後で、証明するまでもなく、この時点で最適解が得られていることは明らかですよね。
0675132人目の素数さん垢版2018/10/11(木) 00:44:09.83ID:XBFA4KXK
>>669
p=13, a≠0 (mod p) とすると
 (a^3 -1)(a^3 +1)(a^3 +5)(a^3 -5) = (a^6 -1)(a^6 +1-2p) ≡ (a^6 -1)(a^6 +1) = a^(p-1) -1 ≡ 0 (mod p)

a^3 = ±1, ±5 (mod p)

5x^3 +11y^3 + pz^3 = 0 ⇒ x≡y≡0 (mod p)
∴ z^3 ≡ 0 (mod pp)
∴ z≡0 (mod p)
0676132人目の素数さん垢版2018/10/11(木) 01:05:37.64ID:OZ6fRFUS
こうもできる。参考までに。
>>669
(-11/5)^4 ≡ 3^4 ≡ 3 (mod 13) (∵ -11/5 ≡ 3 (mod 13))
∴ (-11/5) not in ker(-)^4 = im(-)^3。
∴ 5x^3 + 11y^3 ≡0 (mod 13) ⇒ x ≡ y ≡ 0 (mod 13) (∵ otherwise (-11/5) ≡ (x/y)^3 (mod 13))
x ≡ y ≡ 0 (mod 13) ⇒ 13z ≡ 0 (mod 13^3) ⇒ z ≡ 0 (mod 13)
0677132人目の素数さん垢版2018/10/11(木) 10:49:26.73ID:JZn9qutH
大学の数学を勉強したいと思うのですが、どのような順番で勉強するのがよいでしょうか。
まずは微積分、線形代数から始めてみようと思うのですが、この後はどうしたらいいのでしょうか。
0678132人目の素数さん垢版2018/10/11(木) 11:02:30.62ID:dFMUDM3M
集合と位相とか?
興味のある分野を見つけて、その勉強に必要な知識を逆算する方が良いと思うが
0679132人目の素数さん垢版2018/10/11(木) 11:18:52.86ID:JqxDHm5z
すべての内角が120°である凸六角形の6辺の長さをa,b,c,d,e,fとおくとき、これらの中で相異なるものは最大でも3種類しかないことを示せ。
0680132人目の素数さん垢版2018/10/11(木) 11:21:26.50ID:JZn9qutH
>>678
最終的に数理ファイナンスを勉強したいと思うのですが、高校数学までしか勉強したことがなくて…
0682132人目の素数さん垢版2018/10/11(木) 12:33:42.94ID:QTHfApUE
>>679
本当にそう?
(a,b,c,d,e,f)=(4,7,5,2,9,3)は?
0686132人目の素数さん垢版2018/10/11(木) 17:36:27.92ID:ihKDrhDc
>>666
たぶんこれです
どうもありがとうございます
Eが3000mのとき何Pが予想されるか
Fのポイントが5000PならFは何mなのか
も計算したいので、グラフを描くことになるだろうとは考えてました
0688132人目の素数さん垢版2018/10/11(木) 20:55:26.23ID:JxWPyNKY
>>687
係数行列の行列式を計算するだけだが
何が分からないのだ?
0693132人目の素数さん垢版2018/10/11(木) 22:45:36.33ID:ofJjjGE+
eの2.1乗を小数点第3位まで計算したいです。
電卓そろばん計算機コンピュータ計算尺などがない、いわゆる手計算の場合、
どうやって求めるのが手っ取り早いですか?

試験中で使えるぐらいの実践的な方法を教えてください。
0696132人目の素数さん垢版2018/10/11(木) 23:09:06.04ID:7PKu0HUr
>>693
VIPの方でもマルチしてたんですね

私はそんな試験問題出すのは現実的ではないので、あなたが何か勘違いをしてるんじゃないかと思ってるんです

たとえば、他の方法を使えば簡単に求められるだとかですね

元の問題を書いてください
0697132人目の素数さん垢版2018/10/11(木) 23:12:57.77ID:0weyKuKI
e^2 を計算して、1+0.1+(0.1)^2/2 あたりを掛け算すればいいんじゃないの?
eを覚えてないなら…1+1+1/2+1/6+…で頑張る
こんなのやりたくないけどな
0698132人目の素数さん垢版2018/10/11(木) 23:15:19.32ID:xmxC4T19
(1+x/n)^nがe^xに一様収束することを示せという問題が解けません。
教えてください!
0699132人目の素数さん垢版2018/10/11(木) 23:25:05.97ID:Rq7tM4w4
>>695
kの値はあってます
別の方に聞いた結果、xyzの関係はx:y:zで表すそうです
2個の連立同次一次方程式のx:y:zの関係は公式で求められますね
ありがとうございました
0701132人目の素数さん垢版2018/10/11(木) 23:27:42.34ID:+j9+yq4P
>>687>>699
kx+y-3z=0から

y=3z-kx……α
kx=3z-y……β

5x-3y-kz=0にαを代入して

kz=5x-3y=5x-3(3z-kx)=5x-9z+3kx……@

4x-7y+(k+1)z=0にαを代入して

(k+1)z=7y-4x=7(3z-kx)-4x

kz=21z-7kx-4x-z=20z-7kx-4x……A

@とAから

5x-9z+3kx=20z-7kx-4x

10kx=29z-9x……B

Bにβを代入して

10(3z-y)=29z-9x
30z-10y=29z-9x
∵z=10y-9x……C

Cから

x=(10y-z)/9

y=(9x+z)/10
0702132人目の素数さん垢版2018/10/11(木) 23:29:27.27ID:XBFA4KXK
>>679

120゚をなす3方向への射影を考えると
 (a-d)/2 + (b-e) + (c-f)/2 = 0,
 (b-e)/2 + (c-f) + (d-a)/2 = 0,
 (c-f)/2 + (d-a) + (e-b)/2 = 0,
これより
 a-d = c-f = e-b,

>>682 はこれを満足する。
0704132人目の素数さん垢版2018/10/11(木) 23:50:10.14ID:JqxDHm5z
p,rは相異なる素数、qは1<q<pをみたす素数とする。
(p,q)/r!が整数となる素数の組(p,q,r)をすべて求めよ。
0707132人目の素数さん垢版2018/10/12(金) 00:42:19.48ID:qmB9G7el
>>706

(1+x/n)^n = exp(n log (1+x/n)) で exp は局所一様連続だから n log(1+x/n) → x が局所一様収束を言えば良い。
n log (1+x/n) = x + nO((x/n)^2) なので桶。
0709132人目の素数さん垢版2018/10/12(金) 12:36:23.22ID:jKHSwFRK
直方体のどの3点をむすんでひらいて得られる三角形も、鈍角三角形ではないことを示せ。
0711132人目の素数さん垢版2018/10/12(金) 15:45:15.75ID:oQ+V5cXR
意味はわかるけどしょうもない。
頂点の座標を全非負にとればOA・OB全部非負。
0713132人目の素数さん垢版2018/10/12(金) 17:17:04.29ID:jKHSwFRK
aを実数とする。
次の式が成立する0でない整数m,nが存在するためのaの条件を求めよ。
(m^2+1)/m = (n+a)/n
0714132人目の素数さん垢版2018/10/12(金) 17:56:12.87ID:jKHSwFRK
xyz空間の点Aと点Pは、OA=3、AP=2、1≦OP≦3/2を満たしながら動く。
ただしOは空間の原点である。
折れ線OAPの動きうる領域の体積を求めよ。
0715132人目の素数さん垢版2018/10/12(金) 18:02:53.21ID:jKHSwFRK
(1)次の3条件を満たす四面体の例を挙げよ。
・どの辺の長さも整数
・どの面の面積も整数
・体積は整数

(2)(1)において、少なくとも1つの条件で「整数」を「素数」に変更する。その場合、3条件を満たす四面体が存在するか。
存在する場合、どの条件を変更してもよいか、すべて述べよ。
0716132人目の素数さん垢版2018/10/12(金) 19:56:35.44ID:b/v1Oc9z
確率について

宝くじでのお話です
一等が0.000009713007815474608%の確率の物があります
今回自分は287口購入し0.002787633243041213%という確率で1等が当たることになりました
これは3桁近く確率が上がっていますよね?

例えばなんですけど0.1%の物が2桁確率が上がり10%になったらかなり当たりそうな気がしますが今回のように3桁近く上がっても正直当たる気配は恐ろしい程ありません

それは元の確率が恐ろしい程低いからというのが原因ではあると思うのですが0.1%→10%より確率は上がっているとみてよろしいのでしょうか?
小数点第〜以下は何桁上がろうと確率の上昇率は無意味なのでしょうか?
0717132人目の素数さん垢版2018/10/12(金) 20:25:35.46ID:6VjVia9c
人生は有限時間しかないので、無限回抽選ができるわけでなく
宝くじが年4回あるとして、4*60年で一生に240回しか引けない

240回程度で0.002%を一度でも引ける確率はあまり高くないので、
毎回287口買ってても、60年で宝くじ1等に一度でも当選する確率は1000回に1回とかしかない

案外引けそうじゃんと思うかも知んないけど、期待値で言えば毎回287枚買うのを1万年続けても一度しか当たらないみたいな感じだから

何枚買おうと一生のうちに億万長者になれる確率がかなりゼロに近いのは変わらない
0718132人目の素数さん垢版2018/10/12(金) 20:29:01.09ID:b/v1Oc9z
ありがとうございます
以前1億で3%ちょいで一度に複数口買って効果があるのは数億単位お金をつぎ込まないと無意味と聞いたことがあります
やっぱこのレベルだと対して変わらないんですね…
大人しく10口くらいにしてあくまでお遊びなの忘れないようにします
0719132人目の素数さん垢版2018/10/12(金) 20:31:39.04ID:6VjVia9c
一生が100万年くらいあって、無限回抽選ができるなら
1000倍早く当選するけどね
一生はそんなにないから…
0720132人目の素数さん垢版2018/10/12(金) 20:35:11.88ID:b/v1Oc9z
>>719
そうですよね
仮にお金があったとしても寿命があるわけですし
それを考えると当選確率だけでなく宝くじに参加できる回数も考慮しないとで
やっぱ恐ろしい
0721132人目の素数さん垢版2018/10/12(金) 20:41:32.21ID:6zXSta7a
■■■□□□■■■
■■■□□□■■■
■■■□□□■■■
□□□■■■□□□
□□□■■■□□□
□□□■■■□□□
■■■□□□■■■
■■■□□□■■■
■■■□□□■■■ 👀
Rock54: Caution(BBR-MD5:1341adc37120578f18dba9451e6c8c3b)
0722132人目の素数さん垢版2018/10/12(金) 22:43:22.37ID:FcltUanb
数学のことを訊ける知人がいないので、ここに質問させていただくことにしました
宜しくお願いいたします

○原チャリの法定最高速度である時速30キロはマッハでいうとマッハ幾つになるのでしょうか?
ちょっと調べたらマッハ1は時速約1200キロと書いてありました
変な質問で申し訳ありませんが、どうかお答えください
0723132人目の素数さん垢版2018/10/12(金) 22:52:17.59ID:72cesl8m
マッハとは音速と比べてどうかという話なんですね
室温程度ならマッハはあなたのいうくらいになるので、0.025マッハくらいですかね
0724132人目の素数さん垢版2018/10/13(土) 00:35:56.45ID:YOeldhda
>>713
こんな感じじゃないのか

r = n / m とする
n, m は0でない整数 ⇔ r は 0 でない有理数

元の式に n = rm を代入して r について解くと
r = a / (m^2 - m +1)

右辺の分母は整数なので
r は 0 でない有理数 ⇔ a は 0 でない有理数
0725132人目の素数さん垢版2018/10/13(土) 12:24:56.33ID:yOHq4j0d
>>724
aが有理数ってのが必要条件であるのはほとんど自明だけど、
十分条件にはなってないでしょ。

たとえば、|a|<1だとnの整数解が存在しないことは簡単に示せる。
0726132人目の素数さん垢版2018/10/13(土) 12:52:29.68ID:41sNyvN9
m^2 -m +1 = a/r = (a/n)m

m^2 +{(a/n) -1}m +1 = 0

a/n が整数で無いとすると、(a/n)d が整数となる最小の正整数 d を取れば

(m^2 +1)d +
0727132人目の素数さん垢版2018/10/13(土) 14:24:24.39ID:N9u30B23
>>713 はどうしようもないでしょ?
|a| = (m+1/m-1)n
となる自然数 m,n が存在する時だけど正直こっからどうしようもない。
右辺が m,n について単調に増大するからアルゴリズムくらいは存在するけど明示的な条件はつくれないよ、たぶん。
数論まともに勉強した知識からでてきた問題じゃなくて適当に思いつくまま作った問題でしょ?
学ぶべきトコなんかなんもないよ。
0730132人目の素数さん垢版2018/10/13(土) 14:44:31.83ID:USJtVTFl
全=無、無=全

これに勝るものはないのでしょうか?
0732132人目の素数さん垢版2018/10/13(土) 16:40:39.64ID:P0/MSS7D
n以下の自然数で、相異なる素数2個の積として表せるものの個数をa[n]、相異なる素数3個の積として表せるものの個数をb[n]とおく。
lim[n→∞] b[n]/a[n] =0 を証明せよ。
0733722垢版2018/10/13(土) 19:30:53.70ID:QfN2n5nP
>>723
感謝
ありがとうございました
0735132人目の素数さん垢版2018/10/13(土) 21:49:52.04ID:P0/MSS7D
kを非負整数とし、自然数nについての関数
f(n)=n^2+kn+1
を考える。f(1),f(2),...f(100)のうち素数であるものの個数をg(k)とおくとき、g(k)の最小値を求めよ。
またそれを与えるkを全て決定せよ。
0736132人目の素数さん垢版2018/10/14(日) 01:52:32.09ID:xkRoYFRI
g(2)=0が最小なのは1秒でわかるが、それ以外にg(k)=0に
なるkがあるかどうかは知らん。
0737132人目の素数さん垢版2018/10/14(日) 02:20:20.60ID:t/H/Tw4Y
質問です。

f[n](x) = (1/x d/dx)^n exp(-x)/x

とします。

f[0] = exp(-x)/x、f[1] = -(x+1)exp(-x)/x^3、f[2] = (x^2+3x+3)exp(-x)/x^5、
f[3] = -(x^3+6x^2+15x+15)exp(-x)/x^7、f[4] = (x^4+10x^3+45x^2+105x+105)exp(-x)/x^9、…

lim[n→∞] f[n](-1) 2^n n!/(2n)! を求めたいのです。
どうも -1/e に収束するらしいです。
どなたか証明できますか?
0743BLACKX ◆SvoRwjQrNc 垢版2018/10/14(日) 03:59:40.77ID:WRFSD9Ui
>>735
この手の問題ってk1 k2って置いて足したパターンはいくつ?って解くんだけど
そもそもn^2+kn+1だから掛けたら1になる数字しかない
2→(n+1)^2

ちなみに0も存在するけどn^2+1で問題の定義から虚数解なのでNG
0744132人目の素数さん垢版2018/10/14(日) 06:25:27.25ID:0CPQSloM
>>737 >>739

f[0](x) = exp(-x)/x = √(2/πx) K_{1/2}(x),

f[n](x) = (-1)^n exp(-x) Σ[k=0,n] C(n+k, n-k) (2k-1)!! / x^(n+k+1)

 = √(2/πx) K_{n+1/2}(x),

ただし (-1)!! = 1!! = 1 とする。

f[n](-1) = √(-2/π) K_{n+1/2}(-1),

K_{…}(x) は第1種の不完全楕円積分と云うらしい。
0745132人目の素数さん垢版2018/10/14(日) 06:38:59.38ID:6VEy8x08
>>744
それなんです。
変形ベッセル関数でパラメータが半整数の関数。
それの n→∞ のときの >>737 の極限が求まるというレスがこのスレ?であってそれの証明がわかんなくて困ってるんですよ。
まぁ困ってるって言っても気持ち悪いだけですけど。
0746132人目の素数さん垢版2018/10/14(日) 06:58:37.10ID:0CPQSloM
>>744
まちがえた。K_{…}(x) は第2種の変形ベッセル函数でござった。

f[n](x) = √(2/πx) K{n+1/2}(x)

 = (1/n!) (x/2)^n∫[1,∞] exp(-xt) (tt-1)^n dt

 = (1/n!) exp(-x)/x ∫[0,∞] exp(-t) t^n (1-t/2x)^n dt

 = (1/n!) exp(-x)/x Σ[r=0,∞] (n+r)! C[n, r] (2x)^(-r),
0747132人目の素数さん垢版2018/10/14(日) 16:20:18.17ID:zUCY3+71
nは3以上の自然数、kは1<k<nを満たし平方数でない自然数とする。
各nに対しn^2-kを素数とするようなkが少なくとも1つ存在することを示せ。
0748132人目の素数さん垢版2018/10/14(日) 17:41:26.76ID:obbD/tK3
>>747
それは証明できないんじゃなかったっけかな?
π(x+y)-π(x)>0 が言えるためには最低でもある定数ε>0が存在してy>x^(1/2+ε)までしか言えないって話を聞いた希ガス。
0749132人目の素数さん垢版2018/10/14(日) 19:18:51.41ID:dxn070zT
基礎的な問題ですいません
1列目の式がなぜ2列目になるのかわかりません
途中式を省かずに教えてもらえますか?
2列目の左側が平方完成でこの形になるのはわかるんですが右側がわかりません

https://i.imgur.com/zXHEZid.jpg
0751132人目の素数さん垢版2018/10/14(日) 19:56:36.87ID:NT2gFiqK
>>749
>>749

a((x-(-a+2)/2a)^2 - ((-a+2)/2a)^2) - a^2-a+2
=a(x-(-a+2)/2a)^2 -a((-a+2)/2a)^2 -a^2-a+2
=おしまい
多分あってると思うけど目がちかちかして自信がない

-aと-a^2を写しまちがえてるのに気が付いてないってのはやめてくれよ
0752132人目の素数さん垢版2018/10/14(日) 20:01:41.87ID:rYLVHAc9
>>749
平方完成 でググればすげー親切な解説見つかるからそれ読むといいよ
ここは数式が見づらいし
0754132人目の素数さん垢版2018/10/14(日) 20:23:11.55ID:KkBlRZKF
>>749
左側の平方完成

-a-a+2-(-a+2)^2/4a
=-2a+2-(-a+2)^2/4a
=2-2a-(a^2-4a+4)/4a
=(8a-8a^2-a^2+4a-4)/4a
=(-9a^2+12a-4)/4a
=-(9a^2-12a+4)/4a∵

以上
0756132人目の素数さん垢版2018/10/14(日) 21:14:26.76ID:9zQHOaSO
質問です

2^x ≠ 12y (x,yともに自然数)

この式の証明は可能でしょうか
0757132人目の素数さん垢版2018/10/14(日) 21:14:42.85ID:dxn070zT
みなさんご親切にありがとうございます
書かれてる式をにらめっこしながら頑張ってみます
0760132人目の素数さん垢版2018/10/14(日) 21:51:11.37ID:dxn070zT
>>759
ありがとうございます

>>749これわかりました

理由があって家でひとりで勉強してるもんだから聞く人がいないんですよ
だからまた初歩的なこと聞きにくるかもしれませんがその時はお願いします
0761132人目の素数さん垢版2018/10/14(日) 22:35:25.34ID:nRibaf3U
もっと順を追ってやっていった方がいいと思うよ
場当たり的過ぎる
先人が試行錯誤の上に作り上げた教育課程を自ら構築するつもりなのか?
0762132人目の素数さん垢版2018/10/14(日) 22:56:37.36ID:5PthFd38
>>750
>>761

質問にちゃんと答えてる人がいる一方で、答えもせずに説教をする馬鹿もいる
この違いがなぜ生まれるのかを考えよう
0764132人目の素数さん垢版2018/10/14(日) 23:16:25.64ID:ZJ8mHGiC
>>751

> -aと-a^2を写しまちがえてるのに気が付いてないってのはやめてくれよ

これ、どういう意味?
0765132人目の素数さん垢版2018/10/14(日) 23:57:26.25ID:NT2gFiqK
>>764
>>749の画像の1行目の後ろの方
-a-a+2
って書いてあるけど
計算はちゃんと -a^2-a+2 を使ってやってるよね?ってこと
0768132人目の素数さん垢版2018/10/15(月) 00:44:37.64ID:id4K6nR+
ああ、分った。
最初の質問者は2行目の右側が問題集かなにかの解答と違っているのが分らない、と言っている、という意味ね。
0769132人目の素数さん垢版2018/10/15(月) 02:23:42.78ID:Zm7H7leg
アラン・コンヌとウィリアム・ジェイムズ・サイディズはどっちの方が頭が良いですか?
0770132人目の素数さん垢版2018/10/15(月) 09:25:03.54ID:FRzng5Ty
>>761
こういう奴がもし教育関係の職についてたら生徒はかわいそうだな
749は平方完成のやり方はわかってるのに式の半分の展開がわからないと言ってる
それならどこが引っ掛かってるのかを察知してあげないとな
「順を追ってやる」→「順を追って教えてる」立場の人ならよくある質問
0772132人目の素数さん垢版2018/10/15(月) 12:22:30.59ID:/TyV0zg+
>>761
教育なんてそんな細部まできっちり決めるもんじゃないぜ
んなことしようとするから
掛け算の順序問題なんてアホな話が出てくる
0773132人目の素数さん垢版2018/10/15(月) 12:54:47.12ID:kOpwpmpP
トランプの束がある
2〜10までの数字が描かれたカードが各スートに1枚ずつと、ジョーカーのカードが24枚ある
全てを混ぜて無作為に切り直して12枚のカードを無作為に引いたとき
その12枚のカードのうちジョーカー以外にいずれも違う数字が書かれている確率はいくらか
0774132人目の素数さん垢版2018/10/15(月) 14:10:53.06ID:7e+ZqB9F
2n枚のカードがあり、それぞれには1,2,...,2nの数が1つずつ書かれている。
この中からn枚のカードを取り出すとき、取り出したn枚のカードに書かれている数の和Sについて考える。

(1)Sは{n(n+1)/2}以上{n(2n+1)-n(n+1)/2}以下の全ての整数値をとるか述べよ。

(2)Sの期待値を求めよ。
0779132人目の素数さん垢版2018/10/15(月) 14:50:05.08ID:j4+CUj76
そんなに自作問題を公開したいなら自作問題スレを作ればどうですか?
あなたの問題を見たい人はそのスレも見てくれるでしょう
0782132人目の素数さん垢版2018/10/15(月) 17:01:42.98ID:I979f5xZ
平川-松村の定理  の証明おしえて
0784132人目の素数さん垢版2018/10/15(月) 17:39:15.65ID:7e+ZqB9F
半径1の円に内接する正七角形の対角線の長さの総和を求めよという問題が分かりません。
正七角形の対角線の長さが直接求まらないのでどう工夫したらいいでしょうか。
0789132人目の素数さん垢版2018/10/16(火) 04:34:53.21ID:xW+nW6TE
mを3以上の自然数とする。
2を底とする対数について、自然数nと実数aを用いて
log_2 (m) = (n+a)/(n-a)
と表すことを考える。 

(1)aをmとnで表せ。
(2)以下の不等式の左辺を最小にする素数pと有理数bの組(p,b)を求めよ。
log_2 (2018) - (p+b)/(p-b) > 0
0791132人目の素数さん垢版2018/10/16(火) 04:48:31.55ID:xW+nW6TE
a[1]=2
a[n+1]=a[n]/{1+a[1]+a[2]+...+a[n]}
で表される数列{a[n]}を考える。

(1)lim[n→∞] a[n] =0 を示せ。
(2)lim[n→∞] (n^k)*a[n]  が0でない有限の値に収束する自然数kを求めよ。
0794132人目の素数さん垢版2018/10/16(火) 08:35:16.42ID:5DYkLdwz
>>756
両辺を3で割ってみる。


>>771
sin(π/8) + sin(7π/8) = √{2-2cos(π/4)} = √(2-√2),
sin(2π/8) + sin(6π/8) = √2,
sin(3π/8) + sin(5π/8) = √{2+2cos(π/4)} = √(2+√2),
sin(4π/8) = 1,
∴ S(π/8) = √(2-√2) + √2 + √(2+√2) + 1,

(2-√2) - 0.76^2 = 1.4224 - √2 > 0,
(2-√2) - 0.77^2 = 1.4071 - √2 < 0,
∴ 0.76 < √(2-√2) < 0.77

(2+√2) - 1.84^2 = √2 - 1.3856 > 0,
(2+√2) - 1.85^2 = √2 - 1.4225 < 0,
∴ 1.84 < √(2+√2) < 1.85

(与式) > 0.76 + 1.41 + 1.84 + 1.00 = 5.01
(与式) < 0.77 + 1.42 + 1.85 + 1.00 = 5.04


>>784
 辺   L1 = 2sin(π/7) = -2sin(8π/7),
 対角線 L2 = 2sin(2π/7),
 対角線 L3 = 2sin(3π/7) = 2sin(4π/7),
 いずれも7本づつある。
 -L1 + L2 + L3 = 2{sin(2π/7)+sin(4π/7)+sin(8π/7)} = √7,
 L1・L2・L3 = √7,
 L3 = L1・(3-L1^2)
 
 L^6 -7L^4 +14L^2 -7 = 0,


>>790
存在しない。
 n=2018, 2019, 2020 のとき
  C[n,2018] ≦ C[2020,2] = 2039190 < 123456789
 n≧2021 のとき
  C[n,2018] ≧ C[2021,3] = 1373734330 > 123456789
0795132人目の素数さん垢版2018/10/16(火) 09:19:29.13ID:5DYkLdwz
>>771
S = √(2-√2) + √2 + √(2+√2) + 1 = 5.027339492126…

>>784
L1 = 2sin(π/7) = 0.8677674782351
L2 = 2sin(2π/7) = 1.563662964936
L3 = 2sin(3π/7) = 1.9498558243636

L1+L2+L3 = 4.38128626753476
0798132人目の素数さん垢版2018/10/16(火) 21:45:49.96ID:xW+nW6TE
p,qを素数、kを自然数とする。
△ABCは∠A=60°、AB=p、AC=q、BC=kの三角形である。
p,q,kの間に成り立つ関係式を求めよ。
0799132人目の素数さん垢版2018/10/16(火) 22:53:10.33ID:Rp6DSvYR
少佐と大佐の間には中佐があります
小陰唇と大陰唇の間には何がありますか?
0801132人目の素数さん垢版2018/10/16(火) 23:03:28.51ID:xW+nW6TE
一辺の長さが1の正四面体SとTがある。
Sは空間に固定され、TはSと1点のみを共有しながらSの外部を移動する。
Tが動きうる領域の体積を求めよ。
0803132人目の素数さん垢版2018/10/17(水) 02:09:37.79ID:kvrMD9Ju
xyz空間の半球
x^2+y^2+z^2=1 (x≧0)
を平面x=sおよびx=t(0<s<t<1)で切り、切り分けられた立体のs≦x≦tの部分とt≦x≦1の部分の体積が等しくなるようにする。

いまtをsの関数と見てt=f(s)とおくとき、次の極限を求めよ。

lim[s→1] (1-f(s))/(1-s)
0804132人目の素数さん垢版2018/10/17(水) 02:26:57.50ID:RkkcdSW0
>>737
自己解決。
なんのことはない。
exp(-x)/x をマクローリン展開すればいいだけ。
第0項を除く部分は0にいってしまう。
お騒がせしました。
0805132人目の素数さん垢版2018/10/17(水) 05:14:41.19ID:CNsWZSmr
>>791

S = 1 + a[1] + a[2] + … + a[n] + … = 3.91202535564
が収束するから、n → ∞ のとき
 a[n+1] ≒ a[n] / S,   … 等比数列っぽい。
a[n] ≒ 11.127284700 / S^n,

ln(a[n]) ≒ 2.409400 - 1.364055233655 n,
0806132人目の素数さん垢版2018/10/17(水) 05:21:15.19ID:CNsWZSmr
〔類題〕
半径1の円に内接する正七角形の
 (対角線の長さの総和) - (辺の長さの総和) =
の (2/3)乗 を求めよ、という問題が分かりません。。。
0807132人目の素数さん垢版2018/10/17(水) 05:33:01.02ID:kvrMD9Ju
kを実数とする。
実数xについての方程式
x^3-kx+1 = 0 ...(F)
について以下の問いに答えよ。

(1)kが十分大きいとき、(F)は相異なる3つの実数解を持つことを示せ。

(2)kが十分大きいとき、(F)の3つの解をα、β、γ(α<β<γ)とする。
以下の極限(ア)〜(オ)をそれぞれ求めよ。
(ア)lim[k→∞] α
(イ)lim[k→∞] β
(ウ)lim[k→∞] γ
(エ)lim[k→∞] αβ
(オ)lim[k→∞] γ/α
0808132人目の素数さん垢版2018/10/17(水) 07:10:34.08ID:CNsWZSmr
>>807

(1)
題意より k > 0 としてよい。

F(-1-k/3) = -(k/3)^3 < 0,

F(0) = 1 > 0,

k > 3・(1/4)^(1/3) のとき
F(√(k/3)) = 1 - 2・(k/3)^(3/2) < 0,

F(√k) = 1 > 0,

∴ k > 3・(1/4)^(1/3) のとき
中間値の定理により各区間に実解が1個以上ある。相異なる3つの実解を持つ。

(2)
 (ア) α 〜 -√k - 1/(2k) +3/(8k^2.5) → -∞,
 (イ) β 〜 1/k + 1/k^4 → 0,
 (ウ) γ 〜 √k - 1/(2k) -3/(8k^2.5) → ∞,
 (エ) αβ = - 1/γ 〜 - 1/(√k) - 1/(2kk) → 0,
 (オ) γ/α 〜 -1 + 1/(k^1.5) → -1,
0809132人目の素数さん垢版2018/10/17(水) 07:32:44.28ID:XmI0cwXc
問1: 2多項式の平方の和 f_1^2 + f_2^2 として表される多項式の全体は, 乗法に関して半群をつくる事をしめせ.
(服部昭「現代代数学」 p.5 より)

多項式について特に記載がないのですが, 有理数係数の1変数多項式だと思います。
簡単な例だと
(x^2 + x^2)(x^2 + (2x)^2) = 10x^4 = (x)^2 + (3x)^2
こんな感じで乗法に関して閉じてるらしいのです (本当かな...)
どうかよろしくお願いします。
0813132人目の素数さん垢版2018/10/17(水) 11:41:40.69ID:uOvStamk
y=x^2のグラフの上に傾き正のある直線を引いたところ、a、bの2点で交わった。

x座標が負の点をaとした場合、aのx座標の絶対値はbのそれより小さい。


これはグラフ書くと直感的に明らかですが、図形的に説明する方法はありますか?

直線の式立てて二次方程式の解の公式使えば計算ですぐ分かりますが
直感的に説明できないのが気持ち悪くて
0814132人目の素数さん垢版2018/10/17(水) 11:52:59.02ID:eVoD0jAd
aを通り傾き0の直線を引く。
この直線の傾きを、少し正に/負に 変化させたとき、交点がどのように変化するか考察。
0815132人目の素数さん垢版2018/10/17(水) 11:55:50.76ID:eVoD0jAd
どちらでも、かまわないかもしれないけど、一応訂正
誤:aを通り傾き0の直線を引く。
正:bを通り傾き0の直線を引く。
0816132人目の素数さん垢版2018/10/17(水) 12:12:33.18ID:q4TTBiFC
直観的に明らかとか言ってるけど、x座標が両方とも正になる場合があるのには気付いてる?

単純に
a,bの座標をそれぞれ(Xa,Ya)と(Xb,Yb) 但しXa<Xb
を考えれば
傾き正だから、Yb>Ya (>0)なので、両辺のルートを考えれば |Xb| > |Xa|, になる

図形的に考えれば、「Y座標が大きいほうがY軸から離れている」
ってこと。
0818132人目の素数さん垢版2018/10/17(水) 13:02:17.30ID:uOvStamk
色々な解答ありがとうございますm(_ _)m

両方正になるパターンを忘れてました……
直線がy軸の正の部分と交わるという条件が言いたかったことです。

簡単というか秒で言えそうですね……なぜ煮詰まったのか不思議です。ありがとうございました
0819132人目の素数さん垢版2018/10/17(水) 13:03:43.56ID:uOvStamk
二次曲線と直線が共有点を持つかどうかという問題では、単純に連立するだけでよく、解の範囲が二次曲線の取りうるxyの条件を満たすかどうかは調べる必要が無いのに

二次曲線どうしが共有点を持つかどうか判定する場合にはその条件を調べなければならないのはなぜですか?
0821132人目の素数さん垢版2018/10/17(水) 13:33:56.24ID:uOvStamk
単に連立して得られる方程式の実解と実際の交点が一対一対応しないのはなぜか?ということです。
0822132人目の素数さん垢版2018/10/17(水) 13:48:37.28ID:lYXNgkR/
でかるとせんせーに喧嘩売るぞって話?
0824イナ ◆/7jUdUKiSM 垢版2018/10/17(水) 15:48:40.97ID:T1WitPnt
>>801
正三角錘Tが動く領域内部にある正三角錘Sは領域に含まれない。
Sのすぐ外の部分は3つの領域からなる。
正三角柱4つ={(√3)/4}×4
=√3
扇形柱6つ=π(1^2){(360-90-90-109.5)/360}
=47π/40
球1つ=(4π/3)(1^3)
=4π/3
あわせると、
Tが動く領域=4π/3+47π/40+√3
=(301/120)π +√3
0826132人目の素数さん垢版2018/10/17(水) 16:45:22.47ID:uOvStamk
>>823


楕円x^2+2y^2=1、放物線2y=x^2+11の交点を求めたい。

交点となるxyはx^2=2y-11を満たすので
楕円の式に代入して2y^2+2y-12=0、y^2+y-6=0

y=2,-3となるが、どちらも楕円にはかすりもしてないので解にはならない。楕円の図形的条件を考えないといけない。

こうなるのはなぜでしょうか?
0828132人目の素数さん垢版2018/10/17(水) 16:53:37.26ID:0klAX64q
>>826
x^2+2y^2=1 & 2y=x^2+11
⇔y^2+y-6=0 & x^2=2y-11
であって、2式はワンセット。
y^2+y-6=0を解いた y について x^2=2y-11 を満たす x があるかどうかは確認しないとわからない。
両方OKのときもあれば、片方だけOKのときもあれば、全滅するときもある。
一次式を利用して一文字消去した場合には対応する x が必ず見つかる。
0829132人目の素数さん垢版2018/10/17(水) 16:53:45.31ID:eVoD0jAd
>>826
交点と言うからには、(x,y)を求めてから、言ってください。

y座標だけ求まったとしても、それに対応するxが実数として
存在しなければ、それは、交点ではありません。
0831132人目の素数さん垢版2018/10/17(水) 17:18:22.82ID:uOvStamk
いえ、この場合は実数条件を考慮しないとダメ、というのは分かるんですよ

なぜ直線と二次曲線の交点の場合はそれを考えなくてよくなるのでしょうか?というのが最初の質問です
0832132人目の素数さん垢版2018/10/17(水) 17:36:08.92ID:CLF9yvIF
直線と二次曲線だって考えなきゃダメじゃね?
y=x^2+1とy=0の交点を求めようとして連立させてx^2+1=0とすると虚数解しか出て来なくて解無し、つまり交点無しってわかるだろ?
0833132人目の素数さん垢版2018/10/17(水) 17:42:17.61ID:CVjHYV3z
直線の式をy=ax+b(a,bは実数)とする
ある曲線がこの直線と交わるか交わらないか、という問題を考えよう

連立した方程式を仮にxについて解いて実数解が得られたとすれば、関係式y=ax+bによって対応するyの値も自動的に実数になる
逆に、xについて解いて虚数解が得られたとすれば、対応するyの値も自動的に虚数になる
なので、直線との交点を求める際に限ってはxについて解くかyについて解くかに関わらず、一方の値が実数なのか否かさえ見れば良いことになる

もちろん直線との交点ではない場合は>>826のように、一方の値が実数であったとしてももう一方の値が虚数になることがあり得るので、それも確かめないといけない
0838132人目の素数さん垢版2018/10/17(水) 20:31:32.39ID:9LFKH85i
東大医学部医学科で断然トップの人と、東大理学部数学科で断然トップの人はどっちの方が頭が良いのでしょうか?
0843イナ ◆/7jUdUKiSM 垢版2018/10/18(木) 01:16:28.16ID:fIJ2dSz/
>>839ご指摘ありがとう。
>>837修正。
Tが動く領域は、正三角柱4つと扇形柱6つと球1つからなる。
(正三角柱4つ)={(√3)/4}×4
=√3
(扇形柱6つ)=π(1^2){(360-90-90-109.5)/360}×6
=70.5π/60
=47π/40
(球1つ)=(4π/3)(1^3)
=4π/3
あわせると、
(Tが動く領域)=4π/3+47π/40+√3
=(301/120)π +√3
0844132人目の素数さん垢版2018/10/18(木) 02:44:29.46ID:ybZLuwXw
Oを原点とするxy平面の点A(1,1)を中心とする半径r(1≦r<√2)の円Cがある。

Cの周とx軸との交点のうち、原点Oに近い方をPとする。また、y軸との交点のうち原点に近い方をRQとする。

扇形APQの面積をS(r)とし、また線分OP、線分OQ、Cの劣弧PQとで囲まれる領域の面積をT(r)とする。

このとき、次の極限を求めよ。

lim[r→√2] {(√2 - r)*S(r)}/{T(r)}
0846132人目の素数さん垢版2018/10/18(木) 04:28:52.29ID:Dw4OfxmO
>>826

xx = X とおくと

「楕円」は放物線 X = 1 -2yy となり、
「放物線」は直線 2y = X+11 となる。

これらは (X,y) = (-7,2) (-17,-3) の2点で交わる。

X≧0 の交点のみが(実)xy-平面上の交点(x,y)に対応する。
X<0 の交点は xが虚数になるので、(実)xy-平面上では絣もしない。
0847イナ ◆/7jUdUKiSM 垢版2018/10/18(木) 04:53:42.15ID:fIJ2dSz/
>>845>>843
108°ぐらいかなとは思ったんだけど。
底角1、斜角(√3)/2の二等辺三角形の頂角。
正四面体の辺と辺がなす角。
なぜかと言われても自然の摂理だから。一周を360°と決めたから、109.5°になったとしか言いようがない。
0849132人目の素数さん垢版2018/10/18(木) 17:25:18.82ID:v2a6/08p
正四面体は(1,0,0,0),(0,1,0,0),(0,0,1,0),(0,0,0,1)とか
(-3,1,1,1),(,1-3,1,1),(1,1,-3,1),(1,1,1,-3)で表せる。

中心から2つの頂点を見た時の角度をtとすると、
cos(t)=(-3,1,1,1).(1,-3,1,1)/(9+1+1+1)=-1/3 だから
arccos(-1/3) あるいは、
(180/pi)arccos(-1/3)=109.471220634490691369245999339962435963006843100907948288...°
0852132人目の素数さん垢版2018/10/18(木) 19:27:14.93ID:6fhQd4Cs
頂点が1/4で上に凸の放物線

y=-x^2/676+1/4が

座標(3,10/49)を通るように調整してくれ〜(・ω・)ノ
0854132人目の素数さん垢版2018/10/18(木) 20:57:05.48ID:S3KlGNXW
>>852
y=-x^2/676+1/4 (x≠3),10/49(x=3)
0856132人目の素数さん垢版2018/10/18(木) 23:31:59.86ID:ZLom+Usi
わからない、教えて
抽選ボックスが2つ、どちらかから1つからボールを1つだけ引き当選の有無を確認する。
抽選ボックスAはボールが3コ、ボックスBは7コ。
一等は1本、2等は2本、計3本がどちらかのボックスに偏っているとする。

この時どちらのボックスを引くのが良いか?または同じか?
0857イナ ◆/7jUdUKiSM 垢版2018/10/18(木) 23:33:54.23ID:fIJ2dSz/
(正三角柱4つ)={(√3)/4}×4
=√3
(扇形柱6つ)=π(1^2){(360-90-90-109.47122063449069)/360}×6
=7.052877936550931π/6
=(1.1754796560918218333……)π
(球1つ)=(4π/3)(1^3)
=4π/3
=1.333……
あわせると、
(Tが動く領域)=(2.5088129894251551666……)π+√3
(5/2)π+√3<
(301/120)π+√3=2.508333……
<(2.5088129894251551666……)π+√3

簡単な分数にはならないかと思ったが、そんな簡単じゃなかった。
0858132人目の素数さん垢版2018/10/18(木) 23:45:03.55ID:LmxfrDVL
>>844
r→√2の極限だと高次の微小量を無視すれば円弧PQは直線として考えられるぞ
x=√2-rと置くと
T=x^2
S=x(√2-x)
xS/Tにx=0を代入して、答えは√2だ
厳密な証明は、まあ頑張れ
0859132人目の素数さん垢版2018/10/18(木) 23:47:44.94ID:y4R+MJMW
>>855
こんな感じか?

θ = ∠OAP とし、
AOを斜辺とし、x軸を底辺とする直角三角形の面積をUとすると
S = πr^2 * 2θ / (2π) = θr^2
U = r sin(π/4-θ) / 2
T = 1 - 2U - S

先ほどの直角三角形の辺の長さと角度の関係から
r = 1/cos(∠A) = 1/cos(π/4-θ)
よって U = 1/2 * sin(π/4-θ)/cos(π/4-θ)、S = θ / cos(π/4-θ)^2

T/S = (1 - 2U)/S - 1
= (1 - sin(π/4-θ) / cos(π/4-θ)) 2cos(π/4-θ)^2 / θ - 1

f(θ) = (1 - sin(π/4-θ) / cos(π/4-θ)) 2cos(π/4-θ)^2 とすると
f’(θ) = 2 (cos(2θ) + sin(2θ)) なので
(ここは綺麗な式にしなくてもとにかく微分できていればいい)

lim T/S = lim f(θ)/θ - 1 = f’(0) - 1 = 1
θ→0
0861132人目の素数さん垢版2018/10/19(金) 02:05:32.23ID:gzQJ/Bd2
>>859
ありがとうございます。
美しい結論、程よい難易度ですね
私の作問能力の高さを再確認いたしました
0862132人目の素数さん垢版2018/10/19(金) 02:48:48.84ID:/MhliacY
なんにしろ答えは√2だな
適当な問題の背景が透けて見えてる

2T/(√2-r)が大雑把にTの三角形の高さで、S/(T/(√2-r))はSの底辺の極限。だから√2
0865132人目の素数さん垢版2018/10/19(金) 03:01:29.91ID:gzQJ/Bd2
>>864
数学的深みはゲームとしての面白さではなく研究により得られるものです
私はゲームとしての面白さを追求いたします
0866132人目の素数さん垢版2018/10/19(金) 04:20:06.08ID:jtToVnaO
a, bを正の実数として、双曲線:
(x^2/a^2) - (y^2/b^2) = 1
の上の点P(Pのx座標,y座標はともに正とする)における接線へ
この双曲線の焦点(√(a^2+b^2),0), (-√(a^2+b^2),0)から
下した垂線の足をそれぞれH, H'とすると、
H, H'は頂点A(a,0), A'(-a,0)を直径とする円周上にあることを証明せよ。
0868132人目の素数さん垢版2018/10/19(金) 06:14:46.67ID:rcCrT93A
>>866だけど
スマンが当方はわかった
双曲線の性質を使えばめっちゃ簡単だった
考えてわからない奴はバカ
0869132人目の素数さん垢版2018/10/19(金) 06:49:41.40ID:UmCMoNsS
>>866

原点Oを通らない任意の直線を
 kx - Ly = 1,   … (1)
とする。 (kk+LL≠0)
F から(1)におろした垂線:
 L{x - √(aa+bb)} + ky = 0,
F ' から(1)におろした垂線:
 L{x + √(aa+bb)} + ky = 0,
をまとめて
 Lx + ky = ±L √(aa+bb),   …(2)

(1)と(2)の交点 H,H ' (x,y)では

(kk+LL)(xx+yy) = (kx-Ly)^2 + (Lx+ky)^2 = 1 + (aa+bb)LL,

 xx + yy = {1 + (aa+bb)LL}/(kk+LL),

∴ 右辺が一定値になるように(k,L)をとればよい。

(1) を2次曲線
 {k/x(P)}xx - {L/y(P)}yy = 1,
の点Pにおける接線とし、
 x(P)/k + y(P)/L = aa+bb
とすれば、この条件を満足する。
 xx + yy = aa.
0870132人目の素数さん垢版2018/10/19(金) 08:28:57.75ID:UmCMoNsS
>>869

(1) は双曲線
 (x/a)^2 - (y/b)^2 = 1,
の接線だから
 k = x(P)/aa,
 L = y(P)/bb,

これを使うと
 (ak)^2 - (bL)^2 = 1,
 1+ (aa+bb)LL = aa(kk+LL),
0871132人目の素数さん垢版2018/10/19(金) 13:00:12.33ID:/MhliacY
>>859
いくつかの間違いを修正して、wolframセンセーに頑張ってもらった結果
(一度じゃ計算成功しなかったけど)
答えは√2です

1. Uの定義がおかしい
UはAPを斜辺とし…とすべき(というか、計算ではそうなっている)

2.
T/S = (1 - 2U)/S - 1
= (1 - sin(π/4-θ) / cos(π/4-θ)) 2cos(π/4-θ)^2 / θ - 1
の 2cos(π/4-θ)^2の最初の2はいらない
T/S = (1 - 2U)/S - 1
= (1 - sin(π/4-θ) / cos(π/4-θ)) cos(π/4-θ)^2 / θ - 1

T/S → 0 になる

3.
求めるのは、T/Sではなくて、
(√2-r) (S/T)

>>859のやり方なら、φ=Pi/4-θと置いて、簡略化しながら計算しないと計算量が嫌になるかも。


書くのしんどいから書かないけど
△AOPの面積をVとすれば、V=√2/2 rsinθで
T=2V-Sだから計算はぐっと楽
>>855を書いた時はこれを想定してた
普通に手計算できるレベル
0875132人目の素数さん垢版2018/10/19(金) 16:00:45.25ID:NBYzEtA1
>>873
思い付きの質問、4元数体の関数論があるみたいだから一般論があるのかと思って聞いてみた

>>874
ありがとう
0876学術垢版2018/10/19(金) 16:16:42.06ID:LC9EEibV
数学はモノの方便みたいなところもあるよね。簡略化しすぎるといい体作りに
ならない面があると思うが。まだ数学頭脳はほとんど起きていない。
0877学術垢版2018/10/19(金) 16:17:56.65ID:LC9EEibV
精神のまといを数学者でも雇って数式化してもらいたいなあ。精神障碍者だし。
0878132人目の素数さん垢版2018/10/19(金) 17:06:30.72ID:TGAmzOye
>>872
ヒルベルト空間でよくね
っていうか微積自体ある特殊な内積空間の位相的側面の話では?
0881132人目の素数さん垢版2018/10/19(金) 17:34:52.44ID:mv6/b+kI
100個の自然数 1,2,3,...100から50個の数字を次の条件を満たすように選ぶとどうなるか
条件1 任意の二数は互いに素
条件2 全部の和を最小にする
0882132人目の素数さん垢版2018/10/19(金) 17:34:59.58ID:6IbeljhY
教科書の演習問題についてですが自力でなかなか解けません..
[問題]
{Yn}がn=1,2,...について自由度nのχ^2分布に従う確率変数のとき、
(Yn-n)/√(2n)が標準正規分布に法則収束することを示せ。

という問題です。
積率母関数を求めて極限を取る方法で示そうとしているのですがどうもうまくいきません。。。
解説お願いします。
0884132人目の素数さん垢版2018/10/19(金) 18:25:23.36ID:mv6/b+kI
>>881
> 条件1 任意の二数は互いに素

ごめん。「互いに素」ではなくて「互いに約数、倍数の関係になっていない」に訂正
0887132人目の素数さん垢版2018/10/19(金) 22:57:17.43ID:tYw/U/2m
以下の命題を証明してください。

F を閉凸集合、 z を F に含まれない点とする。このとき、次を満たすベクトル a およびスカラー Θ が存在する:

∀x ∈ F、 a^T * z < Θ < a^T * x.
0888132人目の素数さん垢版2018/10/19(金) 23:12:45.48ID:DKRhmVm3
fを実係数n次多項式、s_0,s_1,...,s_nを相異なる実数とすると
f(x+s_0),f(x+s_1),f(x+s_2),...,f(x+s_n)は一次独立であることを示してください
0889132人目の素数さん垢版2018/10/19(金) 23:29:01.73ID:rSBjQu9b
方法A:X回中65/10000X回成功
方法B:Y回中7/1000Y回成功

という統計データがあるとき
「真の(正確な)成功確率が方法Bの方が高い」確率が
80%以上である為の最小のXとYを求めよ
よろしくお願いします
0890132人目の素数さん垢版2018/10/19(金) 23:40:34.82ID:5btDxqP5
q=1−{{165n−3n^2+936}/(193n−7n^2+1248)}

n=3のときにqはいくつですか?
0893132人目の素数さん垢版2018/10/20(土) 02:36:55.24ID:/zyiypza
>>888
実数体のなかでならn=0以外では成立しない。
多項式環のなかで一次独立ならVandermonde行列式を考えれば自明。
0894132人目の素数さん垢版2018/10/20(土) 10:16:02.31ID:fEQDQMFE
xyz空間の円板C:x^2+y^2=1,z=0の周または内部の点A(a,b,0)における方べきの値をf(a,b)とおく。
また空間の原点をOとしたときの半直線OAとx軸の正の部分とのなす角をθ(a,b)、積f(a,b)・sinθ(a,b)=g(a,b)と定める。
ただしθ(a,b)は0≦θ(a,b)<2πを動く。

(1)f(a,b)をa,bで表せ。
(2)a,bが動くとき、点P(a,b,g(a,b))が囲む領域をVとする。Vを平面x=t(-1≦t≦1)で切った断面図を描け。
0895132人目の素数さん垢版2018/10/20(土) 10:17:36.58ID:fEQDQMFE
894は(1)は簡単でしたが、(2)で断面図を描くところで手が止まります。極座標でもやってみましたが難しくて計算ができません。
教えてください。
0896132人目の素数さん垢版2018/10/20(土) 10:21:27.53ID:18CdzPVG
以下の命題を証明してください。

F を閉凸集合、 z を F に含まれない点とする。このとき、次を満たすベクトル a およびスカラー Θ が存在する:

∀x ∈ F、 <a, z> < Θ < <a, x>.
0897132人目の素数さん垢版2018/10/20(土) 12:12:10.35ID:saQgO1Bc
サイコロを繰り返し投げ、出た目が直前の回に出た目の約数でなくなったら終了します。
n回目にサイコロを投げ、かつその目が1である確率 p[n] を求め、n回目に終了する確率をp[n]とp[n+1]を用いて表してください。
プロセス(解き方)もお願いします。
0898132人目の素数さん垢版2018/10/20(土) 12:36:42.60ID:35006q00
>>893
どう自明なのかわからないです
0899132人目の素数さん垢版2018/10/20(土) 12:40:29.18ID:fEQDQMFE
>>897
普通に考えればいい
n-1回目が
1→n回目が2,3,4,5,6で終了
2→n回目が3,4,5,6で終了
3→n回目が2,4,5,6で終了
4→n回目が3,5,6で終了
5→n回目が2,3,4,6で終了
6→n回目が4,5で終了
あとはa[n]を上の結果使ってa[n-1]とつなげるだけ
p[n]経由しなくても直接解ける
0900132人目の素数さん垢版2018/10/20(土) 12:46:32.17ID:/MrLnf1N
2のべき指数で分類するとこうか?

>>885
S = [64] + [・] + [48] + [40+56] + [36+44+52+60] + [34+38+42+…+66] + [35+37+39+…+99]
                                   (9個)        (33個)
 = 64 + 0 + 48 + 96 + 192 + 450 + 2211
 = 3061,


>>886
 48→24
S = 64 + 0 + 0 + 120 + 192 + 450 + 2211
 = 3037

>>891
 4の倍数のうち、40,52,56,60 →半分, 64→16

S = [・] + [・] + [16] + [24] + [20+28+36+44] + [26+30+34+…+66] + [35+37+39+…+99]
                                 (11個)        (33個)
 = 0 + 0 + 16 + 24+ 128 + 506 + 2211
 = 2885,
0901132人目の素数さん垢版2018/10/20(土) 13:10:33.07ID:yaPDybmU
16+20+22+24+26+28+30+33+34+35+
36+37+38+39+41+42+43+45+46+47+
49+50+51+53+54+55+57+58+59+61+
62+63+65+67+69+71+73+75+77+79+
81+83+85+87+89+91+93+95+97+99=2830
0904132人目の素数さん垢版2018/10/20(土) 13:58:22.46ID:w/u4gzJ2
1〜100だからかえってわかりにくい。
いっそ1〜10000から5000個とかで考えた方がいい。
奇数kに対して2べき×kの全体をC[k]とする。
1〜10000=C[1]+C[3]+…C[9999]
同じ類から2つ取れないので各類から一個づつ。
C[9999]は全部9999の倍数なので3333は取れない。
よってC[3333]から選ばれるのは6666の倍数。
同様にしてC[1]〜C[3333]の各類で選ばれるのは2…6666の倍数。
同様にしてC[1]〜C[1111]の各類で選ばれるのは4…13332の倍数。

の必要条件出しといて十分性チェックして完了。
0905132人目の素数さん垢版2018/10/20(土) 14:59:57.68ID:saQgO1Bc
>>899
質問の目的はn回目に終了する確率を上手に求めることです。誘導を使うも、誘導を無視してn回目に終了する確率を直接求めてもらうも構いません。ただしなるべく計算のいらない面白い解法を追求したいです。
0906132人目の素数さん垢版2018/10/20(土) 15:05:59.12ID:saQgO1Bc
>>905はいわば>>897の補足みたいなものと解釈してください、レス先を間違えました

>>899
a[n]とはなんでしょうか
何を主張するものか理解できないし、もっと詳しく説明して頂けないでしょうか
>>897を確認してください
0908132人目の素数さん垢版2018/10/20(土) 15:32:23.56ID:vN0Acfvc
n回目の目がkで未終了の確率p(k,n)、q(k,n)=6^np(k,n)として
q(1,n+1)= q(1,n)+…+ q(6,n)
q(2,n+1)= q(2,n)+ q(4,n)+ q(6,n)
q(3,n+1)= q(3,n)+ q(6,n)
q(4,n+1)= q(4,n)
q(5,n+1)= q(5,n)
q(6,n+1)= q(6,n)
こんなモンなんか一工夫したいと思える余地ない希ガス。
0909132人目の素数さん垢版2018/10/20(土) 18:02:35.43ID:kWakH5+C
>>890
次の式はn=3,[0≦c≦124]の範囲ですべてq=10/49

∴q=1−{{165n−3n^2+(39+39c)}/{(216−c)n−7n^2+(52+52c)}}

■q=10/49 ∵n=3,c=23
0911132人目の素数さん垢版2018/10/20(土) 19:19:30.40ID:fEQDQMFE
2^n+1と3^n+2を17で割ったとき、余りが等しくなるような最小の自然数nを求めよ。
0912132人目の素数さん垢版2018/10/20(土) 19:23:53.29ID:fEQDQMFE
凸六角形ABCDEFの対角線AD、BE、CFの長さはいずれも1であるという。
このような凸六角形の最大値と最小値が存在するかを述べよ。存在するならばその値を求めよ。
0915132人目の素数さん垢版2018/10/20(土) 20:52:35.60ID:fEQDQMFE
aとbは互いに素な自然数で、cとdも互いに素な自然数である。
ab=cdかつa≠cかつa≠dであるa,b,c,dの例を挙げよ。また、a=2018となる場合は存在するか。
0918132人目の素数さん垢版2018/10/21(日) 01:20:02.15ID:wgL9G251
>>910

I_n = ∫[0,1] 1/(1+x^n) dx
 = (1/n)∫[0,1] 1/(1+y) y^(1/n -1) dy
 = (1/2n) {ψ((n+1)/2n) - ψ(1/2n)},

ここに ψ(x) = Γ '(x)/Γ(x), (digamma函数)

∫[0,1] 1/(1+x^2018) dx
 = (1/4036) {ψ(2019/4036) - ψ(1/4036)}
 = 0.999656719605351957806207034918974864517522986561577745876
0919132人目の素数さん垢版2018/10/21(日) 01:52:10.61ID:JIJeBFXr
先日ここでマッハの意を問わせてもらった者です
その節はありがとうございました

ついでに伺いたいのですが「平均速度マッハ1」という表現(書き方)は間違いでしょうか?
例えば「平均時速60キロ」は聞き慣れててしっくり来るのですけど
「平均速度マッハ1」ってのは聞き慣れていません

もし平均速度をマッハで書きたい場合はどうすればいいですか?
0921132人目の素数さん垢版2018/10/21(日) 06:56:44.00ID:k1ajnchQ
916です。ヒントだけでも教えてください。focus gold なども見ましたが全然わかりません。
0922132人目の素数さん垢版2018/10/21(日) 07:24:33.26ID:p2Myh/Bc
以下の命題を証明してください。

F を閉凸集合、 z を F に含まれない点とする。このとき、次を満たすベクトル a およびスカラー Θ が存在する:

∀x ∈ F、 <a, z> < Θ < <a, x>.
0927イナ ◆/7jUdUKiSM 垢版2018/10/21(日) 11:41:46.10ID:MYCwKHXh
>>773答えもう出てる?
>>857
2〜10は各スート一枚ずつなんで、
9×4=36枚
ジョーカー24枚
あわせて36+24=60枚
すべての取り方は、
60C12=60・59・58・……・49/12・11・10・……・1

つづく。
0928132人目の素数さん垢版2018/10/21(日) 12:55:18.85ID:aS+HsF0h
連続するn個の自然数k,k+1,...,k+n-1を2つのグループに分ける。また次の操作(T)を行う。

(T)一方のグループに含まれる自然数の和と他方のグループに含まれる自然数の和が等しくなるようにする。

(1)(T)が可能なとき、k,nはどのような整数か。

(2)あるk,nをとったところ、その連続する自然数は(T)が可能であった。またその連続する自然数の中から、ある自然数1つを取り去ると、(T)は不可能になるという。取り去る自然数が満たすべき条件を述べよ。
0930132人目の素数さん垢版2018/10/21(日) 13:31:59.24ID:l2E3XuiN
>>927
1万回のシミュレーションを1万回やって平均を求めてみた

x=c(rep(2:10,4),rep(0,24))
f <- function(){
y=sample(x,12)
z=y[which(y!=0)]
length(z)==length(unique(z))
}
re=replicate(1e4,mean(replicate(1e4,f())))

> summary(re)
Min. 1st Qu. Median Mean 3rd Qu. Max.
0.0992 0.1085 0.1106 0.1106 0.1127 0.1217
0931イナ ◆/7jUdUKiSM 垢版2018/10/21(日) 15:33:14.26ID:MYCwKHXh
>>927
(確率)=(その場合の数)/(すべての場合の数)
すべての場合の数は先に示した。
その場合の数は、
ジョーカーが1枚2枚のときは数字のカードが少なくとも1枚2枚かぶるのでありえない。
よってジョーカーが3枚から12枚のときを考える。
ジョーカーが3枚のとき、
24C3・4^9=23・22・4^10
ジョーカーが4枚のとき、
24C4・4^8=6・23・11・7・4^8
ジョーカーが5枚のとき、
24C5・4^7=23・22・21・4^8
ジョーカーが6枚のとき、
24C6・4^6=23・11・7・19・4^7
ジョーカーが7枚のとき、
24C7・4^5=23・11・19・18・4^6
ジョーカーが8枚のとき、
24C8・4^4=23・11・19・9・17・4^4
ジョーカーが9枚のとき、
24C9・4^3=23・11・19・17・4^5
ジョーカーが10枚のとき、
24C10・4^2=23・11・19・17・6・4^3
ジョーカーが11枚のとき、24C11・4=23・19・17・3・7・4^3
ジョーカーが12枚のとき、24C12=23・19・13・7・4
これらをすべて足して、すべての場合の数で割ると、
――つづく。
0932132人目の素数さん垢版2018/10/21(日) 17:09:21.51ID:l2E3XuiN
>>930
re=NULL
re[1:2]=0
for (k in 3:12){
re[k]=choose(24,k)*choose(9,12-k)*4^(12-k)/choose(60,12)
}
sum(re)

> sum(re)
[1] 0.1106278

シミュレーション解とほぼ一致
0933132人目の素数さん垢版2018/10/21(日) 17:30:10.66ID:l2E3XuiN
Prelude> choose (n,r) = product[1..n] `div` product[1..n-r] `div` product[1..r]

Prelude> fromIntegral(sum $ map (\k -> choose(24,k)*choose(9,12-k)*4^(12-k)) [0..12]) /fromIntegral(choose(60,12))
0.1106278297721166
0935132人目の素数さん垢版2018/10/21(日) 19:30:38.13ID:ltcwrDDV
トランプの束がある
2〜10までの数字が描かれたカードが各スートに1枚ずつと、
ジョーカーのカードが24枚ある
全てを混ぜて無作為に切り直して12枚のカードを無作為に引いたとき
その12枚のカードのうちジョーカー以外にいずれも違う数字が
書かれている確率はいくらか

2〜10各スート一枚ずつ9×4=36枚
ジョーカー24枚
合計60枚

この中から12枚ではなく10枚のカードを取り出すとすると
数字のカード6枚、ジョーカー4枚となる

この組み合わせの確率は

(9x8x7x6x5x4)/9^6=60480/531441
               =0.11380379007
0937132人目の素数さん垢版2018/10/21(日) 20:18:12.12ID:aS+HsF0h
放物線y=x^2上の2点P,QはPQ=1を満たしている。点Pのx座標は点Qのx座標より小さいとする。

(1)P(p,p^2)とする。線分PQ上の一点Kを無作為に選び、点A(0,a)と結んで線分AKを作る。AKの長さの期待値E(p,a)をp,aで表せ。

(2)aを固定し、pの関数f(p)をf(p)=E(p,a)-(AP+AQ)/2と定義する。
f(p)と0の大小を比較せよ。
0940132人目の素数さん垢版2018/10/21(日) 21:42:58.34ID:B3jo5NYm
とりあえず、ゴリ押しで式を書き並べて整理して積分したらいいんじゃないの?
最終的には(0,0,1)か(0,0,2)からの角度で置換積分することになりそうだけど
文字3個くらい置いて計算していけばとりあえず一本道だと思う
自作?
0941132人目の素数さん垢版2018/10/21(日) 23:07:24.17ID:fSpMiCT5
>>938
Pの座標を(a,b,c)として
U(0,b,1)
W(0,b,0)
t = ∠WUP とすれば
a = sin(t)
c = 1-cos(t)

t を固定した時
0 ≦ b ≦t sin(t)
求める立体の x = a における断面の面積S(a)は t sin(t) { 1 -cos(t)}

∫_{0≦a≦1} S(a) da = ∫_{0 ≦ t ≦ π/2} t sin(t)cos(t) { 1 -cos(t)} dt
= (π/8) -(2/9)
みたいな感じ
計算は合ってるかは知らん
0942132人目の素数さん垢版2018/10/21(日) 23:28:20.59ID:ltcwrDDV
>>935
12枚の時は

  2.916{(9x8x7x6x5x4x3)/9^7}
=0.11061728395

061728395循環節の長さ9の循環小数になる
0944132人目の素数さん垢版2018/10/22(月) 00:05:10.04ID:E8LyAx4E
>>935
10枚引いた時の確率を12枚に置き換えるには

α=1458139/1500000=0.97209266666

6が循環節の長さ1の循環小数を係数としてかける

β=(9x8x7x6x5x4)/9^6=60480/531441
  =0.11380379007

とすると

αβ≒0.97209266666x0.11380379007
   ≒0.11062782976
0945132人目の素数さん垢版2018/10/22(月) 00:38:48.92ID:0aLL4RLP
>>944
30 桁計算させたけど違うよ?

Prelude Data.List Data.Ratio> let dec x y = map fst $ iterate (¥(n,(x,y))->(div (10*x) y,(mod (10*x) y,y))) (0,(x,y))
Prelude Data.List Data.Ratio> let decstr x y = concat $ map show $ dec x y
Prelude Data.List Data.Ratio> take 30 $ decstr 20413946 184528125
"011062782976849734965875798066"
Prelude Data.List Data.Ratio> take 30 $ decstr 7371811052 66636135475
"011062782977211659797262575272"
0947132人目の素数さん垢版2018/10/22(月) 02:36:43.42ID:m6H0QzkR
M_n(C)を複素成分のn次行列全体とし、C^(n^2)との対応で位相を入れます。
このときM_n(C)の元aをaの転置に写す写像が連族であることはどのように示せるでしょうか?
0948132人目の素数さん垢版2018/10/22(月) 02:40:26.39ID:DzGenx4d
自然数からなる単調増加数列{a[n]}で、以下の性質を全て満たすものが存在するか述べよ。

(1)i=1,2,...に対し、a[2^i]とa[2^i+1]は互いに素

(2)自然数jに対し,a[2j-1]とa[2j+1]をともに割り切る2以上の自然数が存在する

(3)n≧3のとき、常に漸化式a[n]=pa[n-1]+qa[n-2]が成り立つような自然数p,qが存在する。
0949イナ ◆/7jUdUKiSM 垢版2018/10/22(月) 02:48:37.81ID:GdrzxeMu
>>931
ジョーカー以外の数字がぜんぶバラバラの確率は、
3028441372×100÷1399358844975
=0.216416353(%)
0950132人目の素数さん垢版2018/10/22(月) 02:54:56.75ID:CpCVN4SV
>>948
Prelude> let x = map fst $ iterate (¥(x,y) -> (y,6*y+x)) (2,3)
Prelude> take 10 x
[2,3,20,123,758,4671,28784,177375,1093034,6735579]
0951132人目の素数さん垢版2018/10/22(月) 06:44:16.11ID:71Di82/e
>>941
ありがとうございます
0952132人目の素数さん垢版2018/10/22(月) 06:45:33.50ID:71Di82/e
>>940ありがとうございます。学校から出された課題です。
0955名無しさん@そうだ選挙に行こう! Go to vote!垢版2018/10/22(月) 10:06:42.32ID:87JVnPFu
世界的建築家とスペースシャトルのパイロットはどっちの方が空間認識能力が上ですか?
0956イナ ◆/7jUdUKiSM 垢版2018/10/22(月) 10:15:13.31ID:GdrzxeMu
>>949
>>930の実験値は、
0.216416353の半分ぐらいの値のようだ。

計算間違いしたかな。約分したとき2を忘れたとかならありうる。
0.1082081765(%)
0961132人目の素数さん垢版2018/10/22(月) 13:05:56.57ID:yi4KPPpT
>>948
 存在する。

p = q-1 とおくと 漸化式 (3) の特性根は q=p+1 と -1.

一般項は

a[n] = { (3p±1)(p+1)^{n-1} + (-1)^n・(-pp+p±1) }/(p+2),

a[1] = p と a[2] = 2p±1 は互いに素。

(2) 漸化式より、

 a[1] ≡ a[3] ≡ … ≡ a[2j-1] ≡ a[2j+1] ≡ 0 (mod p)
 a[2] ≡ a[4] ≡ … ≡ a[2j] ≡ … ≠ 0,     (mod p)

問題は (1) だが…
0964イナ ◆/7jUdUKiSM 垢版2018/10/22(月) 15:55:35.74ID:GdrzxeMu
>>960その場合の数をぜんぶ足すとこから。
ジョーカーが3枚のとき、
24C3・4^9=23・22・4^10
ジョーカーが4枚のとき、
24C4・9C8・4^8=6・23・11・7・9・4^8
ジョーカーが5枚のとき、
24C5・9C7・4^7=23・22・21・9・4・4^8
ジョーカーが6枚のとき、
24C6・9C6・4^6=23・11・7・19・3・7・4^8
ジョーカーが7枚のとき、
24C7・9C5・4^5=23・11・19・18・3・7・6・4^6
ジョーカーが8枚のとき、
24C8・9C4・4^4=23・11・19・9・17・9・2・7・4^4
ジョーカーが9枚のとき、
24C9・9C3・4^3=23・11・19・17・3・7・4^6
ジョーカーが10枚のとき、
24C10・9C2・4^2=23・11・19・17・9・6・4^4
ジョーカーが11枚のとき、24C11・9C1・4=23・19・17・3・7・9・4^3
ジョーカーが12枚のとき、24C12=23・19・13・7・4

(その場合の数)=23・22・4^10+6・23・11・7・9・4^8+23・22・21・9・4・4^8+23・11・7・19・3・7・4^8+23・11・19・18・3・7・6・4^6+23・11・19・9・17・9・2・7・4^4+23・11・19・17・3・7・4^6+23・11・19・17・9・6・4^4+23・19・17・3・7・9・4^3+23・19・13・7・4
=
0968132人目の素数さん垢版2018/10/22(月) 16:48:17.46ID:6Vwg3PAT
>>934
Wolfram先生に1000桁表示してもらいました。
https://www.wolframalpha.com/input/?i=N%5B7371811052%2F66636135475,+1000%5D

0.110627829772116597972625752724145352308187707069307653303704734386834578059690
51808972720142576665532538522410463960057551641803099326567001820869024517811745
14457390207771498921846802971432370568455448083591014999508417996234347201990107
60535104395622966609319265899400508414612559732929200153319665481396225881600016
36109285492744880700931734216839350706659508603503690802831629845503131647506453
77968626863861510570290165825376445271716141638989607087504949580811506386355308
06943152790929462285117607955040252880150985376452009801968486678661192274070722
58642261847043283987800914710833176509325475705792345845818274472796473346205856
03520099692575997182705769748121786619859500488237159434402209381725854053213310
23661077638446289265396508950236358225724373761787391527899825286199191910746081
57264239969792455915226527472930407058543486160952223197634346306605050013218822
54607142642075613254191343844583898418217807070391187027341639217411414568530694
043823525016626873949130376096438836889198..
0969132人目の素数さん垢版2018/10/22(月) 17:35:10.83ID:DzGenx4d
分子が1、分母がn桁の正整数である有理数全体からなる集合をS_nとする。
S_nの要素のうち、循環節の長さを最小とするものを1つ取り、その長さをm[n]とする。同様に循環節の長さを最大とするものについてその長さをM[n]とする。

(1)m[n]を求めよ。

(2)以下を示せ。
(a) lim[n→∞] m[n]/M[n] = 0
(b) M[n]≦M[n+1]
(c) M[n]<10^n
0970132人目の素数さん垢版2018/10/22(月) 18:32:30.96ID:Bec2HI7q
>>965
P3がΣが2個でてきてうまくできません
どうすればいいですか?
0972132人目の素数さん垢版2018/10/22(月) 19:12:47.07ID:7iHP/wTl
m、nは1以上の自然数とする。
S_n^mΣ_{k=1,...,n} k^m
の値を綺麗な式で表示する事は可能ですか?
0973132人目の素数さん垢版2018/10/22(月) 19:13:16.25ID:7iHP/wTl
訂正

m、nは1以上の自然数とする。
S_n^m = Σ_{k=1,...,n} k^m
の値を綺麗な式で表示する事は可能ですか?
0978132人目の素数さん垢版2018/10/22(月) 22:17:49.64ID:DzGenx4d
nを2以上の整数、a[0]=0とする。
整数1,2,...,nを2つのグループAとBに分ける。ただしAとBのいずれにも1つ以上の整数が入るものとする。

いま1からnまでの整数から1つを選ぶ。n個の整数のうちどれが選ばれるかは同様に確からしいものとする。
選ばれた整数がAに属していた場合、a[1]をa[1]=a[0]+0とし、Bに属していた場合a[1]=a[0]+1とする。
以下同様にして整数を選ぶことを繰り返し、a[2],a[3],...、を定める。

a[k]が偶数となる確率はk、AとBへの振り分け方、に依存する。その確率をp[k,A,B]とおく。

しかしn個の整数をどのようにAとBに振り分けても、以下が成り立つことを示せ。
lim[k→∞] p[k,A,B] = 1/2
0981132人目の素数さん垢版2018/10/23(火) 00:19:11.35ID:50P4ShkH
>>979
2∫[0→π/2]sin^n x dx
=∫[0→1]t^(n/2-1/2)(1-t)^(-1/2) dt (sin^2 x = t、2sinx cosx dx = dt、2dx = t^(-1/2)(1-t)^(-1/2) dt)
=B(n/2+1/2,1/2)
0983132人目の素数さん垢版2018/10/23(火) 04:36:12.50ID:hJH+d7Hk
数学界で一番権威ある論文誌の名前がAnnals of Mathematics(数学のアナル)
ってマジ??
0984132人目の素数さん垢版2018/10/23(火) 05:51:27.28ID:dMSY06HH
AB=c,BC=a,CA=bである△ABCの外接円をKとする。
Kの劣弧AB,BC,CA上にそれぞれ点P,Q,Rをとり、△PQRと△ABCの面積が等しくなるようにする。
このとき、△PQRの重心となり得る領域の面積を求めよ。
0985132人目の素数さん垢版2018/10/23(火) 05:58:24.35ID:dMSY06HH
∫[1→n] 1/x dx = I[n]
Σ[k=1,2,...,n] 1/k = S[n]
とおく。
次の極限が0でない定数に収束するような有理数pを求めよ。
ただしγはオイラーの定数である。

lim[n→∞] {S[n]-I[n]-γ}/n^p
0986132人目の素数さん垢版2018/10/23(火) 06:07:21.06ID:dMSY06HH
3辺の長さがa,b,c(0<a≦b≦c)の直方体ABCD-EFGHがある。
その対角線である線分AG上で点Pを動かし、4つの線分長の積PA・PG・PB・PD=Lと定める。
Lが最大となるとき、PがAGの中点と一致するかどうかを判定せよ。
0988132人目の素数さん垢版2018/10/23(火) 15:33:00.64ID:K3lfmPoe
(2)のxについての(0,0)においての偏微分係数の求め方がわかりません。教えて欲しいです。そもそも(0.0)において連続じゃなくないので存在しないかなと思ったら存在するらしく、しかも0ではありませんでした。

https://i.imgur.com/D5gVZjc.jpg
0989132人目の素数さん垢版2018/10/23(火) 15:45:55.81ID:foOj88Cn
>>985

I[n] = log(n),

S[n] - γ = ψ(n+1) = log(n) + 1/(2n) - 1/(12n^2) + 1/(120n^4) - 1/(252n^6) + …

ただし ψ(x) = Γ '(x)/Γ(x) は digamma函数である。

lim(n→∞) {S[n] - I[n] -γ}n → 1/2,

p = -1.
0990132人目の素数さん垢版2018/10/23(火) 18:06:19.95ID:foOj88Cn
>>989

〔Wolstenholmeの定理〕
素数 p に対して
p≧5 ⇒ 1 + 2^(-1) + 3^(-1) + …… + (p-1)^(-1) ≡ 0  (mod pp)
p≧5 ⇒ 1 + 2^(-2) + 3^(-2) + …… + (p-1)^(-2) ≡ 0  (mod p)
p≧7 ⇒ 1 + 2^(-3) + 3^(-3) + …… + (p-1)^(-3) ≡ 0  (mod pp)
p≧7 ⇒ 1 + 2^(-4) + 3^(-4) + …… + (p-1)^(-4) ≡ 0  (mod p)
p≧7 ⇒ 1 + 2^(-5) + 3^(-5) + …… + (p-1)^(-5) ≡ 0  (mod p)
p≧7 ⇒ 1 + 2^(-7) + 3^(-7) + …… + (p-1)^(-7) ≡ 0  (mod p^3) ?
0991132人目の素数さん垢版2018/10/23(火) 18:27:04.92ID:foOj88Cn
>>973
〔Faulhaberの定理〕

・m が奇数のとき
 S_m (n) = Σ_[k=1,...,n] k^m = {1/(m+1)} P_m(n(n+1))
 P_m は (m+1)/2 次のモニック多項式。

・m が偶数のとき
 S_m (n) = Σ_[k=1,...,n] k^m = {1/(m+1)}(n+1/2) P_m(n(n+1))
 P_m は m/2 次のモニック多項式。
0995132人目の素数さん垢版2018/10/24(水) 09:16:49.21ID:EgKzyAb9
完全に最難関大学の数学って感じだな
どこかの模試の過去問とかなのか?
0997132人目の素数さん垢版2018/10/24(水) 11:24:48.90ID:gdPWKmcN
>>993
Kは単に底面が半径aで高さaの円柱じゃないの?
0998132人目の素数さん垢版2018/10/24(水) 12:30:17.56ID:jMnLPXeV
>>992
次スレに書いとこうか?
10011001垢版Over 1000Thread
このスレッドは1000を超えました。
新しいスレッドを立ててください。
life time: 37日 16時間 12分 11秒
10021002垢版Over 1000Thread
5ちゃんねるの運営はプレミアム会員の皆さまに支えられています。
運営にご協力お願いいたします。


───────────────────
《プレミアム会員の主な特典》
★ 5ちゃんねる専用ブラウザからの広告除去
★ 5ちゃんねるの過去ログを取得
★ 書き込み規制の緩和
───────────────────

会員登録には個人情報は一切必要ありません。
月300円から匿名でご購入いただけます。

▼ プレミアム会員登録はこちら ▼
https://premium.5ch.net/

▼ 浪人ログインはこちら ▼
https://login.5ch.net/login.php
レス数が1000を超えています。これ以上書き込みはできません。

ニューススポーツなんでも実況